You are on page 1of 81

Cambridge International

AS & A Level Further Mathematics

Further Mechanics
STUDENT’S BOOK: Worked solutions

Anthony Alonzi, Chris Chisholm


Series Editor: Dr Adam Boddison

Pure Mathematics 1 International Students Book Title page.indd 1 14/11/17 10:46 pm


57736_Pi_viii.indd 1
WS TITLE PAGE_Further Mechanics.indd 1 6/18/18 10:36
31/07/18 3:21 PM
AM
1
WORKED SOLUTIONS

Worked solutions
1 Motion of a projectile
Please note: Full worked solutions are provided as an aid to learning, and represent one approach to answering
the question. In some cases, alternative methods are shown for contrast.
All sample answers have been written by the authors. Cambridge Assessment International Education bears no
responsibility for the example answers to questions taken from its past question papers, which are contained in this
publication.
Non-exact numerical answers should be given correct to 3 significant figures, or 1 decimal place for angles in
degrees, unless a different level of accuracy is specified in the question.

Prerequisite knowledge 0 = 40 sin 45° − 10t


t = 2.83 s
1 a Horizontal: Vertical:
When t = 2.828
10 cos 23° = 9.21 10 sin 23° = 3.91
y = 40 × 2.828 sin 45° − 1 × 10 × 2.8282
b Horizontal: Vertical: 2
12.6 cos 45° = 8.91 12.6 sin 45° = 8.91 Maximum height = 40.0 m
c Horizontal: Vertical: b Time of flight = 2 × 2.828 = 5.656 s
29 cos 65° = 12.3 29 sin 65° = 26.3 Range = 40 × 5.656 × cos 45°
d Horizontal: Vertical: Range = 160 m
0.2 cos 29° = 0.175 0.2 sin 29° = 0.0970 3 a Time of flight is found by finding t when y = 0
2 a v = u + at = 2 + 3 × 4 = 14 m s−1 0 = 8.8 sin 42° t − 1 × 10 t2
2
b v2 = u2 + 2as = 12 + 2 × 2 × 4 = 17, v = 4.12 m s−1 t(8.8 sin 42° − 5t) = 0
c u2 = v2 − 2as = 252 − 2 × 10 × 16 = 305, u = 17.5 m s−1 t = 0 or t = 1.178
1
d s = ut + at 2 = 5 × 5 + 0.5 × 4 × 52 = 75 m Tiger will be in air for 1.18 s
2
e v = u + at = 10 + −2 × 5 = 0 m s−1 b Range = Ut cos q
3 a v2 = u2 + 2as = 02 + 2 × 10 × 10 = 200, v = 14.1 m s−1 = 8.8 × 1.178 × cos 42°
1
b s = ut + at 2, 10 = 0 × t + 0.5 × 10 × t2, = 7.70 m
2
5t2 = 10, t = 1.41 s It will cross the river with 2.70 m to spare.
c 
Motion only occurs in two dimensions, air
4 a Maximum height, v = 0
resistance is negligible, tiger is a point mass
v = u + at, 0 = 2 + −10 × t, t = 0.2 s
4 a x = Ut cos q
b v2 = u2 + 2as, 0 = 22 + 2 × −10 × s, s = 0.2 m
x = 30 × 3 × cos 53°
Exercise 1.1A x = 54.2 m
1
1 a u = 25, q = 45° b y = Ut sin q − 2 gt2
1 1
y = Ut sin q − 2 gt2 y = 30 × 3 × sin 53° − × 10 × 32
2
1 y = 26.9 m
0 = 25t sin 45° − 2 × 10 × t2
c vx = U cos q
t(25 sin 45° − 5t) = 0
vx = 30 cos 53°
t = 0 or t = 3.536
= 18.05 m s−1
Time in air = 3.54 s
vy = U sin q − gt
b Range, x = ut cos 45°
vy = 30 × sin 53° − 10 × 3
x = 25 × 3.536 × cos 45°
= −6.041 m s−1 i.e. downwards
x = 62.5 m
So speed = 18.052 + (−6.041)2 = 19.0 m s−1
2 a u = 40, q = 45°
Time to maximum height
1
©HarperCollinsPublishers 2018 Cambridge International AS & A Level Mathematics: Further Mechanics 9780008271893

71893_P001_008.indd 1 14/06/18 10:05 PM


1 Motion of a projectile

5 First find the time of flight: b Range = 35 × 4.427 × cos 20°


1
0 = 22t sin q − × 10 × t2 = 146 m
2
0 = t(22 sin q − 5t) c 
Motion only occurs in two dimensions, air
22sin θ resistance is negligible, stone is a point mass,
t = 0 or t =
5 gravity is constant
Then find the range: 2 a u = 20, q = 0°, h = 20
x = Ut cos q Time of flight is found by finding t when y = 0
12 = 22 × 22sin θ × cos q 0 = 20t sin 0° − 1 × 10 × t2 + 20
5 2
30 t = −2 or t = 2
= sin 2θ
121
So time of flight = 2 s
2q = 14.36°
b Range = 20 × 2 × cos 0°
q = 7.18°
= 40 m
6 a When t = 4, y = 0
3 a u = 29.4, q = 30°, h = 49
0 = 25 × 4 × sin q − 1 × 10 × 42
2 Time of flight is found by finding t when y = 0
100 sin q = 80
80 4 0 = 29.4t sin 30° − 1 × 10 × t2 + 49
sin θ = = 2
100 5 5t2 − 29.4t sin 30° − 49 = 0
4 3
b If sin θ = 5 then cos θ = 5 Using the quadratic formula:
Range = Ut cos q t = −1.988 or t = 4.928
3 So particle will hit ground after 4.93 seconds.
x = 25 × 4 ×
5
= 60 b Range = 29.4 × 4.928 × cos 30°
7 a Maximum height is when vy = 0 = 125 m
0 = 10 sin 30° − 10t c vx = U cos q
t = 0.5 s = 29.4 cos 30°
Maximum height = 10 × 0.5 × sin 30° − 1 × 10 × 0.52 = 25.46 m s−1
2
= 1.25 m vy = U sin q − gt
= 29.4 sin 30° − 10 × 4.93
b Range = Ut cos q
= −34.58 m s−1 (i.e. downwards)
= 10 × 0.5 × cos 30°
Speed = 25.462 + (−34.58)2 = 42.9 m s−1
= 4.33 m
If q is an angle between the ground and the
d 
c When y = 1
direction of the particle as it hits the ground then
1 = 10 sin 30°t − 5t2
5t2 − 5t + 1 = 0 (34.58
)
q = tan−1 25.46 = 53.6°
Using the quadratic formula: 4 u = 20, q = −10°, h = 50
t = 0.2764 s and t = 0.7236 s Time of flight is found by finding t when y = 0
So it is above 1 m for 0.447 seconds.
0 = 20t sin (−10°) − 1 × 10 × t2 + 50
2
Exercise 1.2A
5t2 − 20t sin (−10°) − 50 = 0
1 a u = 35, q = 20°, h = 45 Using the quadratic formula:
Time of flight is found by finding t when y = 0 t = −3.529 or t = 2.834
1
0 = 35t sin 20° − × 10 × t2 + 45 So particle will hit the ground after 2.83 seconds.
2
5t2 − (35 sin 20°)t − 45 = 0 Range = 20 × 2.834 × cos (−10°)
Using the quadratic formula: = 55.8 m
t = −2.033 or t = 4.427
So time of flight = 4.43 s

2
©HarperCollinsPublishers 2018 Cambridge International AS & A Level Mathematics: Further Mechanics 9780008271893

71893_P001_008.indd 2 14/06/18 10:05 PM


1
WORKED SOLUTIONS

1 2
5 a When the stone lands in the sea, y = 0 b y = Ut sin q − gt
2
1
0 = U × 3 × sin 0° − 2 × 10 × 32 + h 1
0 = 40t sin q − gt2
2
0 = h − 45
80t sin q − gt2 = 0
h = 45 m
t(80 sin q − gt) = 0
b   73.52 − 452
Distance from base of cliff = t = 0 or 80 sin q − gt = 0
= 58.11 m
58.11 = U × 3 × cos 0° t = 0 or 80 sin q = gt
U = 19.4 m s−1 80sin θ
c 80 sin q = gt so t =
g
6 Find the time taken for the ball to travel 4 m
horizontally: Equating this with the expression from part a:
4 = 25t cos q 80sin θ 3
= sec θ
4 g 2
t = 25cos θ
160 sin q cos q = 3g
The height when at this point must be 2.8 m. 3g
2 sin q cos q =
2 80
× sin θ − 1 × 10 ×  
4 4
2.8 = 25 × 3g
25cos θ 2  25cos θ  sin 2θ =
80
16
2.8 = 4 tan q − sec2 q gx 2(1 + tan 2 θ )
125 4 y = x tan q −
350 = 500 tan q − 16 (1 + tan2 q ) 2U 2
16 tan2 q − 500 tan q + 366 = 0 gx 2(1 + 4)
y = 2x −
Using the quadratic formula: 2U 2
2
tan q = 0.75 or tan q = 30.5 4xU 2 5gx
y= 2 − 2
So q = 36.9° or q = 88.1° 2U 2U
Smallest possible angle is 36.9°. x(4U 2 − 5gx)
y=
2U 2
Exercise 1.3A
V 2 sin 2θ
5 a R=
g
2U sin θ 2U sin 30° 2U × 1
2 =U
1 a Time of flight = = = g g
g g 2V 2 sin θ cos θ
=
2
g
2
U 2 sin 60° U 23 U 2 3
b Range = U sin 2θ =
g g
=
2
g
=
2
2g
2 2 =
( )( )
2V 2 3 4
5 5
U sin θ U sin 30° g
c Maximum height = =
2g 2g
24V 2
U ×(
2
2)
1 2
U2 = 25g m
= =
2g 8g
b Projected at a
2 For Mei, q = 135° would be launching the particle
V 2 sin 2α
backwards. Her range is correct but negative. She R=
g
should have found the smallest value of 2q for
Projected at (90° – a)
which cos 2q = 0 which would have given an 2
acute value of q . R = V sin 2(90° − α)
g
U2
If 2q = 90°, then q = 45° and the range is g m. sin 2(90° – a ) = sin (180° – 2a)
For Xing, although the graph of sin q has a sin (180° – 2a ) = sin (180°) cos (2a)
maximum value for 90°, the graph of sin 2q has a – sin (2a) cos (180°)
maximum value for 45°, from which the = sin 2a
2
maximum range is U m. V 2 sin 2α
So R =
g g
3 a x = ut cos q These ranges are the same.
60 = 40t cos q
3
t = 2 sec q

3
©HarperCollinsPublishers 2018 Cambridge International AS & A Level Mathematics: Further Mechanics 9780008271893

71893_P001_008.indd 3 14/06/18 10:05 PM


1 Motion of a projectile

10
6 Starting from the formula y = x tan q −
(
gx 2 1 + tan 2 θ ): t = 23sin θ
2U 2 1 2
y = Ut sin q − gt + h
For A, q = 0, U = 14 m s−1 2

(
gx 2 1 + tan 2 0° ) −10x 2
0 = 23t sin q − 5t 2 + 20

( ) ( ) + 20
2
y = x tan 0° − = 10 10
2 × 14 2 392 0 = 23 sin q − 5 23sin θ
23sin θ
For B, q = 45°, U = 28 m s−1 500
0 = 30 −
y = x tan 45° −
(
gx 2 1 + tan 2 45° ) = x − 20x 2 529sin 2 θ
2 × 28 2 1568 15 870 sin2 q = 500
Since A is 60 m vertically above B, the particles 500
sin q =
15 970
collide if
q = 10.2°
2 2
x − 20x = −10x + 60 1
1568 392 b y = Ut sin q − gt2+ h
2
1568x − 20x2 = −40x2 + 94 080 0 = 23t sin 10.19° − 5t2 + 20
20x2 + 1568x – 94 080 = 0 5t2 – (23 sin 10.19°)t − 20 = 0
Using the quadratic formula: Using the quadratic formula:
x = −118 or x = 39.8 t = −1.63 or t = 2.45
So the particles collide after travelling a horizontal So the time of flight is 2.45 s.
distance of 39.8 m.
4 a vx = U cos q = 18 cos 20°
Exam-style questions vy = U sin q − gt = 18 sin 20° − 10 × 3 = 18 sin 20° − 30
18sin 20° − 30 
1 a If it reaches its maximum height after 1.5 s, Hits ground at angle of q = tan −1 
 18cos 20° 
it will return to 1.5 m after 3 s.
= 54.6°
2v sin θ
t=
g b speed = (18cos 20°)2 + (18sin 20° − 30)2
2v sin 45° = 29.2 m s−1
3=
10
v = 21.2 c 
Displacement after 3 s is the same as the height
2 2 of the starting point.
b Maximum height = v sin θ + 1.5
2g 1
  y = Ut sin q − 2 gt2
2 2
h = 21.2 sin 45° + 1.5 −h = 18 × 3 × sin 20° − 0.5 × 10 × 32
2 × 10
h = 12.7    h = 26.5 m
U 2 sin 2θ
2 a R= 1
g 5 a y = Ut sin q − 2 gt2
U 2 sin70° 2 = (35 sin 38°)t − 5t2
25 =
10
5t2 – (35 sin 38°)t + 2 = 0
2 250
U = Using the quadratic formula:
sin70°
U = 16.3 m s−1 t = 0.0949 or t = 4.215
b When t = 1 The ball hits the platform on the downward
vx = U cos q = 16.31 cos 35° = 13.36 phase of its motion so this happens after 4.22
seconds.
vy = U sin q − gt = 16.31 sin 35° − 10 × 1
= −0.6450 m s−1 b When t = 4.215
2
So speed = (13.36) + (−0.6450) = 13.4 m s−1 2 vx = U cos q = 35 cos 38° = 27.58
0.6450 vy = U sin q − gt = 35 sin 38° − 10 × 4.215
Direction = tan −1 = 2.76° below the
13.36 = −20.60 (i.e. downwards)
horizontal
speed = 27.582 + (−20.60)2 = 34.42 m s−1
3 a     x = Ut cos q
The ball strikes the platform at a speed of
10 = 23t sin q 34.4 m s−1.

4
©HarperCollinsPublishers 2018 Cambridge International AS & A Level Mathematics: Further Mechanics 9780008271893

71893_P001_008.indd 4 14/06/18 10:05 PM


1
WORKED SOLUTIONS

6 a Find t when y = 0: Using the quadratic formula:


1 t = 0.1467 or t = 1.364
y = Ut sin q − gt2 + h
2
0 = 10 × t × sin 30° − 5t2 + 19 So the ball is above 1 m for 1.22 s.
5t2 – (10 sin 30°)t − 19 = 0 b When t = 0.1467
Using the quadratic formula: vx = U cos q = 12 cos 39° = 9.326
t = −1.512 or t = 2.512 vy = U sin q − gt = 12 sin 39° − 10 × 0.1467 = 6.085
vx = U cos q = 10 cos 30° = 8.660 speed = (9.326)2 + (6.085)2 = 11.1 m s−1
vy = U sin q − gt = 10 sin 30° − 10 × 2.512 = −20.12
2
speed = (8.660) + (−20.12) = 2
21.9 m s−1
q = tan −1 ( 9.326
6.085
) = 33.1°
( )
20.12 When y = 1, the speed is 11.1 m s−1 at an
Strikes the ground at an angle of q = tan −1 angle of 33.1° above horizontal.
8.660
= 66.7° 9 a Particle A
b x = Ut cos q = 10 × 2.512 × cos 30° = 21.8 m u 2 sin 2θ u 2 sin 2θ
Range = , 60 = , 600 = u2 sin 2q
g g
so d = 21.8
Particle B
2
7 a Range = u sin 2θ x = Ut cos q, 75 = ut cos q, t =
75
g u cos θ

15 =
u 2 sin 2θ Time of flight = 75 when y = 0.
10 u cos θ
u2 sin 2q = 150 1
y = Ut sin q − 2 gt2 + h
150
u2 = 1
sin 2θ 0 = Ut sin q − gt2 + h
2
u 2 sin 2 θ 2
0 = u × u cos θ × sin θ − 5  u cos θ  + 25
Maximum height = 75 75
2g
u 2 sin 2 θ 28125
15 =
2 × 10 0 = 75 tan q − u 2 cos2 θ + 25
u sin q = 300
2 2
600 300
300 From above, u 2 = =
u2 = sin 2θ sin θ cos θ
sin 2 θ
0 = 75 tan q – 28 125 + 25
Equating:
150
=
300
sin 2θ sin 2 θ
( 300
sin θ cos θ )
cos2 θ

0 = 75 tan q – 28 125 + 25
150 sin2 q = 300 sin 2q  300cos θ 
 sin θ 
150 sin2 q − 600 sin q cos q = 0 28 125tan θ
0 = 75 tan q – + 25
150 sin q (sin q − 4 cos q ) = 0 300

150 sin q = 0 or sin q − 4 cos q = 0 0 = 22 500 tan q – 28 125 tan q + 7500

q =0 5625 tan q = 7500

or tan q = 4 q = tan −1( )


7500
5625
= 53.13°
q = 76.0° The angle of projection of the particles is 53.1°.
b u 2 = 150 b 600 = u2 sin 2q,
sin 2θ
600
= 150 u2 =
sin106.26
sin152°
u = 17.9 or, leave it as sin(2 × 53.13) so as to not lose
1 the precision
8 a y = Ut sin q − gt2
2
1 = (12 sin 39°)t − 5t2 u = 25 m s−1
5t2 – (12 sin 39°)t + 1 = 0 The initial speed of the particles is 25 m s−1.

5
©HarperCollinsPublishers 2018 Cambridge International AS & A Level Mathematics: Further Mechanics 9780008271893

71893_P001_008.indd 5 14/06/18 10:05 PM


1 Motion of a projectile

gx 2(1 + tan 2 θ ) 2U sin θ


10 a y = x tan q − x =U × × cos q
g
2U 2
2U 2 sin θ cos θ
gx 2(1 + tan 2 30) x=
y = x tan 30° − g
2 × 52

( )
U 2 sin 2θ
10x 2 1 + 1 Range =
3 3 g
y= x−
3 50 b Maximum range when sin 2q = 1
3 4 2 2q = 90°
y= x− x
3 15
q = 45°
b Range is when y = 0
Maximum range occurs when angle of
3 4 2 2
0= x− x projection is 45°, so max range = U
3 15 g
 3 4 
0 = x − x U 2 sin 2θ
 3 15  13 a R =
g
So x = 0 or
3 4
− x =0 6.52 sin2θ
3 15 2=
10
6.52 sin 2q = 20
x = 0 or x = 5 3
4 20
sin 2θ =
So range = 2.17 m 42.25
2q = 28.25 or 2q = 151.8
c y= 3 4 2
x− x +5 q = 14.13 or q = 75.9
3 15
11 a y = 4x − 0.4x2 + 1 The two possible angles are 14.1° or 75.9°
dy above the horizontal.
dx = 4 − 0.8x
2U sin θ
dy b t=
For a turning point, dx = 0 g

t = 2 × 6.5 × sin14.13 = 0.317


So x = 5
10
So maximum height = 4(5) − 0.4(5)2 + 1 = 11 m.
The ball will be in the air for 0.317 s.
b When x = 0, y = 1 gx 2 sec2 θ
14 y = x tan q −
So height of launch is 1 m. 40
c For range, y = 0 x = 1, y = 0.4
2
0 = 4x − 0.4x2 + 1 0.4 = tan q − 10sec θ
40
0 = 20x − 2x2 + 5
sec2 θ
0.4 = tan q −
2x2 − 20x − 5 = 0 4
Using the quadratic formula: 1.6 = 4 tan q − (1 + tan2 q )
x = −0.244 or x = 10.2 1.6 = 4 tan q − 1 − tan2 q
So the range is 10.2 m. tan2 q − 4 tan q + 0.6 = 0
Using the quadratic formula:
12 a For time of flight, y = 0
tan q = 0.1561 or tan q = 3.844
Substitute into y = Ut sin q − 1 gt2:
2
1 q = 8.87° or q = 75.4°
0 = Ut sin q − gt2
2 The two possible angles are 8.87° or 75.4° above
1
From which 0 = t(U sin q − gt) the horizontal.
2
1
So t = 0 or U sin q − gt = 0 15 a Particle P:
2
t = 0 is when the particle is launched so x = 20t cos 60°
2U sin θ
t= . = 10t
g
x = Ut cos q When x = 15 , t = 1.5 s

6
©HarperCollinsPublishers 2018 Cambridge International AS & A Level Mathematics: Further Mechanics 9780008271893

71893_P001_008.indd 6 14/06/18 10:05 PM


1
WORKED SOLUTIONS

b We need to know the height of P at t = 1.5 s Particles land 10 m apart


1 V 2 36 3
y = 20t sin 60° – (10)t2 So 10 = 5 +10
2
y = 10 3t − 5t 2 V 2 = 224.7
When t = 1.5 V = 15.0 m s–1.
y = 10 3(1.5) − 5(1.5)2
18 In order for the ball to pass through the window we
y = 14.73 m need 2 < y < 3.5 when the ball has travelled 12 m.
The particles collide at t = 1.5 s at a height of 14.73 m Time taken to travel 12 m?
For Q x = 14t cos 35°
y = Vt − 5t2 12 = 14t cos 35°
14.73 = V(1.5) − 5(1.5)2 t = 1.046 s
V = 17.3 At this time,
c Max height of P y = 14 × 1.046 × sin 35° – 5 × 1.0462 = 2.93 m
2 2
u sin θ This means the ball will pass through the window.
H =
2g
opposite
19 a tan q = which is the gradient
20 2sin 2 60° adjacent
=
20 of the curve
= 15 m dy
Since y = 5x − x2, dx = 5 − 2x = tan q
16 a Max height of P
b When q = 10°,
u 2sin 2 θ
H= 5 – 2x = tan 10°
2g
1
322sin 2 55° x = (5 − tan 10°) = 2.412
= 2
20
When x = 2.412, y = 5(2.412) – (2.412)2
= 34.3557… = 34.4 m (to 3 s.f.)
= 6.24 m
1
b of max height = 11.45 m (to 4 s.f.)
3 Mathematics in life and work
2
of max height = 22.90 m (to 4 s.f.)
3 1 u = 5, q = 30°, with block, h = 0.6
For P,
Without block:
y = 32t sin  55° − 5t2
U 2 sin 2θ 52 sin 60°
When y = 11.45, R= = 10 = 2.165 m
g
11.45 = 32t sin  55° − 5t2 With block:
Solving using the quadratic formula gives: Time of flight, y = 0
t = 0.4809 or t = 4.762 1
y = Ut sin q − gt2 + h
2
When y = 22.90, 0 = (5 sin 30°)t − 5t2 + 0.6
22.90 = 32t sin  55° − 5t2
5t2 − 2.5t − 0.6 = 0
Solving using the quadratic formula gives:
Using the quadratic formula:
t = 1.108 or t = 4.135
t = −0.1772 or t = 0.6772
So total time between this range = (1.108 – 0.4809)
So time of flight = 0.677 s.
+ (4.762 – 4.135) = 1.25 s (to 3 s.f.)
x = ut cos q
17 Range of P
  = 5 × 0.6772 × cos 30°
122 sin 60° 36 3
R= = = 2.932 m
10 5
Range of Q So he can dive 0.767 m further using the
2
V sin 90° V 2 starting block.
R= =
10 10

7
©HarperCollinsPublishers 2018 Cambridge International AS & A Level Mathematics: Further Mechanics 9780008271893

71893_P001_008.indd 7 14/06/18 10:05 PM


1 Motion of a projectile

2 Without the block:


2U sinθ
Time of flight = = 2 × 5 × sin30° = 0.5 s
g 10
So speed at landing:
vx = U cos q
= 5 cos 30°
= 4.330 m s−1
vy = U sin q − gt
= 5 sin 30° − 10 × 0.5
= −2.5 m s−1
2.5
Enters the water at an angle of tan −1 = 30.0°
4.33
With the block:
Time of flight = 0.6772 s
vx = U cos q
= 5 cos 30°
= 4.330 m s−1
vy = U sin q − gt
= 5 sin 30° − 10 × 0.6772
= −4.272 m s−1
−1 4.272
Enters the water at an angle of tan 4.330 = 44.6°
He will enter the water at the smallest angle
when diving from the poolside.
3 u = 5.2, q = 30°, h = 0.6
Time of flight, y = 0
1
y = Ut sin q − gt2 + h
2
0 = (5.2 sin 30°)t − 5t2 + 0.6
5t2 − 2.6t − 0.6 = 0
Using the quadratic formula:
t = −0.1731 or t = 0.6931
So time of flight = 0.693 s
x = ut cos q
= 5.2 × 0.6931 × cos 30°
= 3.121 m
So he will be able to travel an extra 18.9 cm.

8
©HarperCollinsPublishers 2018 Cambridge International AS & A Level Mathematics: Further Mechanics 9780008271893

71893_P001_008.indd 8 14/06/18 10:05 PM


2
WORKED SOLUTIONS

2 Equilibrium of a rigid body


Please note: Full worked solutions are provided as an aid to learning, and represent one approach to answering
the question. In some cases, alternative methods are shown for contrast.
All sample answers have been written by the authors. Cambridge Assessment International Education bears no
responsibility for the example answers to questions taken from its past question papers, which are contained in this
publication.
Non-exact numerical answers should be given correct to 3 significant figures, or 1 decimal place for angles in
degrees, unless a different level of accuracy is specified in the question.
Prerequisite knowledge 5 Both methods are valid. Both methods lead to an
answer of the form Fd sin q. Chen’s method is more
1 W = mg efficient in this example.
W = 0.125 × 10 = 1.25 N 6 a Clockwise: 8 × 3 sin 30° = 12 N m
2 Horizontal component = 9 cos 52° = 5.54 N Anticlockwise: 10 × 3 sin 60° = 25.98 N m
Vertical component = 9 sin 52° = 7.09 N Overall turning moment = 25.98 − 12 = 14.0 N m
anticlockwise
3 F = µR
b The turning moment will still be
7.2g
µ= F = = 0.4 anticlockwise but smaller (11.0 N m).
R 18g
7 a 13 × 4 = 6 × 5 + 11d
4 a Resolving vertically, R + 20 sin 30° = 5g, R = 40 N
d=2
b F = µR = 0.25 × 40 = 10 N b (15 − 11) × 2 = 8 N m anticlockwise
c Resultant force = ma 8 Let x m be distance of the man from the centre of
20 cos 30° − 10 = 5a the seesaw.
a = 1.46 m s−2 60g × x = 36g × 2
Exercise 2.1A x = 1.2 m
9 a Taking moments anticlockwise
1 a i 6 × 5 = 30 N m ii 3.5 × 12 = 42 N m 7 × 4 + 9 × 6 = 82 N m
b i 28 ÷ 4 = 7 N ii 15 ÷ 4 = 3.75 N Turning moment = 82 N m anticlockwise
b i Let force = F N
2 Clockwise: 12 × 4 = 48 N m
2 × F = 82
Anticlockwise: 17 × 3 = 51 N m
F = 41 N
Overall turning moment = 51 − 48 = 3 N m F acts vertically downwards.
anticlockwise
ii R + 9 = 7 + 41
3 a 3 × 9 = 27 N m clockwise R = 39 N
b 5 × 4 + 3 × 5 = 35 N m clockwise 10 The value of U is correct but the working is incorrect.
c 5 × 9 = 45 N m clockwise She has taking moments incorrectly with cos and sin.
d 5 × 15 − 3 × 6 = 57 N m clockwise She has then also evaluated the compound
angle formulae incorrectly.
4 a Clockwise: 7 × 3 = 21 N m
a Resolving vertically:
Anticlockwise: 7 × 4 = 28 N m
T cos a = U cos a
Overall turning moment = 28 − 21 = 7 N m
T=U
anticlockwise
q=U
b 7 × 3 + 7 × 4 = 49 N m clockwise
b Taking moments about O:
c Clockwise: 7 × 3 = 21 N m
T × 8 sin a = U × 3 cos a
Anticlockwise: 7 × 4 = 28 N m q × 8 sin a = q × 3 cos a
Overall turning moment = 28 − 21 = 7 N m 3
tan a =
anticlockwise 8
d 7 × 3 + 7 × 2 sin 45° = 30.9 N m anticlockwise ()
a = tan−1 3 = 20.6°
8
9
©HarperCollinsPublishers 2018 Cambridge International AS & A Level Mathematics: Further Mechanics 9780008271893

71893_P009_024.indd 9 15/06/18 8:00 PM


2 EQUILIBRIUM OF A RIGID BODY

c Resolving horizontally: c 7.6x = 1.4 × 5 + 4.3 × 2 + 1.9 × 1


V = T sin a + U sin a 7.6y = 1.4 × 1 + 4.3 × 6 + 1.9 × 1
V = q sin a + q sin a x = 2.30 y = 3.83
V = 2q sin a So ( x , y ) = (2.30, 3.83)
3 6q 73 d 680x = 230 × 2 + 342 × 4 + 108 × 9
V = 2q × =
73 73
Exercise 2.2A x = 4.12
680 y = 230 × 3 + 342 × 1 + 108 × 2
1 a 3.7x = 0.2 × 2 + 0.7 × 1 + 0.8 × 0.5 + 1.2 × 0.2 y = 1.84
3.7x = 1.74 So ( x , y ) = (4.12, 1.84)
x = 0.470 m from A
2 12.5x = 0 × 5 + 4 × 2.5 + 4 × 2 + 0 × 3
b 9x = 0.9 × 2 + 2.1 × 3 + 2.2 × 4
x = 1.44
9x = 16.9 12.5 y = 0 × 5 + 0 × 2.5 + 2.5 × 2 + 2.5 × 3
x = 1.88 m from A
y =1
c 4mx = 0.3 × m + 1 × m + 1.2 × m + 1.3 × m So ( x , y ) = (1.44, 1)
4mx = 3.8 m 3 4mx = 2 × m + 6 × m + 2 × m + 6 × m
x = 0.95 m from A x =4
d 10.5mx = 0.2 × 2m + 0.7 × 6m + 0.8 × 2m + 1.2 × 0.5m 4m y = 5 × m + 5 × m + 8 × m + 8 × m
10.5mx = 6.8m y = 6.5
x = 0.648 cm from A So ( x , y ) = (4, 6.5)
2 12x = 0 × 2 + 0.4 × 2 + 0.8 × 2 + 1.2 × 2 + 1.6 × 2 + 2 × 2 4 (10 + m) × 4 = 2 × 1 + 3 × 8 + 5 × 4 + 2 × m
12x = 12 40 + 4m = 46 + 2m
x = 1 m from A m = 3 kg
3 (3m + 3.2) × 1.2 = 0.2 × m + 1.3 × 2m + 2.5 × 3.2 13 y = 2 × 1 + 3 × 1 + 5 × 5 + 3 × 3
3.6m + 3.84 = 2.8m + 8 y =3
0.8m = 4.16 5 (5 + m + n) × 1.6 = 3 × 0 + 4 × n + 4 × 2 + 0 × m
m = 5.2 kg 8 + 1.6m + 1.6n = 4n + 8
4 450 × 40 = 200 × 10 + 100 × 30 + 150 × d 2.4n − 1.6m = 0 1
18 000 = 150d + 5000 (5 + m + n) × 2.5 = 3 × 0 + n × 0 + 5 × 2 + 5 × m
150d = 13 000 12.5 + 2.5m + 2.5n = 10 + 5m
d = 86.7 cm from A 2.5n − 2.5m = −2.5 2
5 (3m + 3) × 6 = m × 1 + 2m × 7 + 3 × 9 Solving simultaneously:
18m + 18 = 15m + 27 Substituting m = 1.5n into 2
3m = 9 gives n = 2 and m = 3
m = 3 kg Exercise 2.3A
Exercise 2.2B 1 a
1 a 8x = 3 × 4 + 5 × 5 8y = 3 × 2 + 5 × 5
x-coordinate y-coordinate
x = 4.625 y = 3.875 of centre of of centre of
So ( x , y ) = (4.63, 3.88) Shape Area mass mass

b 10x = 2 × 5 + 5 × 6 + 3 × 5 Large rectangle 54 6.5 5


10y = 2 × 3 + 5 × 2 + 3 × 0 Small rectangle 12 9 9.5
x = 5.5 y = 1.6
Lamina 66 x y
So ( x , y ) = (5.5, 1.6)
10
©HarperCollinsPublishers 2018 Cambridge International AS & A Level Mathematics: Further Mechanics 9780008271893

71893_P009_024.indd 10 15/06/18 8:00 PM


2
WORKED SOLUTIONS

66x = 54 × 6.5 + 12 × 9 66 y = 54 × 5 + 12 × 9.5 e


x = 6.95 (to 3 s.f.) y = 5.81 (to 3 s.f.) Shape Area x-coordinate y-coordinate
of centre of of centre of
b
mass mass
Shape Area x-coordinate y-coordinate
Large 72 9 7
of centre of of centre of
rectangle
mass mass
Small 18 4.5 13
Large 60 9 4.5
rectangle
rectangle
Circle π 10 8
Medium 21 13.5 10.5
rectangle Lamina 90 − π x y
Small 15 4.5 9.5
rectangle (90 − π) x = 72 × 9 + 18 × 4.5 − 10 × π
Lamina 96 x y (90 − π) y = 72 × 7 + 18 × 13 − 8 × π
x = 8.03 (to 3 s.f.) y = 8.21 (to 3 s.f.)
96x = 60 × 9 + 21 × 13.5 + 15 × 4.5 f
96 y = 60 × 4.5 + 21 × 10.5 + 15 × 9.5 Shape Area x-coordinate y-coordinate
x = 9.28 (to 3 s.f.) y = 6.59 (to 3 s.f.) of centre of of centre of
mass mass
c
Rhombus 32 8 10
x-coordinate y-coordinate
of centre of of centre of Rectangle 4 8 10.5
Shape Area mass mass
Lamina 28 x y
Large 90 9 8.5
rectangle
28x = 32 × 8 − 4 × 8 28 y = 32 × 10 − 4 × 10.5
Medium 10 11 8.5
x =8 y = 9.93 (to 3 s.f.)
rectangle
Small 6 6 10.5 2 From diagram, x = 6
rectangle Using formula with r = 2, a = p
2
Lamina 74 x y 2 × 2sin p
y =2+ 2 = 2.85
3p
74x = 90 × 9 − 10 × 11 − 6 × 6 2
3 Distance of centre of mass from (4, 2)
74 y = 90 × 8.5 − 10 × 8.5 − 6 × 10.5
x = 8.97 (to 3 s.f.) y = 8.34 (to 3 s.f.) 2 × 4 × sin p
= 4 = 2.401
3× p
d 4
p
x-coordinate y-coordinate So x = 4 + 2.401 cos = 5.70
4
of centre of of centre of p
and y = 2 + 2.401 sin = 3.70
Shape Area mass mass 4

Rectangle 60 8 7 4 Using formula with r = 10, a = p


12
Triangle 15 8 11 2 × 10 sin p
Distance from O to centre of mass = 12
Lamina 75 x y 3p
12
= 6.59 cm
75x = 60 × 8 + 15 × 8 75 y = 60 × 7 + 15 × 11
5 Length of median = 32 − 1.52 = 2.60
x =8 y = 7.8 2
So centre of mass lies × 2.60 = 1.73 cm along
3
each median from the vertex.

11
©HarperCollinsPublishers 2018 Cambridge International AS & A Level Mathematics: Further Mechanics 9780008271893

71893_P009_024.indd 11 15/06/18 8:00 PM


2 EQUILIBRIUM OF A RIGID BODY

Exercise 2.4A 6 Assuming the origin is at the bottom left of the


cross-section.
3 3
1 r = × 4 = 1.5
8 8
Shape Area x-coordinate y-coordinate of
The centre of mass lies 1.5 cm above the centre
of centre of centre of mass
of the base of the hemisphere.
mass
2 a 3 h = 3 × 16 = 12 so centre of mass lies 12 cm
4 4 Large 1500 15 25
below the vertex. This means it is 4 cm above rectangle
the base.
Small 750 45 12.5
3 3
b h = × 16 = 12 so centre of mass lies 12 cm rectangle
4 4
below the vertex. This means it is 4 cm above Cross- 2250 x y
the base. section
c The centre of mass of a solid cone does not
depend on its radius, only its height. 2250x = 1500 × 15 + 750 × 45
2250 y = 1500 × 25 + 750 × 12.5
3 x and y will lie directly above the centre of the
base of the cuboid. x = 25 y = 20.8 (to 3 s.f.)
z = 25 (due to the symmetry of the step)
Shape Volume z-coordinate of
7 Assuming the centre of the flat base of the cylinder
centre of mass
is (y, z) = (0, 0)
Cuboid 128 4
Shape Volume x-coordinate of centre
8 + 1 × 6 = 9.5 of mass
Pyramid 32
4
r
Cylinder πr2 r = πr3 2
Solid 160 z

Hemisphere 2 pr 3 3 19
160z = 128 × 4 + 32 × 9.5 3 2r + r= r
8 8
z = 5.1
5 3
The centre of mass is 5.1 cm above the base. Solid pr x
3
4 x = 0 and y = 0 due to the symmetry of the solid.
5 3 r 2 19
pr x = pr 3 × + pr 3 × r
3 2 3 8
Shape Volume z-coordinate of
centre of mass 5 3 25 4
pr x = pr
3 12
Cone 3 13
261.8 × 10 = 7.5 x= r
4 8

Hemisphere 3 The centre of mass is 13 r from the end of the


261.8 10 + × 5 = 11.88 8
8 sculpture.
Solid 523.6 z 8 x = 12.5 and y = 12.5 because of symmetry.

523.6z = 261.8 × 7.5 + 261.8 × 11.88 Shape Area z-coordinate of


z = 9.69 centre of mass

3 Large square 625 12.5


5 r = 6 so r = 16 cm
8
Small square 25 2.5
Cross-section 600 z

600z = 625 × 12.5 − 25 × 22.5


z = 12.1 (to 3 s.f.)

12
©HarperCollinsPublishers 2018 Cambridge International AS & A Level Mathematics: Further Mechanics 9780008271893

71893_P009_024.indd 12 15/06/18 8:00 PM


2
WORKED SOLUTIONS

Exercise 2.5A e
R
1 a
R

25g N W 40g N
20g N 30g N 40g N 2m
4m
4m
f
8m R2

b
T T
12 m
A B R1 7g N
C
0.8 m 2.5 m
5m
9g N
2 a It is rigid, straight and has uniform mass.
6m
b Moments about C:
15 m
50g × 3 + 60g × 0.5 = 30g × 2 + RD × 2
c RD = 60g = 600 N
5g N 3g N Resolving vertically:
RC + RD = 30g + 60g + 50g
RC + 60g = 140g
A B
RC = 80g = 800 N
3 a Let x m be the distance of the centre of mass
W from A.
6m Moments about C:
2g × 4 + 20g(x − 1) = 30g × 1
d
20(x − 1) = 22
RC RD
x − 1 = 1.1
x = 2.1 m
A B b Resolving vertically:
C D
RC = 30g + 20g + 2g
20g N RC = 52g = 520 N
2m 6m 4m 4 Let A be the end of the pole that he is holding and
12 m let B represent the position of his other hand.
Moments about A:
6m
5g × 2 = RB × 0.8
RB = 12.5g = 125 N
Resolving vertically:
RA acts in the same direction as W (also
described in the solution to 1f).
RA + 5g = 12.5g
RA = 75 N
13
©HarperCollinsPublishers 2018 Cambridge International AS & A Level Mathematics: Further Mechanics 9780008271893

71893_P009_024.indd 13 15/06/18 8:00 PM


2 EQUILIBRIUM OF A RIGID BODY

5 a Let JL = x m d The boulders are particles with their masses


Moments about L: concentrated at a single point so that the
perpendicular distances are exact.
40g(12 − x) = 24gx + 16g(x − 6)
8 a Let the weight of the rod be W and the distance
480 − 40x = 24x + 16x − 96
of the centre of mass of the rod from C be x m.
576 = 80x
For the 10 kg object:
x = 7.2 m 5
Resolving vertically: W + 10g = RA1 + RB1 = 4 RB1
b Resolving vertically: RB1 = 4 (W + 10g)
5
RL = 24g + 16g + 40g Moments about A:
RL = 80g = 800 N W(x − 3) + 10g × 7 = RB1 × 5
c The masses are concentrated at a single point 4
W(x − 3) + 70g = (W + 10g) × 5
5
and the centre of mass of the beam is at its W(x − 3) + 70g = 4W + 40g
midpoint.
W(x − 3) = 4W − 30g
6 a Let string be attached at A. For the 5kg object:
Moments about A: 7
Resolving vertically: W + 5g = RA2 + RB2 = RB2
3
7g(0.375 − d) = 5gd + 3g(0.375 + d) 3
RB2 = (W + 5g)
7
2.625 − 7d = 5d + 1.125 + 3d Moments about A:
1.5 = 15d W(x − 3) = RB2 × 5 + 5g × 3
d = 1 m or 10 cm 3
10 W(x − 3) = (W + 5g) × 5 + 15g
7
b T = 3g + 5g + 7g 15
W(x − 3) = (W + 5g) + 15g
T = 15g = 150 N 7
15
7 a Resolving vertically: Hence 4W − 30g = (W + 5g) + 15g
7
RA + RB = 64g + 16g 28W − 210g = 15W + 75g + 105g
RA + 3RA = 80g 13W = 390g
4RA = 80g W = 30g = 300 N
RA = 20g The weight of the rod is 300 N.
RB = 20g × 3 = 60g = 600 N b W(x − 3) = 4W − 30g
b Let AB = x 30g(x − 3) = 120g − 30g
Moments about A: 30g(x − 3) = 90g
64g × 3 + 16g × 6 = 60gx x−3=3
288 = 60x x=6
x = 4.8 m The distance of the centre of mass from C is 6 m.
c Resolving vertically: Exercise 2.5B
RA + RB = 64g + Mg
1 Since TK = 0, the position of K is not required. Let
RA + 5RA = 64g + Mg
the position of the particle from A be x m.
6RA = 64g + Mg
1 Moments about J:
RA = (64g + Mg)
6 6g × d = 9g(d − x)
Moments about B:
6gd = 9gd − 9gx
Mg × 1.2 + RA × 4.8 = 64g × 1.8
1 9x = 3d
Mg × 1.2 + (64g + Mg) × 4.8 = 64g × 1.8 1
6 x= d
7.2M + 307.2 + 4.8M = 691.2 3
1
The particle is d m from A.
12M = 384 3

M = 32

14
©HarperCollinsPublishers 2018 Cambridge International AS & A Level Mathematics: Further Mechanics 9780008271893

71893_P009_024.indd 14 15/06/18 8:00 PM


2
WORKED SOLUTIONS

2 Let the distance of the centre of mass from A be x m. M(x − 2) = 54


RC = 0 Mx − 2M = 54
Moments about D: Mx = 2M + 54
4g × 0.7 = 10g (1.4 − x) After block placed, RE = 0.
2.8 = 14 − 10x Moments about F:
10x = 11.2 72g × 1.5 = Mg(5 − x)
x = 1.12 108 = M(5 − x)
The distance of the centre of mass from A is 1.12 m. 108 = 5M − Mx
3 a Since the rod is on the point of tilting about D, Mx = 5M − 108
the reaction force at E is 0 N. Hence 2M + 54 = 5M − 108
b Moments about D: 162 = 3M
mg × 6 + 70g × 1.5 = 50g × 3 M = 54
6m + 105 = 150 Substituting M = 54 into Mx = 2M + 54:
6m = 45 54x = 108 + 54
m = 7.5 54x = 162
4 a Moments about P: x=3
20g × 2 = RQ × 5 The distance of the rod’s centre of mass from A
RQ = 8g = 80 N is 3 m.
Resolving vertically: Exercise 2.6A
RP + RQ = 20g
1 Let the reaction force acting between the ladder and
RP + 8g = 20g the floor be RF, the friction force acting between the
RP = 12g = 120 N ladder and the floor be FF and the reaction force
acting between the ladder and the wall be RW.
b RP = 0
a Resolving vertically:
Moments about Q:
RF = Mg N
Mg × 1 = 20g × 3
b F = µR
M = 60
FF = 0.4RF = 0.4Mg N
5 a RV = 0
c Resolving horizontally:
Moments about U:
RW = FF = 0.4Mg N
5g × 8 + W × 3 = 70g × 1
W = 10g = 100 d Let the length of the ladder be 2a m.
Moments about point of contact of ladder
b Let the mass of the added load be M kg.
with floor:
RU = 0
Mga sin a = RW × 2a cos a
Moments about V:
Mg sin a = 0.4Mg × 2 cos a
(M + 5)g × 3 = 10g × 2 + 70g × 6
tan a = 0.8
3M + 15 = 20 + 420
a = tan−1(0.8) = 38.7°
3M = 425
425 2 Let distance of the centre of mass of the ladder
M=
3 from B be x.
The mass of the load that has been added is
425 kg. Let the reaction force acting between the
3 ladder and the ground be RB, the friction force
6 Let x m be the distance of the rod’s centre of mass acting between the ladder and the ground be
from A. FB and the reaction force acting between the
Before block placed: ladder and the wall be RA.
Moments about E: Resolving horizontally:
Mg(x − 2) = 18g × 3 RA = FB
15
©HarperCollinsPublishers 2018 Cambridge International AS & A Level Mathematics: Further Mechanics 9780008271893

71893_P009_024.indd 15 15/06/18 8:01 PM


2 EQUILIBRIUM OF A RIGID BODY

Resolving vertically: 4
27g + F = RC
50g = RB 3 C

Moments about B: 27g + 4 (324g − 6RC) = RC


3
50g × x cos 70° = RA × 8 sin 70° 27g + 432g − 8RC = RC
50g × x cos 70° = FB × 8 sin 70° 459g = 9RC
1 RC = 51g N
50g × x cos 70° = RB × 8 sin 70°
4
1 FC = 324g − 6 × 51g = 324g − 306g = 18g N
50g × x cos 70° = 4 × 50g × 8 sin 70° F 18g 6
µC = C = =
x = 2 tan 70° RC 51g 17
x = 5.49 m 5 Let the reaction force acting between the ladder
and the ground be RB, the friction force acting
3 Let the reaction force acting between the ladder
between the ladder and the ground be FB and the
and the ground be RG, the friction force acting
reaction force acting between the ladder and the
between the ladder and the ground be FG and the
wall be RA. Let the maximum distance the man can
reaction force acting between the ladder and the
climb safely be x m.
wall be RW.
Resolving horizontally:
Resolving horizontally:
FB = RA
RW = FG
Resolving vertically:
Resolving vertically:
RB = 60g + 70g = 130g
36g + 45g = RG
Moments about B:
RG = 81g
60g × 1.75 cos a + 70g × x cos a = RA × 3.5 sin a
Moments about G:
105g + 70gx = 3.5RA tan a
45g × 2 sin 30° + 36g × 3 sin 30° = RW × 6 cos 30°
70
198g sin 30° = 6RW cos 30° 105g + 70gx = R
13 A
33g tan 30° = RW 13
RA = (105g + 70gx)
70
RW = 11g 3 RA = 19.5g + 13gx
FG = µRG FB ⩽ µRB
F R 11g 3 11 3 2
µ= G = W = = FB ⩽ RB
RG 81g 81g 81 5
2 FB
4 Let the reaction force acting between the ladder 5 ⩾ RB
and the ground be RC, the friction force acting
2 RA
between the ladder and the ground be FC, the
5 ⩾ 130g
reaction force acting between the ladder and the
wall be RD and the friction force acting between 2 19.5g + 13gx
5⩾ 130g
the ladder and the wall be FD.
1 2
F D = RD 19.5g + 13gx ⩽ × 130g
6 5
Resolving horizontally: 19.5g + 13gx ⩽ 52g
FC = RD = 6FD 13gx ⩽ 32.5g
Resolving vertically: x ⩽ 2.5
FD + RC = 54g The man can safely climb 2.5 m.
FC = 6(54g − RC) = 324g − 6RC 6 Let the reaction force acting between the ladder
Moments about D: and the ground be RC, the friction force acting
between the ladder and the ground be FC, the
54g × a cos a + FC × 2a sin a = RC × 2a cos a
reaction force acting between the ladder and the
54g + 2FC tan a = 2RC wall be RD and the friction force acting between
27g + FC tan a = RC the ladder and the ground be FD.
4 4 FD = µRD and FC = µRC
Given that sin a = , tan a = .
5 3
16
©HarperCollinsPublishers 2018 Cambridge International AS & A Level Mathematics: Further Mechanics 9780008271893

71893_P009_024.indd 16 15/06/18 8:01 PM


2
WORKED SOLUTIONS

Resolving horizontally: c Resolving vertically:


FC = RD RA + RC cos 25° = 12g
Resolving vertically: RA = 12g − 81.57 cos 25°
RC + FD = 40g RA = 46.07 N
RC + µRD = 40g d Magnitude = 34.472 + 46.072 = 57.5 N

( )
RC = 40g − µRD
Direction = tan−1 46.07 = 53.2° to the horizontal
RC = 40g − µFC 34.47
RC = 40g − µ(µRC) 2 Let the magnitude of the components of the
reaction at A be X and Y and of the tension in the
RC = 40g − µ2RC
cable be T.
(1 + µ2)RC = 40g a Moments about A:
40g
RC = 8g × 0.8 = T sin 60° × 1.6
1 + µ2
T sin 60° = 4g
Moments about D:
T = 46.2 N
40g × a cos a + FC × 2a sin a = RC × 2a cos a
40g + 2FC tan a = 2RC b Resolving horizontally:
20g + FC tan a = RC X = T cos 60° = 23.09 N
4 4 Resolving vertically:
Given that sin a = , tan a = .
5 3 T sin 60° + Y = 8g
4
20g + F = RC 4g + Y = 8g
3 C
Y = 4g
20g + 4 µRC = RC
3
Magnitude = 23.092 + ( 4g ) = 46.2 N
2

20g = RC 1 − µ(
4
3 ) Direction = tan−1 
4g 
= 60.0° to the
20g  23.09 
RC = horizontal
1 − 43 µ
40g 20g 3 Let the friction at the wall be F N, the reaction at
Hence =
1 + µ2 1 − 3 µ
4
the wall be R N and the tension be T N.

(
4
)
40g 1 − 3 µ = 20g (1 + µ2) a From the right-angled triangle, cos a = ,
12
13
from which sin a = 5 .
2 − 8 µ = 1 + µ2 13
3
6 − 8µ = 3 + 3µ2 Moments about A:

3µ2 + 8µ − 3 = 0 Mg × 4a = T sin a × 12a


1
(3µ − 1)(µ + 3) = 0 T sin a = Mg
3
5 1
µ = 1 or −3 T×
13 3
= Mg
3
1 13
Since 0 ⩽ µ ⩽ 1, µ = T = Mg
3 15
Exercise 2.6B b Resolving vertically:
F + T sin a = Mg
1 Let the reaction at the ground be RA and the
1
friction at the ground be FA. F + Mg = Mg
3
a Moments about A: 2
F = Mg
12g × 0.9 cos 25° = RC × 1.2 3
Resolving horizontally:
RC = 81.6 N
R = T cos a
b Resolving horizontally: 13 12 4
R = Mg × = Mg
FA = RC sin 25° = 81.57 sin 25° = 34.5 N 15 13 5
F ⩽ µR

17
©HarperCollinsPublishers 2018 Cambridge International AS & A Level Mathematics: Further Mechanics 9780008271893

71893_P009_024.indd 17 15/06/18 8:01 PM


2 EQUILIBRIUM OF A RIGID BODY

F 7 a Resolving horizontally:
µ⩾ R
TA cos 20° = TB cos 50°
2 Mg
3 TB cos50°
µ⩾ 4 TA =
Mg cos20°
5 Resolving vertically:
5
µ⩾ 6 TA sin 20° + TB sin 50° = 2g
4 Let the magnitude of the components of the TB cos50°
sin 20° + TB sin 50° = 2g
reaction at A be X and Y. cos20°

Moments about A: TB(cos 50° tan 20° + sin 50°) = 2g


2g
6g × 5a = 2F × 2a + F × 10a TB = = 2g = 20.0 N
cos50° tan 20° + sin 50°
14F = 30g
TA = 20cos50° = 13.7 N
F = 300 N cos20°
14 b Moments about A:
Resolving vertically:
2gx = TB sin 50° × 3
Y + F = 6g
x = 3 sin 50° = 2.30
Y = 6g − 300 = 540 N
14 14 Exercise 2.7A
Resolving horizontally:
X = 2F = 600 N 1 a At the point of sliding, F = μR

( )
14
540 R = 12g so F = 0.32 × 12g = 38.40 N
Direction = arctan = arctan 0.9 to the horizontal
600
P = F = 38.4 N
5 Let the friction at the ground be F N and the
reaction at the ground be R N. b R = 12g − P sin 15°
a Moments about A: So F = 0.32(12g − P sin 15°)
7.5g × 2 cos a = 49 × 2.5 So P cos 15° = 0.32(12g − P sin 15°)
cos a = 0.8167 0.32 × 12g
P = (cos15° + 0.32sin15°) = 36.6 N
a = 35.2°
b Resolving horizontally: 2 a Let A be the bottom right corner, which is the
pivot point.
F = 49 sin a = 28.28 N
Moments (A) = 2g × 5 − 20P
Resolving vertically:
At the point of toppling, moments about A = 0
R + 49 cos a = 7.5g
So 10g − 20P = 0
R = 34.98 N
28.27 P = 5N
µ= F = = 0.808
R 34.98 b At the point of sliding, F = μR
6 Let the magnitude of the components of the
R = 2g so F = 0.3 × 2g = 6 N
reaction at A be X and Y and the thrust be T.
Let AB be x m. P = F = 6N
a Moments about A: c The cuboid would topple first.
3
0.6g × 1 x = T cos 75° × 4 x 3 a Sliding: At the point of sliding, F = μR
2
T cos 75 = 0.4g F = 0.2 × 2.5g = 5 N
0.4g P = F = 5N
T= = 15.5 N
cos75°
Toppling: Let A be the bottom right corner,
b Resolving vertically:
which is the pivot point.
Y + 0.4g = 0.6g
Moments (A) = 2.5g × 20 − 50P
Y = 0.2g = 2.00 N
At the point of toppling, moments about A = 0
Resolving horizontally:
2.5g × 20 − 50P = 0
X = T sin 75° = 14.93 N
P = 10 N
Magnitude = 22 + 14.922 = 15.1 N
So cuboid will slide first.
18
©HarperCollinsPublishers 2018 Cambridge International AS & A Level Mathematics: Further Mechanics 9780008271893

71893_P009_024.indd 18 15/06/18 8:01 PM


2
WORKED SOLUTIONS

b Sliding: At the point of sliding, F = μR 2 Let the distance of the centre of mass from A be x m.
F = 0.2(2.5g + P sin 10°) Moments about V:
P cos 10° = 0.2(2.5g + P sin 10°) 6mgd = 8mg (4d − x)
0.2 × 2.5g 6d = 32d − 8x
P= = 5.26 N
cos10° − 0.2sin10°
8x = 26d
Toppling: Let A be the pivot point.
x = 3.25d
Moments (A) = 2.5g × 0.2 − 0.5P cos 10°
At the point of toppling moments about A = 0 3 Resolving vertically:
2.5g × 0.2 − 0.5P cos 10° = 0 RX + RY = 7g + 5g + 9g
P = 10.2 N RX + 2RX = 7g + 5g + 9g
So cuboid will slide first. 3RX = 21g
RX = 7g, RY = 14g
4 a Sliding:
Let the distance WY be x m.
Resolving horizontally: F = 4g sin a
Moments about W:
Resolving vertically: R = 4g cos a
5g × 7 + 9g × 14 = 7g × 2 + 14gx
At the point of sliding, F = μR
161 = 14 + 14x
4g sin a = 0.3 × 4g cos a
14x = 147
tan a = 0.3
x = 10.5
a = 16.7°
WY = 10.5 m
b Toppling occurs when the centre of mass is
directly above the lower corner of the cuboid. 4 a The frustum is a cone of base 10 cm and height
2.5
( )
This occurs when a = arctan 5 = 26.6°
h with a cone of base 5 cm and height h − 8
removed.
c The cuboid will slide first. By similarity, h = 16 cm.
5 a Sliding: Shape Volume z-coordinate of
Resolving horizontally: F = 12g sin 20° centre of mass
Resolving vertically: R = 12g cos 20° Large cone 1676 (4 s.f.) 4
At the point of sliding, F = μR
Small cone 209.4 10
12g sin 20° = μ × 12g cos 20°
Frustum 1467 z
μ = tan 20° = 0.364
b Toppling occurs when the centre of mass is 1467z = 1676 × 4 − 209.4 × 10
directly above the pivot point. z = 3.14 cm above the centre of the base.
5
( )
This occurs when a = arctan 12 = 22.6° b
Exam-style questions Shape Volume z-coordinate of
centre of mass
1 a Let the reaction force at C be RC.
Frustum 1467 3.142
Resolving vertically:
60g + 90g + 40g = RC + 95g Hemisphere 261.8 9.875

RC = 95g = 950 N New solid 1729 z

b Moments about C: 1729z = 1467 × 3.142 + 261.8 × 9.875


90g(x − 1) + 40g × 9 = 95g × 6 + 60g × 1 z = 4.161 cm above the base.
90(x − 1) + 360 = 570 + 60 So the centre of mass of the new solid is
90x − 90 + 360 = 630 4.161 − 3.142 = 1.02 cm higher than the centre
90x = 360 of mass of the old solid.
x=4

19
©HarperCollinsPublishers 2018 Cambridge International AS & A Level Mathematics: Further Mechanics 9780008271893

71893_P009_024.indd 19 15/06/18 8:01 PM


2 EQUILIBRIUM OF A RIGID BODY

5 a The rope is light and inextensible. Centre of mass of AE = 0, 3 + 1 ( )


Moments about D: Assume the mass of each rod is 1
W × 5 + 28 × 11 = TC × 7
∑i =1mi xi you get
n
Using Mx =
5
TC = W + 44 3
7 6x = 1 × 1 + 1 × 2 + 1 × +1×1+1× 1 +1×0
2 2
b Resolving vertically:
6x = 6
TC + TD = W + 28 x = 1 (this can also be found from the

5 symmetry of the shape)
7 W + 44 + TD = W + 28
∑i =1mi yi you get
n
2 Using M y =
TD = W − 16
7
( ) ( ) 3
( )
5 2 6y = 1 × 3 +2 +1× 3 +1 +1× +1
W + 44 = 13 W − 16 2
7 7
5
W + 44 =
26
W − 208
× ( )
3 +1 ×
2
3 +1×
( 3 +1 )
7 7 6y = 5 3 + 4
252 = 3W
5 2
W = 84 y= 3+
6 3
6 Toppling will occur when the centre of mass is So centre of mass = (1, 2.11) (to 3 s.f.)
directly above bottom left corner. b The distance from centre of mass to centre of
This occurs when q = arctan
2.5
( )
= 32.0° AB = 3 + 2 – 2.11 = 1.62

( )
4
1.62
7 a Moments about C: So θ = tan −1 = 58.3º
1
1800 × 8 = W(8 − x) So angle between AB and vertical is 58.3º
14 400 = 8W − Wx 9 Let the reaction force between the ladder and the
Wx = 8W – 14 400 wall be RW, the reaction force between the ladder
Moments about B: and the floor be RF and the friction force between
the ladder and the floor be FF.
1575 × 8 = W(x − 2)
Let the angle between the ladder and the floor
12 600 = Wx − 2W
be a, where tan a = 2.
Wx = 2W + 12 600
Moments about B:
8W – 14 400 = 2W + 12 600
50g × 1.75 cos a + 70gx cos a = RW × 3.5 sin a
6W = 27 000
87.5g + 70gx = 3.5RW tan a
W = 4500
87.5g + 70gx = 3.5RW × 2 = 7RW
b Wx = 2W + 12 600 Resolving horizontally:
4500x = 9000 + 12 600 RW = FF
x = 4.8 Resolving vertically:
8 a Let the bottom left of the diagram be the point RF = 120g
(0,0). FF ⩽ µRF
(
Centre of mass of AB = 1, 3 + 2 ) 1
RW ⩽ 3 × 120g
Centre of mass of BC = ( 2, 3 + 1) 87.5g + 70gx
⩽ 40g
7
 
Centre of mass of CD =  3 , 3  87.5g + 70gx ⩽ 280g
2 2 
70gx ⩽ 192.5g
(
Centre of mass of CE = 1, 3 ) x ⩽ 2.75
1 3
Centre of mass of DE =  ,  The person can climb 2.75 m before the ladder
2 2 
slides.

20
©HarperCollinsPublishers 2018 Cambridge International AS & A Level Mathematics: Further Mechanics 9780008271893

71893_P009_024.indd 20 15/06/18 8:01 PM


2
WORKED SOLUTIONS

10 Let A be the point (0, 0) 13 Let A = (0, 0)

Shape Area y-coordinate of Shape Area x-coordinate y-coordinate


centre of mass of centre of of centre of
mass mass
Large circle 4πr2 0
Semi- pr 2 4r
Small circle πr2 r r+ 0
circle 2 3p
Lamina 3πr2 y
r2 2 1
Triangle r r
2 3 3
3πr 2 y = 0 × 4πr 2 + πr 3
Lamina r2
y=
r (1 + π) x y
3 2
r
( )
The distance from A to the centre of mass is
3 r2 p r 2 r + 4r + r 2 × 2 r
a (1 + p)x =
11 a Let the reaction at A be RA and the friction at 2 2 3p 2 3
A be FA r 2
pr 2r3
r3 3
(1 + p)x = + +
Moments about B: 2 2 3 3
2.5g × 3.6 cos a + FA × 7.2 sin a = RA × 7.2 cos a
2.5g + 2FA tan a = 2RA
r2
2
p
( )
(1 + p)x = r 3 + 1
2
(1 + π) x = r(π + 2)
1
FA = RA
2 r(2 + p)
x=
2.5g + RA tan a = 2RA (1 + p)
3 3 r2 r2 1
Given that sin a = , tan a = b 2 (1 + p)y = 2 × 3 r
5 4
3
2.5g + RA × = 2RA r2 r3
(1 + p)y =
4 2 6
5 r
2.5g = 4 RA (1 + p)y =
3
RA = 2g = 20 N
r
y=
b Resolving vertically: 3(1 + p)
RA + T sin b = 2.5g c Let q be the angle between the vertical and AO
T sin b = 2.5g − 2g = 0.5g (1 + p)(2 + p)
x= = 2+p
(1 + p)
0.28T = 0.5g
(1 + p) 1
T = 17.9 N y= =
3(1 + p) 3
c The rod is one-dimensional, straight and rigid,  1 
and the cable is a light inextensible string. q = tan −1  3  = 3.71°
 2 + p
12 Sliding:  

Resolving horizontally: F = 5g sin a 14 a Let the weight of the rod be W N and the
Resolving vertically: R = 5g cos a distance of the centre of mass from A be x m.

At the point of sliding: F = μR On the point of tilting about C, RD = 0.

5g sin a = 0.6 × 5g cos a 20g × 4.8 + W(x − 1.2) = 100g × 1.2

a = arctan 0.6 = 31.0° W(x − 1.2) = 24g

Toppling: will occur when the centre of mass is Wx − 1.2W = 24g


directly above bottom right corner. Wx = 1.2W + 24g
1
This occurs when a = arctan 4 = 14.0() On the point of tilting about D, RC = 0.
250g × 0.8 = 20g × 5.2 + W(5.2 − x)
So the solid will topple when the angle a
reaches 14.0°. 96g = 5.2W − Wx
Wx = 5.2W − 96g

21
©HarperCollinsPublishers 2018 Cambridge International AS & A Level Mathematics: Further Mechanics 9780008271893

71893_P009_024.indd 21 15/06/18 8:01 PM


2 EQUILIBRIUM OF A RIGID BODY

Hence 1.2W + 24g = 5.2W − 96g Resolving vertically:


120g = 4W FW + RG = Mg
W = 30g RG = Mg − FW
The mass of the rod is 30 kg. RG = Mg − µWRW
b Wx = 1.2W + 24g RG = Mg − µWFG
30gx = 1.2 × 30g + 24g RG = Mg − µWµGRG
30x = 60 RG(1 + µWµG) = Mg
x=2 Mg
RG =
The centre of mass is 2 m from A. 1 + µ W µG
b Moments about the point of contact of the
15 a
ladder with the wall:
Shape Length x-coordinate y-coordinate
RG × 8a cos q = FG × 8a sin q + Mg × 2a cos q
of centre of of centre of
mass mass 4RG cos q = 4µGRG sin q + Mg cos q
4RG = 4µGRG tan q + Mg
2r
Semicircle πr 0 p
(below) RG(4 − 4µG tan q) = Mg
Mg
Straight rod 2r 0 0 RG =
4 − 4µG tan θ
Mass at A 2r –r 0 Mg Mg
c Hence 1 + µ µ = 4 − 4µ tan θ
Mass at B πr r 0 W G G

Lamina 2r(π + 2) x y 1 + µWµG = 4 − 4µG tan q


µWµG = 3 − 4µG tan q
2r(p + 2)x = pr2 − 2r2 3 − 4µG tan θ
µW =
2r(p + 2)x = r2(p − 2) µG
2 17 a Assuming origin is at centre of base of
r (p − 2)
x=
2r(p + 2) cross-section
r(p − 2)
x= Shape Area x-coordinate y-coordinate
2(p + 2)
of centre of of centre of
x = r(p − 2) mass mass
2p + 4
2r Rectangle 24 0 3
b 2r(p + 2)y = pr ×
p
Triangle 6 0 7
2r(p + 2) y = 2r2
Lamina 30 x y
2r 2
y=
2r(p + 2)
30 y = 24 × 3 + 6 × 7
r
y=
(p + 2) x =0 y = 3.8
c When r = 5, Height of shape = 6 + 3 = 9
5(p − 2) and 5  o centre of mass is 9 – 3.8 = 5.2 below apex
S
x= y=
2(p + 2) (p + 2) of triangle.
−1  y  2 b Sliding:
So q = tan  x  = (p − 2) = 60.3°
Resolving horizontally: F = 8g sin q
16 a Let the friction force between the ladder and
the wall be FW, the reaction force between the Resolving vertically: R = 8g cos q
ladder and the wall be RW and the friction force At the point of sliding: F = μR
between the ladder and the ground be RG. 8g sin q = 0.25 × 8g cos q
Resolving horizontally:
RW = FG

22
©HarperCollinsPublishers 2018 Cambridge International AS & A Level Mathematics: Further Mechanics 9780008271893

71893_P009_024.indd 22 15/06/18 8:01 PM


2
WORKED SOLUTIONS

q = arctan 0.25 = 14.0°


Toppling will occur when the centre of
2
This occurs when q = arctan 6.37 = 17.4°( )
mass is directly above the pivot point. So the solid will topple first, when the angle

( )
2
This occurs when q = arctan 3.8 = 27.8°
q reaches 17.4°.
20 Let A = (0, 0)
So the solid will slide down the slope first,
when the angle q reaches 14.0°. Shape Area x-coordinate y-coordinate
of centre of of centre of
18 a Let DC = x, angle OCD = 30º mass mass
r 3r
In triangle DOC, tan 30º = r so x = 3 = Rectangle 60 5 3
x 3
3
Semicircle 39.27 5 8.122
Using Pythagoras’ theorem on triangle BCD
2 2
Triangle 15.59 11.73 3
 6r   3r 
BD 2 =  −
 3   3  Lamina 114.86 x y

36r 2 9r 2 a 114.86x = 60 × 5 + 39.27 × 5 + 15.59 × 11.73


= +
3 3
114.86x = 679.22
27r 2
= x = 5.913
3
= 9r2 Horizontal distance between centre of mass
and AE = 5.91 cm
BD = 9r 2 = 3 r
b 114.86 y = 60 × 3 + 39.27 × 8.122 + 15.59 × 3
b Let q be the angle between ED and the vertical
114.86 y = 545.72
y
tan θ = = 3rr = 2 3 y = 4.751
x 3
2 3
Vertical distance between centre of mass
So q = 49.1° and AB = 4.75 cm
19 a Shape Volume z-coordinate of c Let q be the angle between the vertical and AB

Cylinder 502.7
centre of mass
5
q = tan −1 (
4.751
5.913
= 38.8°)
Hemisphere 134.0 11.5 Mathematics in life and work
Solid 636.7 z 1 Let h be the height of the full pyramid.
By similarity, 2(h − 1.15) = h so h = 2.3 m.
636.7z = 502.7 × 5 + 134.0 × 11.5
z = 6.37 2 Shape Volume z-coordinate of
The centre of mass of the solid is 6.37 cm centre of mass
above the flat base. Full pyramid 0.7667 0.575
b Sliding: Small pyramid 0.09583 1.4375
Resolving horizontally: F = 5g sin q Model 0.6709 z
Resolving vertically: R = 5g cos q
0.6709 z = 0.7667 × 0.575 − 0.09583 × 1.4375
At the point of sliding: F = μR
z = 0.452
5g sin q = 0.32 × 5g cos q
The centre of mass of the model is 45.2 cm
q = 17.7°
above the base.
Toppling will occur when the centre of mass
is directly above the pivot point.

23
©HarperCollinsPublishers 2018 Cambridge International AS & A Level Mathematics: Further Mechanics 9780008271893

71893_P009_024.indd 23 15/06/18 8:01 PM


2 EQUILIBRIUM OF A RIGID BODY

3 Sliding: At the point of sliding, F = μR.


R = 125g so F = 0.4 × 125g = 500 N
P = F = 125 N
Toppling: Let A be the bottom right corner,
which is the pivot point.
Moments (A) = 125g × 0.50 − 1.15P
At point of toppling moments about A = 0.
So 125g × 0.50 − 1.15P = 0
P = 543.5 N
So the solid will slide first.

24
©HarperCollinsPublishers 2018 Cambridge International AS & A Level Mathematics: Further Mechanics 9780008271893

71893_P009_024.indd 24 15/06/18 8:01 PM


3
WORKED SOLUTIONS

3 Circular motion
Please note: Full worked solutions are provided as an aid to learning, and represent one approach to answering
the question. In some cases, alternative methods are shown for contrast.
All sample answers have been written by the authors. Cambridge Assessment International Education bears no
responsibility for the example answers to questions taken from its past question papers, which are contained in this
publication.
Non-exact numerical answers should be given correct to 3 significant figures, or 1 decimal place for angles in
degrees, unless a different level of accuracy is specified in the question.

Prerequisite knowledge 2π π
9 ω = 24 × 60 × 60 = 43 200 rad s−1,
1 720° = 4π radians π
v = w r = 43 200 × 6 371 000 = 463 m s−1
2 5 × 2π = 10π radians
π π
3 60° = 3 radians. Arc length = rq = 12 × 3 = 4π cm Exercise 3.2A
4 PE = mgh = 5 × 10 × 6 = 300 J
v2 32
1 1 1 a= = = 36 m s−2
5 KE = mv2 = × 3000 × 82 = 96 000 J r 0.25
2 2
v 2 4.52
6 Total energy = PE + KE 2 a= = = 6.75 m s−2
1 r 3

( )
= 500 × 10 × 10 + × 500 × 152 = 106 250 J 2
2 60 × 1000
v 2
60 × 60
Exercise 3.1A 3 a= = = 6.17 m s−2 (to 3 s.f.)
r 45
directed towards the centre of the circle
1 60 r.p.m. = 60 × 2π = 120π radians per minute
mv 2 0.03 × 4.8 2
= 2π radians per second 4 F = = = 3.46 N directed towards
r 0.2
2 The second hand moves at a speed of 2π radians the centre of the circle
π
per minute. This is the same as radians per 5 a R
30
second.
π rw 2
π
3 v = wr = × 0.1 = 300 m s−1 = 0.0105 m s−1
30
15 × 2π π
4 a ω = = rad s−1 F
60 2
π
b v = w r = × 1 = π m s−1 = 1.57 m s−1
2 2
5 × 2 π
5 ω= = π rad s −1
10
v = w r = π × 0.6 = 0.6π m s−1 = 1.88 m s−1 0.05 g
b Friction force = mrw 2 = 0.05 × 0.25 × 32 = 0.113 N
6 a t = 60π = 12π seconds = 37.7 seconds
5
6 Resolving vertically: R = 2000g = 20 000 N
b ω = v = 5 = 0.167 rad s−1
( )
2
r 30 2000 × 50 × 1000
84π mv 2
60 × 60
7 a Priya: t = = 58.643 seconds Resultant force, F = =
4.5 r 50
88π = 7716 N
Sanjay: t = = 61.436 seconds
4.5 The car is in limiting equilibrium so F = µR:
Difference in time = 2.79 seconds
µR = 7716 so µ = 0.386
v 4.5
b Priya: ω = = = 0.10714 rad s−1
r 42 7 Resolving vertically: R = 600g = 6000 N
Sanjay: ω = v = 4.5 = 0.10227 rad s−1
2 2
r 44 Resultant force, F = mv = 600 × v
r 42
Difference = 0.00487 rad s−1
The motorbike is in limiting equilibrium so F = µR:
8 Distance travelled = v × t = 15 × 35 600 × v 2
= 0.6 × 6000
= 525 m = circumference of the circle 42
C 525 So v = 15.9 m s−1
r= =
2π 2π = 83.6 m

25
©HarperCollinsPublishers 2018 Cambridge International AS & A Level Mathematics: Further Mechanics 9780008271893

71893_P025_032.indd 25 15/06/18 8:08 PM


3 Circular motion

8 Resolving vertically: R = 0.1g = 1N By Newton’s second law (horizontally):


mv 2 0.1 × v 2 2
Resultant force, F = =       R cos 20° = mv
r 0.25 r
The toy is in limiting equilibrium so F = μR: 0.35 × 52
0.1 × v 2 10.233 cos 20° =
= 0.6 × 1 r
0.25       r = 0.910 m
v = 1.22 m s−1
The turntable must be rotating at a speed of 5 Let q be the angle between the normal reaction
1.22 m s−1 for the toy to slide off. force and the horizontal.
Resolving vertically: R sin q = 0.08g
9 Resolving vertically: R = 1500g = 15 000 N

( )
2 Resolving horizontally: R cos q = mrw2
1500 × 45 × 1000
mv 2
60 × 60 where r is the radius of the circle
Resultant force, F = =
r r Radius of circle is 0.3 cos q.
The car is in limiting equilibrium so F = μR:
So R cos q = 0.08 × 0.3 cos q × 102
( )
2
1500 × 45 × 1000 R = 2.4 N
60 × 60
= 0.2 × 15000 0.08g
r This gives sin θ = so q = 19.5°.
r = 78.1 m 2.4
Perpendicular height of the ball below top of
10 Resolving vertically: R = mg = 10m hemisphere, h = 0.3 sin 19.5° = 0.100 m
Resultant force, F = mrw 2 = mxw 2 So distance from base = 0.3 − 0.1 = 0.2 m
For the object not to slide, F  μR
6 Let q be the angle between the normal reaction
mxw 2  10mμ force and the horizontal.
xw 2  10μ 0.4
So cos q ° = 0.6 , q = 48.1897°
10µ
w2  x Resolving vertically: R sin q = 0.3g
R = 4.0249 N
Exercise 3.3A
   Resolving horizontally: R cos q = mrw 2
1 a r = l sin q = 0.5 × sin 30° = 0.25 m
ω 2 = R cosθ = 4.0249cos48.1897° = 22.4
l cos θ 0.5cos 30 mr 0.3 × 0.4
b t = 2π = 2π = 1.31 s         w = 4.73 rad s−1
g 10
c Resolving vertically: T cos q = mg 7 Let q be the angle between the pendulum string
T cos 30° = 2.5 × 10 and the vertical.
T = 28.9 N Resolving vertically: T cos q = mg (where T is
35 the tension)
2 a sin q ° = so q ° = 39.5°
55
Resolving horizontally: T sin q = mrw 2
b Resolving vertically: T cos q = mg
Equating to eliminate T:
T cos 39.5212° = 0.4 × 10
mg mrω 2
T = 5.19 N =
cosθ sin θ
g 10 g rω 2
c ω = = = 4.85 rad s−1 =
l cos θ 0.55cos 39.5212 cosθ sin θ
a = rw 2 = 0.35 × 4.85492 = 8.25 m s−2 g sin θ
ω2 =
d v = rw = 0.35 × 4.8549 = 1.70 m s−1 r cosθ

3 a Resolving vertically: T cos q = mg Since r = sin q and h = cos q :


gr
T × 0.6 = 0.6g ω2 =
rh
0.9
T = 9N g
ω=
g 10 h
b ω = = = 4.08 rad s−1
l cos θ 0.9 × 0.6 8 Resolving vertically: R sin q = mg
0.9
Resolving horizontally (and using Newton’s
4 Resolving vertically: R sin 20° = 0.35g second law):
R = 10.233 N R cos q = mrw2

26
©HarperCollinsPublishers 2018 Cambridge International AS & A Level Mathematics: Further Mechanics 9780008271893

71893_P025_032.indd 26 14/06/18 10:41 PM


3
WORKED SOLUTIONS

Equating to eliminate R: 5 a 
Height at point of release =
 2.5 − 2.5 cos 40°
= 0.58489 m
mg mrω 2
=
sinθ cos θ Total energy at point of release = mgh
r 2
= ω
g = 45 × 10 × 0.58489 = 263.2 N
sin θ cos θ
Maximum speed occurs at the lowest point
g cos θ
ω2 = of the circle.
r sin θ
2 g At lowest point: 1 mv2 = 263.2
ω = 2
r tan θ v = 3.42 m s−1
g
ω= b When the angle made by the rope and the
r tan θ
downward vertical is 25°, h =
 2.5 − 2.5 cos 25°
Exercise 3.4A = 0.234 m
1
At this point mgh + mv2 = 263.2
1 Total energy = mgh + 1 mv2
a  2
2 1
= 2 × 10 × 1.5 + 0.5 × 2 × 32 = 39 J 45 × 10 × 0.23423 + × 45 × v2 = 263.2
2
b At the lowest point the object will have no PE so: v = 2.65 m s−1
1
mv2 = 39 6 a 
When the angle with downward vertical is 110°,
2
1 × 2 × v2 = 39 height = 0.75 + 0.75 sin 20° = 1.01 m
2 b Total energy at the end of the swing = mgh
v = 6.24 m s−1
= 0.5 × 10 × 1.0652 = 5.0326 J
2 At highest point of circle: 1
Energy at start: mgh + 2 mv2 = 5.0326
Total energy = mgh + 1 mv2
2 1
= 0.3 × 10 × 1.4 + 0.5 × 0.3 × 1.52 = 4.5375 J 0.5 × 10 × 0.75 + 2 × 0.5 × u2 = 5.0326
At the lowest point, the object will have no PE so: u = 2.27 m s−1
1 1
mv2 = 4.5375 7 a At the bottom of the circle, total energy = mv2
2 2
1 2 1
2 × 0.3 × v = 4.5375 = × m × (4v)2 = 8mv2
2
v = 5.50 m s−1 1
Total energy at top: mgh + mv2
2
3 a 
Assuming the base of the swing has a height of 0 m: 1
= mg(2l) + 2 mv2
Height of release point = 2.4 − x where x is
Energy is conserved so 2mgl + 1 mv2 = 8mv2
the perpendicular distance of the swing to 2
the point where the rope is attached. 4gl + v2 = 16v2
x=
 2.4 cos 30° so height of swing 4gl = 15v2
15v 2
= 2.4 − 2.4 cos 30° = 0.32154 m l = 4g
b Total energy = mgh = 30 × 10 × 0.32154 = 96.5 J b Maximum force is at the bottom of the circle.
1 2 2
64gmv 2 64gm
c At lowest point: 2 mv 2 = 96.46 Fmaximum = mv = m(4v2) = = N
r 15v 15v 2 15
v = 2.54
4g
The speed at the lowest point is 2.54 m s−1.
Minimum force is at the top of the circle.
2 2 2
4 a 
The maximum speed occurs at the bottom
Fminimum = mv = mv 2 = 4mv2 = 4m N
of the circle. r 15v 15v 15
4
Assume h = 0 at base of circle.
8 At the bottom of the circle, total energy
Energy at start = mgh = 3 × 10 × 2 = 60 J 1 1
( ) 3
2
= mv 2 = m 3gr = gmr J
1 2 2 2
At lowest point: 2 mv2 = 60 Height at any point = r + r sin (90° − q ) = r(1 + cos q )
  v = 6.32 m s−1 Total energy at any point:
b The maximum force occurs at the bottom mgh + 1 mv2 = 3 gmr
of the circle. 2 2
1 3
2 mgr(1 + cos q ) + mv2 = gmr
F = mv = 3 × 40 = 60 N 2 2
r 2
2gr(1 + cos q ) + v2 = 3gr

27
©HarperCollinsPublishers 2018 Cambridge International AS & A Level Mathematics: Further Mechanics 9780008271893

71893_P025_032.indd 27 14/06/18 10:41 PM


3 Circular motion

v2 = 3gr − 2gr(1 + cos q ) b It will complete a full circle if the speed at the
v2 = 3gr − 2gr − 2gr cos q top is greater than 0 and the reaction force does
not decrease to zero before it reaches the top of
v2 = gr − 2gr cos q
the circle.
v2 = gr(1 – 2 cos q )
At top: PE = mgh = 0.1 × 10 × 1 = 1 J
v= gr(1 − 2cos θ )
In order for it to have KE at top, total energy
Exercise 3.5A must be greater than 1 J.
So at bottom: 1 mv2 > 1
1 PE at top = 2 × 10 × 1.5 = 30 J 2
1
× 0.1 × u2 > 1
 or complete motion, total energy must be
F 2
greater than 30 J when at the bottom to allow u > 4.47 m s−1
2
for the object to have some velocity at the top. Reaction force = 0.1 × 4.47 − 0.1 × 10 > 0
1 mv2 > 30 0.5
2 To complete a full circle, u > 4.47 m s−1.
1 × 2 × v2 > 30 5 Total energy at start = mgh + 1 mv2
2 2
v > 5.48 m s−1 = 0.04 × 10 × 0.6 + 0.5 × 0.04 × 2.32 = 0.3458 J
2 a Total energy = mgh + 1 mv2 When v = 0, mgh = 0.3458 so h = 0.865 m.
2
= 0.1 × 10 × 0.6 + 0.5 × 0.1 × 52 = 1.85 J  he particle will come to rest when h = 0.865 m.
T
1 This is above the horizontal so the particle will
At bottom: mv2 = 1.85
2
leave the circle and follow projectile motion.
v = 6.08 m s−1
6 a Total energy at bottom = 1 mv2
b The speed at the top of the circle must be 2
greater than 0 m s−1 and the reaction force must = 0.5 × 0.4 × 62 = 7.2 J
1
not decrease to zero before it reaches the top of At top: mgh + mv2 = 7.2
2
the circle. 1
c At top: total energy 0.4 × 10 × 1 + × 0.4 × v2 = 7.2
2
= 0.1 × 10 × 1.2 + 0.5 × 0.1v2 = 1.85 v = 4 m s−1
2 2
v = 3.61 m s−1 b Total force = mg + mv =  0.4 × 10 + 0.4 × 4
r 0.5
When v = 3.61 ms−1, = 16.8 N
2
reaction force = 0.1 × 3.61 − 0.1 × 10 > 0. 7 Energy at top of bowl = mgh + 1 mv 2
a 
2
0.6
This is greater than 0 so the marble will = 0.15 × 10 × 0.5 + 0.5 × 0.15 × 2.32 = 1.14675 J
complete a full circle inside the pipe. The height at any point is h = 0.75 cos q.
1
3 a Total energy = 1 mv2 = 0.5 × 0.25 × 42 = 2 J So at any point mgh + mv2 = 1.14675
2
2
When it comes to rest, it will have only PE. Total energy = 0.15 × 10 × 0.75 cos q + 0.5 × 0.15 × v2
0.25 × 10 × h = 2 = 1.125 cos q + 0.075v2 ( = 1.14675)
h = 0.8 m b Applying Newton’s second law towards the
b q = cos
0.2
1
−1
( )
= 78.5°
centre of the base:
mg cos q − R =
mv 2
It will make an angle of 78.5° with the r
downward vertical when it comes to rest. v2
When R = 0, 10 cos q =
0.75
4 a 
It will oscillate around its starting point if it 1.14675 − 0.75cos θ
From total energy equation, v =2
comes to rest before it reaches a height of 0.5 m. 0.075
1 1.14675 − 1.125 cos θ
Energy at start = 2 mv2 = 0.05u2 So 7.5 cos q =
0.075
When h = 0.5, PE = mgh = 0.1 × 10 × 0.5 = 0.5 0.5625 cos q = 1.14675 − 1.125 cos q
So 0.05u2 = 0.5, u = 3.16 m s−1 1.6875 cos q = 1.14675
To oscillate around its starting point,
u  3.16 m s−1.
q = cos−1 (
1.14675
1.6875 )
= 47.2°

28
©HarperCollinsPublishers 2018 Cambridge International AS & A Level Mathematics: Further Mechanics 9780008271893

71893_P025_032.indd 28 14/06/18 10:41 PM


3
WORKED SOLUTIONS

8 At the top of the hemisphere, total energy = mgh 0.6


2 a sin θ = so q = 27.5°
= 3 × 10 × 2 = 60 J 1.3
b Resolving vertically: T cos q = mg
Let q be the angle between the line joining the
1.5 × 10
ball and the centre of base of the sphere and T = cos 27.5 = 16.9 N
the upwards vertical.
3 a 
Total energy at top
At any point on the hemisphere total energy = 20 × 10 × 2 + 0.5 × 20 × 22 = 440 J
= mgh + 1 mv2 = 60 Maximum speed is at the bottom.
2
3 × 10 × 2 cos q + 0.5 × 3 × v2 = 60 0.5 × 20 × v2 = 440
60 cos q + 1.5v2 = 60 v = 6.63 m s−1
40 cos q + v2 = 40 mv 2
b T − mg = so T = 1080 N
Applying Newton’s second law towards the r
centre of the base, g 10
4 a ω = = = 1.5302 rad s−1
2 l cos θ 5cos 31.33
mg cos q − R = mv
r
v2
When R = 0, 10 cos q = 2
since θ = sin −1 ( 2.65 ) = 31.33°
v=
ω = 1.5302
v2 = 40 − 40 cos q = 0.589 m s−1
r 2.6
40 − 40cos θ b Resolving vertically: T cos q = mg
So 10 cos q = 2 50 × 10
10 cos q = 20 − 20 cos q T = cos 31.3° = 585 N
30 cos q = 20 5 a Resolving vertically: R = 2500g = 25 000 N
q = 48.2°
mv 2 2500 × v 2
Resultant force, F = =
The ball leaves the hemisphere when q = 48.2°. r 80
At this point v2 = 40 − 40 cos 48.2° The car is in limiting equilibrium so F = μR.
2500 × v 2
v = 3.65 m s−1 = 0.7 × 25 000
80
The ball leaves the hemisphere at a speed
Maximum speed, v = 23.7 m s−1 = 85.2 km h−1
of 3.65 m s−1.
1 1 b Resolving vertically: R = 2500g = 25 000 N
9 a Total energy at bottom of circle = 2 mv2 = 2 mu2
( )
2
1 1 2500 × 45 × 1000
2
60 × 60
At any point, total energy = mgh + mv2 = mu2
2 2 Resultant force, F = mv =
r 80
At any point, height = r − r cos q = r(1 − cos q ) = 4883 N
1 1 The car is in limiting equilibrium so F = μR.
So mgr(1 − cos q ) + mv2 = mu2
2 2
µ = F = 4883 = 0.195
2gr(1 − cos q ) + v2 = u2 R 25 000
v2 = u2 − 2gr(1 − cos q ) 90 × 2π
6 a ω = 60 = 3π rad s−1
b Resolving towards the centre of the circle:
b Resolving vertically: R = 0.2g = 2 N
mv 2
T − mg cos q = Resultant force, F = mrw 2 = 0.2 × d × (3π)2
r
m = 17.765d N
T = mg cos q + (u2 − 2gr(1 − cos q))
r
When the object is in limiting equilibrium,
Exam style questions
F = μR.
g 10 17.765d = 0.4 × 2
1 ω=
a  = = 2.66 rad s−1
l cos θ 1.5cos19.5
  d = 0.0450 m
since q = sin −1 ( )
0.5
1.5
= 19.5°  he maximum distance from centre that
T
the object can be without sliding off the
ω = 2.66
v= = 5.32 m s−1 turntable is 4.50 cm.
r 0.5
l cos θ 1.5cos19.5° r
b t = 2π = 2π = 2.36 s 7 a sin q = = 0.28
g 10 1.5
So r = 1.5 × 0.28 = 0.42 m

29
©HarperCollinsPublishers 2018 Cambridge International AS & A Level Mathematics: Further Mechanics 9780008271893

71893_P025_032.indd 29 14/06/18 10:41 PM


3 Circular motion

28 96
b sin θ = 100 or cos θ = 100 So sin q =
(1.05 − 0.6 )
0.6
g 10 So q = 48.6º
ω= = = 2.64 rad s−1
lcos θ 1.5 × 96 So string becomes slack at an angle of 48.6º
100
above the horizontal.
c Resolving vertically,
12 a Total energy = 0.5 × m × 30x = 15mx
T cos q = 3g
1
At any point, total energy = mgh + 1 mv2 = mu2
T = 31.25 N 2 2
60 × 2π = 2π rad s−1 At any point, height = x − x cos q = x(1 − cos q)
8 ω =
60 1
So mgx(1 − cos q ) + mv2 = 15mx
Resolving vertically: R = 0.3g = 3 N 2
Resultant force, F = mrw 2 = 0.3 × 0.1 × (2π)2 = 1.184 N 2gx(1 − cos q) + v2 = 30x
When the object is in limiting equilibrium, F = μR. v2 = 30x − 20x(1 − cos q)
v2 = 10x + 20x cos q
µ = F = 1.184 = 0.395.
R 3
v2 = 10x(1 + cos q)
The least possible value of μ is 0.395.
Resolving towards the centre of the circle:
9 Resolving vertically: T = 2mg sin q
a  2
(where q is angle between the horizontal T − mg cos θ = mv
r
and the string) m
T = mg cos q + (20x(1 + cos q))
2mg 2mg 100m x
=
()
T= = = 25m N
sin θ 4 4 T = 10m cos q + 20m(1 + cos q))
5 2
T = 30m cos q + 20m
b Resolving horizontally: mv = 2mg cos q
r
T = 10m(3 cos q + 2)
v2 3
= 20 × b When T = 0:
3x 5
v = 36x m s−1 3 cos q + 2 = 0

10 a Height = 4 − 4 cos 50° = 1.4289 m cos q = − 2


3
Total energy = mgh + 1 mv2
2
= 0.05 × 10 × 1.4289 + 0.5 × 0.05 × 32 = 0.939 J
( 32 )
v2 = 10x 1 −

10
At the bottom: 1 mv2 = 0.939 v2 = x
2 3
v = 6.13 m s−1 13 a Let j be the angle between the horizontal and OB.
0.5 5
b When v = 0, mgh = 0.939 so h = 1.88 m. Then sin j = = but q = 90 − j, so
0.7 7
It will reach a height of 1.88 m before 5
sin(90° − q ) = = cos q
changing direction and returning towards its 7
starting point. b 
Applying Newton’s second law towards the
centre of the base:
11 a Total energy = mgh + 1 mv2
2 mv 2
= 0.03 × 10 × 0.6 + 0.5 × 0.03 × 32 = 0. 315 J mg cos q − R =
r
At q below the horizontal When R = 0:
v2
Total energy = 1 × 0.03 × v2 10 cos q =
2 2
+ 0.03 × 10 × (0.6 − 0.6 sin q ) = 0.315 v2 = 20 cos q
0.015v2 + 0.18 − 0.18 sin q = 0.315 v = 3.78 m s−1
0.015v2 = 0.135 + 0.18 sin q c 
At the top of the hemisphere:
v2 = 9 + 12 sin q total energy = 0.4 × 10 × 0.7 + 0.5 × 0.4 × u2
b When the string becomes slack, v = 0 and the = 0.2u2 + 2.8
reaction force < 0, so it only has PE. At any point on hemisphere:
When v = 0, mgh = 0.315 so h = 1.05 m. total energy = mgh + 1 mv2 = 0.2u2 + 2.8
2

30
©HarperCollinsPublishers 2018 Cambridge International AS & A Level Mathematics: Further Mechanics 9780008271893

71893_P025_032.indd 30 14/06/18 10:41 PM


3
WORKED SOLUTIONS

0.4 × 10 × 0.7 cos q + 0.5 × 0.4 × v2 = 0.2u2 + 2.8 T + mg = 2m(u2 − g)


5 T + mg = 2mu2 − 2mg
2.8 × 7 + 0.2v2 = 0.2u2 + 2.8
2 + 0.2v2 = 0.2u2 + 2.8 T = 2mu2 − 3mg
0.2v2 = 0.2u2 + 0.8 For complete circles, T  0 so 2mu2 − 3mg  0
v2 = u2 + 4 3g
Therefore 2u2  3g and u  .
2
u = 3.21 m s−1
c Let min speed (at top of circle) be v.
14 Let the radius of the cone be r and the height This means the max speed (at bottom of
be 4 r . circle) is 2v.
3
The half-angle at the vertex of the cone q The speed of projection is u.
3 At top: loss in KE = gain in GPE
satisfies tan θ = .
4
0.5m(u2 – v2) = 0.5mg
Resolving vertically: R sin q = mg
u2 – v2 = g ➀
R = 10m = 10m = 50m
sin θ 3
5 () 3 At bottom: gain in KE = loss in GPE
0.5m ((2v)2 – u2) = 0.5mg
Let x be the radius of the circle followed by the object.
0.5m (4v2 – u2) = 0.5mg
Resolving horizontally and using Newton’s
4v2 – u2 = g ➁
second law:
➀ + ➁ gives 3v2 = 2g
R cos q = mxw2

( ) ( )
2g
v2 =
mx 80 80xm 3
3r 3r 100xm 2g
= =
()
R= So from ➀ u = 2
+g
cosθ 4 3r 3
Equating: 5 5g
50m 100xm u2 =
= 3
3 3r
5g
150mr = 300xm   u =
3
x = r (i.e. the radius of the circle traced out 16 a Tension in the string = 8g = 80 N
2
by the object is half the radius of the cone) Tcos q = 4g
Using trigonometry, d = r tan θ = 3r so the cos q = 40 = 0.5
2 4 80
perpendicular distance between the circle b Resolving vertically
and the vertex of the cone is 3r where r is the mv 2
4 T sin θ =
radius of the cone. r
2
8g sin q = 4v = 2v2
15 a Initially, total energy = m × 10 × 0.5 = 5m 2
The maximum speed occurs at the bottom 1 3
If cos θ = , then sin θ =
of the circle where h = 0 2 2
8g 3
At this points, KE = 1 mv2 = 5m = 2v 2 so v 2 = 2 3 g
2 2
So v2 = 10 17 At lowest point, total energy = 0.5mv2
v = 3.16 m s−1. At highest point, total energy = 0.5mv + 1.5mg
So max angular speed ω = v = 3.16 = 6.32 rads−1. Energy is conserved so
r 0.5
b At top: loss in KE = gain in GPE 0.5mv2 = 0.5mv + 1.5mg
0.5m(u2 – v2) = 0.5mg 0.5v2 = 0.5v + 15
u2 – v2 = g v2 = v + 30
so v2 = u2 – g ➀ v2 – v – 30 = 0
2
Also T + mg = mv but r = 0.5 so (v – 6)(v + 5) = 0
r
v = 6 or v = –5
T + mg = 2mv2 ➁
So v = 6 m s−1.
Substituting ➀ in ➁ gives
So maximum speed is 6 m s−1.
31
©HarperCollinsPublishers 2018 Cambridge International AS & A Level Mathematics: Further Mechanics 9780008271893

71893_P025_032.indd 31 14/06/18 10:41 PM


3 Circular motion

18 Let x = radius of circle and r = radius of bowl.  esolving these horizontally, by Newton’s
R
Let q be the angle between OP and the circular second law:
face. T sin q = 2ma
a sin θ = 0.5r = 0.5. Therefore q = 30° T sin q = 2mrw2
r 2
b Resolving vertically, 10sin q = mg T = 2mrω
sin θ
10 × 0.5
Therefore m = = 0.5 kg Equating the above two equations:
10
2 2mg 2mrω 2
c Resolving horizontally, 10cos q = mv cosθ
=
sin θ
x

cos θ = 3 and x = (r 2 − (0.5r )2) Since sin q = r .


2 3
g rω 2
3r =
x= cos θ r
2 3
10 3 0.5v 2 g
Therefore = cos θ =
2 3r 3ω 2
2
This gives v2 = 15r so v = 15r b ω = 10
3cos θ
 l – l cos q
19 a The height of P at any point = Min value = 1.83 rad s−1
= l (1 – cos q ) As q increases towards 90°, w increases.
Total energy = 0.5 × 2 × 102 = 100 J
So range of w values is w  1.83 rad s–1
So at any point,
Energy = 0.5 × 2 × v2 + 2 × 10 × l (1 – cos q )
Mathematics in life and work
= v2 + 20l (1 – cos q )
So v2 + 20l (1 – cos q ) = 100 1 Resolving vertically: R = mg = 10m N

So v2 = 100 – 20l (1 – cos q ) mv 2 m × v 2


Resultant force, F = =
r 25
Resolving towards O The go-kart is in limiting equilibrium so
2
T – 20 cos q = 2v F = μR.
l
2 m × v 2 = 0.8 × 10m
T = 20 cos q + 2v 25
l
200 − 40l + 40lcos θ v2
T = 20 cos q + =8
l 25
T = 20 cos q + 200 − 40 + 40 cos q v = 14.1
l
The maximum speed at which the go-karts can
T = 60 cos q + 200 − 40 travel without sliding is 14.1 m s−1
l
or 50.8 km h−1.
b The max l for complete circular motion would 2
require v = 0 at top of circle and reaction force 2 When wet m × v = 0.5 × 10m
25
to be greater than 0. v2
=5
So mgh = 100 25

2 × 10 × 2l = 100 v = 11.2

40 l = 100 The maximum speed at which the go-karts can


l = 2.5 m travel without sliding is 11.2 m s−1
or 40.2 km h−1.
20 a Let T be the tension in the string
So the speed would decrease by 2.9 m s−1 or
Resolving vertically:
10.6 km h−1.
T cos q = 2mg
2mg
T=
cosθ

32
©HarperCollinsPublishers 2018 Cambridge International AS & A Level Mathematics: Further Mechanics 9780008271893

71893_P025_032.indd 32 14/06/18 10:41 PM


4
WORKED SOLUTIONS

4 Hooke’s law
Please note: Full worked solutions are provided as an aid to learning, and represent one approach to answering
the question. In some cases, alternative methods are shown for contrast.
All sample answers have been written by the authors. Cambridge Assessment International Education bears no
responsibility for the example answers to questions taken from its past question papers, which are contained in this
publication.
Non-exact numerical answers should be given correct to 3 significant figures, or 1 decimal place for angles in
degrees, unless a different level of accuracy is specified in the question.

Prerequisite knowledge 3mgµ = T cos q


3mgµ = 8mg
1 RB 3
RA 8
T T µ=
9
B A 3 Carriage has lost both kinetic energy and potential
m 4m energy since it has both slowed and dropped
Loss in potential energy = mgh = 250 × 10 × 4 = 10 000 J
40°
30° Loss in kinetic energy = 1 mu2 – 1 mv2
2 2
30° 40° (where u is speed at start and v final speed)
1 1
mg 4mg = 2 × 250 × 22 – × 250 × 12 = 375 J
2
a RA = 4mg cos 40° Work done against resistances = 400 × 100
RB = mg cos 30° = 40 000 N
F = ma for Work done by carriage is 40 000 – 10 000 – 375 = 29 625 J
4mg sin 40° – T – 0.1mg cos 40° = 4ma 1 (as the energy lost by the motion provides some of
T – mg sin 30° – 0.15mg cos 30° = ma 2 the work required to overcome the resistances).
Adding equations 1 and 2 together Exercise 4.1A
4mg sin40° – 0.1mg cos 40 – mg sin 30°
40 × 0.2
– 0.15mg cos 30° = 5ma 1 T=
3
40sin 40 – cos 40° – 10sin 30° – 1.5cos 30° = 5a T = 2.67 N
a = 3.73 ms–2 λx
2 l=
T
b Using equation 2: 1.4 − l
l = 60 × 37
T – mg sin 30° – 0.15mg cos 30° = 3.73m
37l = 84 − 60l
T = 3.73m + 10m sin 30° + 1.5m cos 30°
T = 10.03m N 97l = 84

c Using parallelogram of forces and the cosine rule: l = 86.6 cm

F2 = (10.03m)2 + (10.03m)2 3 33 = l × 0.3


– 2 × 10.03m × 10.03m cos 70 0.9
l = 33 × 3 = 99 N
F = 11.5m N
2 a Resolve vertically at Z: 4 T = 32 × 0.75l
l
2T sin q = 4mg T = 24 N
T sin q = 2mg
10mg 5 The percentage of the compression of the spring
T= x
3 = × 100
b Resolve vertically at X: l
1.4g = 120 × (l − x)
Rx = mg + T sin q l
Rx = 3mg 14l = 120l − 120x
Resolve horizontally at X: 120x = 106l

33
©HarperCollinsPublishers 2018 Cambridge International AS & A Level Mathematics: Further Mechanics 9780008271893

71893_P033_043.indd 33 15/06/18 8:39 PM


4 HOOKE’S LAW

x 106 2x = 10(0.7 − x)
=
l 120 12x = 7
Therefore percentage of the compression of the
x = 0.583 m
spring is 88.3%.
y = 0.117 m
6 T = 2g
11 T = 130 × 0.1
T = 20 N 0.5
1.5 − l T = 26 N
l = 30 × 20
20l = 45 − 30l Resolving parallel to the plane:

50l = 45 26 = 4g sin q
26
l = 90 cm sin q =
40
350 × x q = 40.5°
7 5.5 × 10 =
0.2
x = 3.14 cm 12 Let x be the extension in the string attached to
8 The tension acts throughout the strings and is point A.
equal to 0.5g = 5 N. Let y be the extension in the string attached to
For S1: 5l1 = 17 × x1 point B.
5 × 1 = 17x1 2+x+3+y=7
x1 = 0.294 m x+y=2
For S2: 5l2 = 20x2 16x
Tx = = 8x
2
5 × 0.6 = 20x2 14y
Ty =
x2 = 0.15 m 3
Total length is 1 + 0.294 + 0.6 + 0.15 = 2.04 m 14y
8x =
3
9 Resolving vertically: 2g + T2 = T1 24x = 14y
Let x1 be the extension in SP and x2 be the 24x = 14(2 − x)
extension in TP. 38x = 28
1.5 + x1 + x2 = 2.7 x = 0.737 m
x1 + x2 = 1.2 y = 1.26 m
45x1
T1 = = 60x1 13 Let x be the extension in string AP.
0.75
45x 2 Let y be the extension in string BP.
T2 = = 60x2
0.75
l 2l
2g + 60x2 = 60x1
P
20 + 60x2 = 60(1.2 − x2) 0.5 + x 0.75 + y
1 kg
120x2 = 52 Tx Ty
A B
x2 = 0.433 m
x1 = 0.767 m 1g
10 Let x be the extension in string attached to X.
Let y be the extension in string attached to Y. 1.5 m

x + y = 0.7 y = 0.15 m (given in the question)


λx x = 1.5 − 0.5 − 0.75 − 0.15 = 0.1 m
TX =
0.5 First calculate the tensions in the strings so you
3λ y know in which direction friction will be acting.
TY = 0.3
As the particle is in equilibrium TX = TY so: Tx = λ × 0.1 = 0.2l
0.5
λ x = 3λ y
0.5 0.3 Ty = 2λ × 0.15 = 0.4l
0.75
2x = 10y Given that Ty > Tx friction will act to oppose force Ty.

34
©HarperCollinsPublishers 2018 Cambridge International AS & A Level Mathematics: Further Mechanics 9780008271893

71893_P033_043.indd 34 15/06/18 8:39 PM


4
WORKED SOLUTIONS

Tx + µR = Ty 16
R l = 100 N
0.2l + 0.3 × 1 × 9.8 = 0.4l
0.3 m
0.2l = 2.94
l = 14.7 N 40 N
0.1 × 60
14 T = = 20 N
0.3
Resolve vertically: 20 cos q = 2g 4gcosq
cos q = 0.98
4gsinq
q = 11.478… = 11.5° 4g

q
q
l = 60 N If sin q = 0.8 then cos q = 0.6.
100 × 0.2
Thrust in compressed spring = = 40
0.5
T
Resolving perpendicular to the slope:
R = 40 sin q + 4g cos q = 40 × 0.8 + 39.2 × 0.6
R = 55.52 N
Resolving parallel to the slope:
2 kg F 4g sin q + 40 = 40 cos q + µ × R
40 × 0.8 + 40 = 40 × 0.6 + µ × 55.52
2g 72 = 24 + 55.52µ

Resolve horizontally: 20 sin 11.48° = F µ = 0.865

F = 3.98 N 17 Let T be the force in the stretched string.

15 F = ma
T − µR = 0.5 × 0.25

1 + x S1 l = 25 T − 0.5 × 4.9 = 0.125


Tx
T = 2.575

2.6 m P 0.5 kg 2.575 = 40 × x


0.2
x = 0.013
Ty
1 + y S2 l = 25 Length of string = 0.2 + 0.013 = 0.213 m
Exercise 4.2A
Let x be the extension in S1.
8 × 0.52
1 W= = 2J
Let y be extension in S2. 2 × 0.5
x + y = 0.6 7 × 0.52
2 EPE = = 1.25 J
2 × 0.7
Tx = 25x
λx
3 T= = 15x = 2g
Ty = 25y l
Resolving vertically: 25x = 25y + 0.5g 20 = 15x
25x = 25(0.6 − x) + 5 x = 1.33
2
50x = 20 EPE = 15 × 1.33 = 13.3 J
2×1
x = 0.4 m
y = 0.2 m

35
©HarperCollinsPublishers 2018 Cambridge International AS & A Level Mathematics: Further Mechanics 9780008271893

71893_P033_043.indd 35 15/06/18 8:39 PM


4 HOOKE’S LAW

50 × 1.52 F = ma
4 EPE = = 28.125 J
2×2 T sin q = mrw 2
Using conservation of energy: where
KE gain = EPE loss
1 T = 0.019581 × 2000 = 52.22
mv2 = 28.125 0.75
2
r = 0.7696 sin 40° = 0.4947
v2 = 56.25
52.22 sin 40° = 4 × 0.4947 × w 2
v = 7.5 m s−1
2000 × x 2 w 2 = 16.96
5 30 =
2×1 w = 4.12 rad s−1
x2 = 0.03
10 loss of EPE = gain in KE
x = 0.173
5 × 1.32 = 1 × 2 × v2
The stretched length of the string is 1.17 m. 2 × 1.7 2
48 × (3l)2 v = 1.58 m s−1
6 60 =
2×l
11 As at lowest point v = 0
60 = 216l
loss of EPE = PE gain
l = 0.278 m
2
100 × 1.92 100 × 1.4 2 1.3g(0.5 + x) = 20 × x
7 Gain in EPE =
2×2

2×2
= 41.25 J 2 × 1.5
20x2 − 39x − 19.5 = 0
50 × 4 2
8 EPE = = 400 J x = 2.36 or −0.413
2×1
By conservation of energy: So x = 2.36 and the lowest point is 3.86 m below X.
Loss of EPE = gain in KE + work done against friction Let the extension at equilibrium point be y.
1 20 × y
400 = 2 × 2.5 × v2 + 0.3 × 2.5 × 10 × 4 1.3g =
1.5
v2 = 296
y = 0.975 m
v = 17.2 m s−1
KE gained + EPE gained = PE lost
2
9 Resolving vertically: 1 × 1.3v2 + 20 × 0.975 = 1.3 × 10 × 1.48
2 2 × 1.5
T cos 40° = 4g
x × 2000 0.65v2 + 6.3375 = 19.24
cos 40° = 40
0.75 v = 4.46 m s−1
x = 0.0196 1000 × 40 2
12 WD = = 2670 J
The stretched length of the string is 0.770 m. 2 × 300

A 13 Gain in EPE = loss in GPE


λ × 1.52
l = 2000 N 2 × 0.5 = 0.7 × 10 × 2
40° l = 6.22 N
0.75 m 14 Model the doll as a particle and assume that
motion is vertical (not side to side).
At start total energy is EPE and PE:
80 × 0.22
EPE = = 5.33 J
x 2 × 0.3
PE = 0.6 × 10 × 0.1 = 0.6 J
At maximum distance the spring will have
extended by x so will still contain EPE, v = 0 so
KE = 0 and it will have gained height so will also
have gained PE.
80 × x 2
EPE = = 133.3x2
2 × 0.3
4g
PE = 0.6 × 10 × (x + 0.2) = 6x + 1.2

36
©HarperCollinsPublishers 2018 Cambridge International AS & A Level Mathematics: Further Mechanics 9780008271893

71893_P033_043.indd 36 15/06/18 8:39 PM


4
WORKED SOLUTIONS

By conservation of energy: v2 = 60 − 2 × 5 × 2
133.3x2 + 6x + 1.2 = 5.93 v = 6.32 m s−1
133.3x2 + 6x − 4.73 = 0 The speed at which the car crashes into the
x = 0.167 or −0.212 (this is the start position) wall is 6.32 m s−1.
So the doll rises a maximum distance of 17 Assume that man falls exactly 70 m and starts 70 m
0.3 + 0.167 = 0.467 m. above ground.
15 At start: GPE = 86 × 10 × 70 = 60 200
A At bottom: gain in EPE = loss of GPE
5000 × x 2 = 60 200
P l 2×l
120 400l = 5000x2
m q
X x Given that the stretched length of rope must
not exceed 70 m: x + l = 70
m
So the equation becomes:
24.08l = x2
mg
24.08l = (70 − l )2
q l 2 − 164.08l + 4900 = 0
l = 125 or 39.3
Relative to A:
Hence maximum value for the rope’s natural
At point P: EPE = 0
length is 39.3 m.
GPE = −mgl sin q = − 5 mgl
13 18 Length of elastic = 2 × (0.0752 + 0.302)0.5 = 0.61847 m
λ × x2
At point X: EPE = Loss of EPE = gain in KE
2×l
5 50 × 0.418472 = 1 × 0.1 × v2
GPE = −mg(l + x) sin q = − mg(l + x) 2 × 0.2 2
13
At both points KE = 0 as at rest. v2 = 437.8
By conservation of energy: v = 20.9 m s−1
5 5 5 λ × x2 19 The model plane starts at equilibrium point.
− mgl = − mgl − mgx +
13 13 13 2×l
15x
2 Resolve vertically: 0.4g =
mgx = λ × x
5 0.4
13 2×l x = 0.107 m
10lmg When plane has been pulled down:
x=
13λ
15 × 0.6 2
8 × 22 EPE =
2 × 0.4
= 6.75 J
16 At start: EPE = = 8J
2×2
After the car has travelled 2 m and the string Find the speed when string becomes slack after
becomes slack: release:
EPE lost = gain in KE + work done against resistance Loss of EPE = gain in KE + gain in GPE
1
8 = 1 × 0.2v2 + 2 × 1 6.75 = 2 ×0.4 × v2 + 0.4 × 10 × 0.6
2
v2 = 60 6.75 = 0.2v2 + 2.4
v = 7.75 m s−1 v = 4.66 m s−1
So the velocity at the point that the string stops Using this as u for motion under gravity,
pulling car is 7.75 m s−1. calculate how far the plane would travel if the
Calculate the deceleration: ceiling was not present.
F = ma v2 = u2 + 2as
1 = 0.2a 0 = 21.75 + 2 (−10)s
a = 5 m s−2 (deceleration) s = 1.09 m
v2 = u2 + 2as So the plane would hit the ceiling.

37
©HarperCollinsPublishers 2018 Cambridge International AS & A Level Mathematics: Further Mechanics 9780008271893

71893_P033_043.indd 37 15/06/18 8:39 PM


4 HOOKE’S LAW

20 At top: GPE = 5 × 10 × 3 = 150 J


1200 × 0.13852 = 1200 × 0.12 + 1 × 2 × v 2
After falling 2 m as the string is still slack: 2 × 0.4 2 × 0.4 2
Loss in GPE = gain in KE v2 = 13.8
150 – 5 × 10 × 1 = 1 × 5 × v2 v = 3.71 m s–1
2
v = 6.32 m s−1 2
A 5m 5m B
Loss in GPE = gain in EPE q
2
150 − 5 × 10 × (1 − x) = 9400 × x
2
100 + 50x = 4700x2 4m
T T
4700x2 − 50x − 100 = 0
x = 0.15 or −0.14
So the length of the string when the particle P
first comes to instantaneous rest is 1.15 m.
mg
21 a KE = 1 × 300 × 102 = 15 000 J
2
b By conservation of energy: EPE transferred to a Length AP = 52 + 4 2 = 6.403 m
elastic rope = 15 000 J
T = 60 × 1.403 = 28.1 N
2000 × x 2 3
= 15 000
2×5 Resolve vertically: force up = force down as
x = 8.7 m in equilibrium
Distance from O will be 13.7 m. 2T sin q = mg
c The calculation above assumes no resistance 4
mg = 2 × 28.1 × = 35.1 N
forces. 41
m = 3.51 kg
2
b EPE = 60 × 2.806 = 39.4 J
Exam-style questions 2×6

λx 3 a Loss in GPE = gain in KE + gain in EPE


1 a T=
l 1 2mg × x 2
λ mg(l + x) = × m × v2 +
300 = × 10 2 2×l
40 2 2g × x 2
2g(l + x) = v +
l = 1200 N l
2 2gx 2
v = 2g(l + x) −
l
b A 25 cm B dv 4gx
q b 2v = 2g −
dx l
dv 4gx
= 0,2g − =0
10 cm dx l
T T
1
x = 2l
( )
2
2g 1 l
( )
(view from above)
1 2
v 2 = 2g l + l −
P 2 l
5gl
v2 =
2
T = 1200 × 6.926 = 416 5gl
20 v=
2
Resolving perpendicular to the line AB
c When v = 0:
2T sin θ = 2a 2gx 2
2g(l + x) − =0
2 × 416 × 10 = 2a l
26.926
x2
a = 154 m s–2 l+x− =0
l
c By conservation of energy x2 – xl − l2 = 0
EPE at start = EPE at O + KE at O
x=
(1 + 5 )l
2
38
©HarperCollinsPublishers 2018 Cambridge International AS & A Level Mathematics: Further Mechanics 9780008271893

71893_P033_043.indd 38 15/06/18 8:39 PM


4
WORKED SOLUTIONS

4 EPE lost = gain in GPE + gain in KE


O 56 mg × 10.369l 2
( ) 1
2× 3 13 = 4mg × 7l + 2 mv2
2 × 2l
q l = 0.3 m 56 g × 107.52l = 56gl + v2
l = 60 N 3 13
v2 = 501gl
6 a GPE lost = EPE gained
45 N
P
l X
6g

Resolve vertically: T cos q = 60


Resolve horizontally: T sin q = 45
tan q = 0.75, so sin q = 0.6 2mg

T × 0.6 = 45
30°
T = 75 N
Compare the equation for EPE and tension in 5mgx 2
2mg × sin 30° =
an extended string: 2l
2l = 5x2
EPE = xT
2 x= 0.4l
8 = 37.5x
x = 0.213 m b Greatest speed is when a = 0.
So the extended length of the string is This is when force up the slope = force down
0.3 + 0.213 = 0.513 m. slope.
2 T = mg sin 30°
 13l  7l 5mgx
5 a AX = (3l )2 +  =
 2  2 l
= mg sin 30°

3l 3l l
A B x=
10
q
Gain in KE + gain in EPE = loss in PE
13
2
5mg ( 10l )
2
T T 1 l
mv 2+ = mg sin 30°
2 2l 10
l = 2l lg lg
X v2 = −
l = l mg 10 20
4mg lg
v=
10
2T sin q = 4mg
7 a When at rest v = 0, hence kinetic energy = 0.
13
2 = 2mg EPE gained = GPE lost
T 7
19.6x 2 = (2 + x) × 0.9g
2 4
14mg 19.6x2 − 36x − 72 = 0
T=
13
x = 3.04 (or −1.21)
3l
14mg λmg 2 OA = 5.04 m
=
13 2l
b Resolve vertically:
λ = 56 F = ma
3 13
19.6 × 3.04 − 0.9 × 10 = 0.9a
b AX = (3l)2 + (12l)2 = 12.369l 2
a = 23.1 m s−2
Extension in each string is 10.369l.
39
©HarperCollinsPublishers 2018 Cambridge International AS & A Level Mathematics: Further Mechanics 9780008271893

71893_P033_043.indd 39 15/06/18 8:39 PM


4 HOOKE’S LAW

8 a KE at start + EPE at start = EPE at end − loss in GPE Using Hooke’s law:

3l T=
(
λ 2 13 − 5
=
)
13
mg
2 5 3
5 13 65mg
l= mg =
2m (
3 2 13 − 5 )78 − 15 13
l 10 a EPE released = KE gained

λ ( 2l )
2
2m 60° m2gl
=
2l 2
l = 8mg
b EPE released = gain in PE + gain in KE

8mg ( 34l )
2
60° 3l 1
= mg + mv 2
2l 4 2
9lg 3lg 2v 2
= +
1 × 2m × U 2 + 3mg ( 2)
l 2
= 3mg ( )
3l 2
2
4 4 4
2 2l 2l
2
v = 3lg
− 2mgl sin 60° v = 3lg
27gl 8 3 gl 3gl
U2 = − − 11 a Resolve vertically:
8 8 8
0.45λ
(
U2 = gl 3 − 3 ) 0.9g =
1.3

U = gl 3 − 3 ( ) l = 26 N
b Original EPE = EPE at instantaneous rest − loss
b Gain in KE + gain in PE = loss of EPE in GPE
1 × 2m × v2 + 2mg × 1.5l sin 60 = 3mg ( 32l )2 26 × 0.4 2 26 × x 2 − 0.9g(x + 0.4)
=
2 2l 2 × 1.3 2 × 1.3
2
4.16 = 26x − 23.4x − 9.36
27gl 12 3 gl
v2 = −
8 8 26x2 − 23.4x − 13.52 = 0
27gl 12 3 gl x = 1.3 (or −0.4)
v= −
8 8
AB = 2.6 m
9 12 a Resolving vertically:
4l
200x
2l 2l = 20g
X Y 0.5
q x = 0.5 m
Initial extension = 1 m
T 3l T 200 × 12
EPE = = 200 J at a distance of
2 × 0.5
0.5 m below equilibrium point.

P b EPE at start = gain in GPE + gain in KE +


EPE when particle first comes to rest
200 = 20 × 10 × (1.5 − x) + 1 × 20 × v2
2
XP = (2l)2 + (3l)2 = 13l 2
+ 200 × (1.5 − x )
2T sin q = 2mg 2 × 0.5
3 200 = 300 − 200x + 10v2 + 200x2−600x + 450
T× = mg
13 v2 = 20x2 − 80x + 55
13 v = 0 as particle is at rest.
T= mg
3
x = 0.882 m

40
©HarperCollinsPublishers 2018 Cambridge International AS & A Level Mathematics: Further Mechanics 9780008271893

71893_P033_043.indd 40 15/06/18 8:39 PM


4
WORKED SOLUTIONS

c 
Assumptions: no air resistance, and that the string 3.24 − 7.2y + 4y 2 17.64 − 25.2y + 9y 2 29
remained taut and therefore had some EPE. + =
3.6 3.6 45
13 a In equilibrium 13y2 – 32.4y + 18.56 = 0
λ × 0.5 = 1.5 × g y = 1.6 or 0.89
2
y = 1.6 is start position so AC = 2 – 0.89 = 1.11 m
l = 60 N
b From conservation of energy (taking point 15 a Resolving vertically:
A as h = 0) 50x
= 2.4g
PE when raised = EPE at 2 m + KE at 2 m 0.3
+ GPE at 2 m x = 0.144 m
At 2 m, extension of spring = 0, therefore 50 × 0.1442
EPE = 2 × 0.3 = 1.73 J
EPE = 0
60 × (0)2 1 b Mass comes to rest when v = 0.
1.5 × g × (0.8 + x) = + × 1.5 × v 2 + 1.5 × g × x
2×2 2 EPE before = EPE when next at rest + gain in PE
60 × (0)2 1
0.8 + x) = + × 1.5 × v 2 + 1.5 × g × x
2×2 2 2
12 + 15x = 0.75v2 + 15x 1.728 = 50 × x + 1.2 × 10 × (0.144 + x)
2 × 0.3
0.75v2 = 12 50x2 + 7.2x = 0
v2 = 16 x = 0, i.e. the spring has returned to its
From equations of motion natural length.

v2 = u2 + 2as 16 a EPE at X = work done against friction + EPE at Y


At A, v = 0, a = –10 36 × 3.12 = 6F + 36 × 1.12
2 × 0.9 2 × 0.9
0 = 16 + (2 × –10 × x)
6F = 168
20x = 16
F = 28 N
 x = 0.8 m 1.1 × 36
b At Y tension = = 44 N which is greater
So OA = 0.8 + 2 = 2.8 m 0.9
than 28 N so the particle starts to move again.
14 a Let x be the extension in spring one and, let y be
When particle is next at rest the work done against
the extension in spring 2.
friction has used up all of the EPE from Y.
(0.6 + x) + (0.9 + y) = 2
EPE at Y = work done against friction
x + y = 0.5 36 × 1.12 = 28s
y = 0.5 – x 2 × 0.9
In equilibrium so: s = 0.86 m

2y Total distance = 6 + 0.86 = 6.86 m


3x
=
0.6 0.9 17 a l = 0.8, l = 30 N m = 0.3 kg
3x 1 − 2x Let x be extension at equilibrium
=
0.6 0.9 At equilibrium
x = 0.15 30x
0.3g =
0.8
AP = 0.75 cm
0.3 × 10 × 0.8
x= = 0.08 m
b EPE at start = EPE in AP + EPE in PB 30
b Particle is extended by a further distance y
3 × 0.22 2 × 0.7 2 29
+ = 30 × (0.08 + y)2
2 × 0.6 2 × 0.9 45 EPE at extension =
2 × 0.8
When particle is next at instantaneous rest v = 0
When particle is released and has returned
and let distance of particle from B = y
to original length, by conservation of energy
From conservation of energy
EPE loss = gain in KE + gain in GPE
2 × (0.9 − y)2 3 × (1.4 − y)2 29
+ =
2 × 0.9 2 × 0.6 45

41
©HarperCollinsPublishers 2018 Cambridge International AS & A Level Mathematics: Further Mechanics 9780008271893

71893_P033_043.indd 41 15/06/18 8:39 PM


4 HOOKE’S LAW

30 × (0.08 + y)2 1  aximum velocity occurs when particle passes


M
= × 0.3 × v2 + 0.3 × 10 (0.08 + y) though equilibrium point.
2 × 0.8 2
18.75(0.0064 + 0.16y + y2) = 0.15v2 + 0.24 + 3y 3mgx
mg =
0.15v2 = 18.75y2 – 0.12 l
which is when
v2 = 125y2 – 0.8
l = 3x
Then motion under gravity u2 = 125y2 – 0.8, 2
v2= 20(l + x) – 30x
v = 0.1, s = 0.8, a = –10 l
v2 = u2 + 2as 2 30x 2
v = 20(3x + x) –
3x
0.01 = 125y2 – 0.8 + 2 × (–10) × 0.8 = 80x – 10x
125y2 = 16.81 = 70x
y = 0.367 cm v = 70x

  So AB = 0.8 + 0.08 + 0.367 = 1.25 m 70l
=
3
18 l =1, modulus of elasticity = l
2 b At A v = 0 so KE = 0
EPE = λ × 2 = 2l
2×1 so by conservation of energy
When string is no longer stretched by 3mgx 2
conservation of energy = mg(l + x)
2×l
EPE lost = gain in KE + gain in GPE 3x 2 = (l + x) (Note this equation can also be
2l = 1 × 0.5 × v2 + 0.5 × 10 × 2 2l
2 found by setting v2 = 0 in equation from part a)
2l = 0.25v2 + 10
3x2 = 2l2 + 2lx
v2 = 8l – 40
3x2 – 2lx – 2l2 = 0
Motion under gravity
Assuming that it reaches v = 0 when s > 1, x=
2l ± ((−2l ) 2
− 4 × 3 × ( −2l )
2
)
u2 = 8l – 40, a = –10 6

v2 = u2 + 2as
x=
2l ± (52l )2

0 < 8l – 40 + 2 × (–10) × 1 6
8l > 60 x = l ± 2l 13
3
l > 7.5
So distance AB = l + l + 2l 13
λmg × 0.3L 3
19 a T =
L
= 0.3lmg
=
(
l 4 + 13 )
b Resolving vertically: 3
mg = T cos 60°
Mathematics in life and work
mg = 0.3lmg × 0.5
20 1 When speed is at a maximum, a = 0 and jumper is
l=
3
in equilibrium.
c T = 2mg
Let x be the distance from platform.
F = ma
Resolving vertically:
2mg sin 60° = mrw 2
3645(x − 36)
3 g = (1.3L sin 60°) w 2 = 88g
36
2g x = 44.69 m
ω=
1.3L Extension of cord at maximum speed is 8.69 m.
20 a Loss in GPE = Gain in KE + gain in EPE Gain in EPE + KE = loss in GPE
1 30mx 2 3645 (x − 36)2 88v 2
10m(l + x) = 2 mv2 + + = 88gx
2×l 2 × 36 2
2
20(l + x) = v2 + 30x 405(x − 36)2
l v 2 = 2gx −
352
v2 = 20(l + x) – 30x v 2 = 806.9
l
v = 28.4 m s−1
42
©HarperCollinsPublishers 2018 Cambridge International AS & A Level Mathematics: Further Mechanics 9780008271893

71893_P033_043.indd 42 15/06/18 8:40 PM


4
WORKED SOLUTIONS

2 From equation above:


3645 (x − 36)2 88v 2
+ = 88gx
2 × 36 2
At maximum extension, v = 0
50.625(x − 36)2 = 880x
50.625(x2 − 72x + 1296) = 880x
50.625x2 − 3645x + 65 610 = 880x
50.625x2 − 4525x + 65 610 = 0
x = 71.2 m (or 18.2 m)
x = 71 m to nearest metre
So assuming the man is less than 3 m tall, the
minimum value of y is 74 m.
3 No air resistance, always assumed extension is
greater than 36 m, assume man is less than 3 m tall.
Modelled man as particle until final stage.

43
©HarperCollinsPublishers 2018 Cambridge International AS & A Level Mathematics: Further Mechanics 9780008271893

71893_P033_043.indd 43 15/06/18 8:40 PM


5 Linear motion under a variable force

5 Linear motion under a variable force


Please note: Full worked solutions are provided as an aid to learning, and represent one approach to answering
the question. In some cases, alternative methods are shown for contrast.
All sample answers have been written by the authors. Cambridge Assessment International Education bears no
responsibility for the example answers to questions taken from its past question papers, which are contained in this
publication.
Non-exact numerical answers should be given correct to 3 significant figures, or 1 decimal place for angles in
degrees, unless a different level of accuracy is specified in the question.
Prerequisite knowledge So −y1 = ln x − 1
4
1 x 1 1 = 1 − ln x
=1−
x +1 x +1 y 4
dy dy
2 = xy 8 x = ey
dx dx
1
∫ y d y = ∫ x dx 1
∫e ∫ x dx
−y
dy =
ln y = 1 x2 + c
2 −e−y = ln x + c
2 ln y = x2 + d
y = 0 when x = 4
dy sin x
3 = −e−0 = ln 4 + c
dx y
c = − ln 4
∫ y d y = ∫ sin x dx −e−y = ln x − ln 4
1 y2 = −cos x + c
−y x
2 −e = ln
y2 = −2 cos x + d 4
4
e −y = ln
dy 5 x
4 =
dx e x
Exercise 5.1A
∫ 1 d y = ∫ 5e
−x
dx
1 a v = ∫ t 2 − 2t dt
y = −5e−x + c
5
dy
= 4x2 − 25 v = 1 t3 − t2 + c
dx 3
When t = 0, v = 14
∫ 1 d y = ∫ ( 4x )
2
− 25 dx
So c = 14
4
y = x3 − 25x + c v = 1 t3 − t2 + 14
3 3
dy
6 x
dx
= 4x2 − 25 b s = ∫ 1 t 3 − t 2 + 14 dt
3
4x 2 − 25 dx 1 4 1 3
s = t − t + 14t + c
∫ 1 d y = ⌠⌡ x 12 3

( )
When t = 0, s = 0
∫ 1 d y = ⌠⌡ 4x − x dx
25
So c = 0
y= 2x2 − 25 ln x + c 1 1
s = t 4 − t 3 + 14t
dy 12 3
7 x = y2 d v
dx 2 a a=
1 dt
∫ y d y = ⌠⌡ x dx
−2
v = 5(5t + 2)−1
−1 = ln x + c dv = −5 × 5(5t + 2)−2
y
dt
When y = 4, x = 1
−25
−1 = ln 1 + c a=
4 (5t + 2)2
c=−1
4

44
©HarperCollinsPublishers 2018 Cambridge International AS & A Level Mathematics: Further Mechanics 9780008271893

71893_P044_054.indd 44 14/06/18 11:06 PM


5
WORKED SOLUTIONS

v2 = 100 − 10x2
b s = ⌠ 5 dt
⌡ 5t + 2 Changes direction when v = 0.
s = ln(5t + 2) + c 0 = 100 − 10x2
When t = 0, s = 0 x = 10 = 3.16 m
So c = −ln 2
7 a = 10e0.5x
s = ln(5t + 2) − ln 2

( )
5t + 2 dv
s = ln v = 10e 0.5x
2 dx

∫ v dv = ∫ 10e
0.5x
dx
3 a a = 5e−2t
1 v 2 = 20e0.5x + c
v = ∫ 5e −2t dt 2
When x = 0, v = 2.
v = −2.5e−2t + c
2 = 20 + c
When t = 0, v = 50
So c = −18
So c = 50 + 2.5 = 52.5
1 v 2 = 20e0.5x − 18
So v = 52.5 − 2.5e−2t 2
v = 40e 0.5x − 36
b When t = 0, v = 50
When x = 5, v = 21.2 m s−1.
As t → ∞, v → 52.5 – 0 = 52.5
8 a a = x3(3 − x)
So 50 ⩽ v < 52.5
dv
v = x 3(3 − x)
4 a x = 5 sin 2πt dx
v = dx = 10π cos 2πt
∫ v dv = ∫ 3x
3
dt − x 4 dx
d 2x 1 2 3 4 1 5
a = 2 = −20π2 sin 2πt v = x − x +c
dt 2 4 5
When t = 5, a = 0 m s−2 When x = 0, v = 0 so c = 0
3 2
b Maximum magnitude of acceleration = 20π2 m s−2 v2 = x4 − x5
2 5
5 a a = 3x − 2 3 4 2 5
v= x − x
v dv = 3x − 2 2 5
dx
b When v = 0
∫ v dv = ∫ (3x − 2) dx 3 2
0 = x4 − x5
2 5
1 2 3 2
( )
v = x − 2x + c 3 2
2 2 x4 − x = 0
When x = 0, v = 0, c = 0 2 5

v2 = 3x2 − 4x x = 0 or 3 − 2 x = 0
2 5
v = 3x 2 − 4x So the object is at rest when x = 0 m and
b When x = 3 when x = 3.75 m.
9 a 10a = 2x − 3x2
v = 3(3)2 − 4(3)
a = 0.2x − 0.3x2
v = 15 = 3.87 m s−1
v dv = 0.2x − 0.3x2
6 a = 10x dx
∫ v dv = ∫ (0.2x − 0.3x ) dx
2
dv
v = −10x
dx
1 2
v = 0.1x2 − 0.1x3 + c
∫ v dv = ∫ −10x dx 2
1 2 When x = 0, v = 0 so c = 0
v = −5x2 + c
2 v 2 = 0.2x2 − 0.2x3
When x = 0, v = 10, c = 50
1 2 v = 0.2x 2 − 0.2x 3
v = 50 − 5x2
2

45
©HarperCollinsPublishers 2018 Cambridge International AS & A Level Mathematics: Further Mechanics 9780008271893

71893_P044_054.indd 45 14/06/18 11:06 PM


5 Linear motion under a variable force

b When v = 0
t = 1 ln 
100 
0.2x2 − 0.2x3 = 0 16  500 − 4v 2 
x2 − x3 = 0 When t = 5
x2(1 − x) = 0 5 = 1 ln  100 2 
16  500 − 4v 
So x = 0 or x = 1
100
The object will travel 1m before instantaneously = e 80
coming to rest. 500 − 4v 2
500 − 4v2 = 100e−80
Exercise 5.2A v = 11.2 m s−1

1 3a = −21v 4 a 70 km h−1 = 70 × 1000 = 19.4 m s−1


60 × 60
a = −7v b 800a = 1600 − 4v
dv v
= −7v
dt dv 1600 − 4v 2
800 =
1 dt v
∫ v dv = ∫ −7 dt
⌠ v
800 dv = ∫ 1 dt
ln v = −7t + c
When t = 0, v = 30 so c = ln 30
(
⌡ 1600 − 4v
2
)
t = 1 ln 30
7 v ( ) ⌠
−100 −8v
(
⌡ 1600 − 4v
2
dv = ∫ 1 dt
)
When v = 15
( )
−100 ln (1600 − 4v2) = t + c
t = 1 ln 30 = 0.0990 s
7 15 When t = 0, v = 15
c = −100 ln (1600 − 4 × 152)
2 a = −15v
c = −100 ln (700)
v dv = −15v
dx  700 
So t = 100 ln 
 1600 − 4v 2 
∫ 1 dv = ∫ −15 dx
When v = 19.444
v = −15x + c  
700
When x = 0, v = 20 so c = 20 t = 100 ln  
 1600 − 4 × 19.444 
2

v = −15x + 20 t = 208 s
When v = 5 5 1000a = 4900 − 0.4v 2
v
5 = −15x + 20
x = 1m 1000 v dv = 4900 − 0.4v 2
dx v
500
3 0.5a = − 4v
v dv 4900 − 0.4v 3
1000 v =
dv 500 − 4v 2 dx v
0.5 =
dt v v2 dv
1000 =1
1⌠ v 4900 − 0.04v 3 dx
 dv = ∫ 1 dt
2 ⌡ 500 − 4v 2
25000 ⌠ −0.12v 2
−  dv = ∫ 1 dx
⌠ −8v
− 1  dv = ∫ 1 dt 3 ⌡ 4900 − 0.04v 3
16 ⌡ 500 − 4v 2
− 25 000 ln(4900 − 0.04v3) = x + c
− 1 ln (500 − 4v2) = t + c 3
16
When x = 0, v = 5
When t = 0, v = 10
c = − 25 000 ln(4900 − 5)
c = − 1 ln (500 − 4 × 102) 3
16 25 000
c=− ln(4895)
c = − 1 ln (100) 3
16

46
©HarperCollinsPublishers 2018 Cambridge International AS & A Level Mathematics: Further Mechanics 9780008271893

71893_P044_054.indd 46 14/06/18 11:07 PM


5
WORKED SOLUTIONS

So x = 25 000 ln  4895 
 4900 − 0.04v 3 
∫ 450(v + 3) dv = ∫ 45 dt + c
3
450v 2 + 1350v = 45t + c
When v = 12 2

x = 25000 ln  4895  When t = 0, v = 0 so c = 0


3  4900 − 0.04 × 123  1
t = 45 (225v2 + 1350v)
= 110 m (3sf )
When v = 4, t = 200 s
6 a F = μR
= μmg Exercise 5.3A
= 0.6 × 0.02 × 10
1 0.3a = 0.3 × 10 − (2 + 0.5v2)
= 0.12
0.3a = 1 − 0.5v2
b 0.02a = −(5v2 + 0.12) For terminal velocity: 1 − 0.5v2 = 0
a = −(250v2 + 6) v = 1.41 m s−1
v dv = −(250v2 + 6) 2 0.5a = −0.5 × 10 − 0.6v
dx
0.5a = −(5 + 0.6v)

 v dv = ∫ −1 dx
⌡ 250v 2 + 6 0.5v dv = −(5 + 0.6v)
dx
1 ⌠ 500v dv = ∫ −1 dx 0.5v
500 ⌡ 250v 2 + 6 ∫ 5 + 0.6v dv = ∫ −1 dx
1 ln(250v2 + 6) = −x + c
500
When x = 0, v = 10

( 25
)
 5 − 6(5 + 0.6v) dv = ∫ −1 dx
⌡ 6

c = 1 ln (250 × 102 + 6)
500 6
⌡ (
5⌠ 1− 5
5 + 0.6v )
dv = ∫ −1 dx

6 ⌡(
 1 − 25 5 + 0.6v ) dv = ∫ −1 dx
c = 1 ln (25 006) 5⌠ 0.6
500
3
So x = 1 ln 
25 006 
500  250v 2 + 6  5 v − 125 ln(5 + 0.6v) = −x + c
6 18
When v = 5 When x = 0, v = 20
 
x = 1 ln  25 006  So c = 50 − 125 ln (17)
500  250 × 52 + 6  3 18
So x = 50 − 125 ln(17) − 5 v + 125 ln(5 + 0.6v)
x = 0.002 77 m 3 18 6 18
7 a 450a = 45 Maximum height is when v = 0.
v+3
So maximum height = 8.17 m.
dv 45
450v =
dx v + 3 3 0.8a = −0.8 × 10 − 0.5v
∫ 450v (v + 3) dv = ∫ 45 dx + c 0.8 dv = −(8 + 0.5v)
dt
∫ ( 450v + 1350v ) dv = ∫ 45 dx + c
2
⌠ 0.8 dv = −1 dt
⌡ 8 + 0.5v ∫
450v 3 1350v 2
+ = 45x + c 8 0.5
5 ∫ 8 + 0.5v
3 2 dv = ∫ −1 dt
150v3 + 675v2 = 45x + c 8
When x = 0, v = 0 so c = 0 5 ln(8 + 0.5v) = −t + c
When t = 0, v = 15
x = 1 (150v3 + 675v2)
45 8
So c = ln (15.5)
When v = 4, x = 453 m 5

b 450a = 45
v+3
So t = 8 ln
5 (
15.5
8 + 0.5v )
dv 45 When v = 5
450 =
dt v + 3
t = 0.623 seconds

47
©HarperCollinsPublishers 2018 Cambridge International AS & A Level Mathematics: Further Mechanics 9780008271893

71893_P044_054.indd 47 14/06/18 11:07 PM


5 Linear motion under a variable force

4 0.3a = −(0.3 × 10 + 0.5v2) 7 a 75a = 75 × 10 − 25v


0.3v dv = −(3 + 0.5v2) 75a = 750 − 25v
dx
dv
3 = 30 − v
⌠ 0.3v dt
 dv = ∫ −1dx
⌡ 3 + 0.5v 2 3
5
∫ 30 − v dv = ∫ 1 dt
ln (3 + 0.5v2) = −x + c
3 −3 ln(30 − v) = t + c
When x = 0, v = 10
When t = 0, v = 0
So c = 5 ln(53)
3 So c = −3 ln 30

So x =
5 
ln
53 
3  3 + 0.5v 2 
(
So t = 3 ln 30
30 − v )
When t = 4
Maximum height is when v = 0.
So maximum height = 4.79 m.
( )
30
4 = 3 ln 30 − v

= ln (
30 − v )
4 30
5 a 7a = 7 × 10 − 20v
3
7 dv = 70 − 20v 4
dt 30
= e3
7 30 − v
∫ 70 − 20v dv = 1 dt −4
30 − v = 30e 3
− 27 ln(70 − 20v) = t + c v = 22.1 m s−1
0
When t = 0, v = 0
7 ln(70) b Terminal velocity is when a = 0.
So c = − 20 60
7 75a = 75 × 10 − 20v − 40v2
=e7
70 − 20v
So t = 7 ln
20
7
70 − 20v( ) 0 = 750 − 20v − 40v2

When t = 3 v = −4.59 or v = 4.09

ln (
7
70 − 20v
=
60
7 ) So her terminal velocity is 4.09 m s−1.

7 60
Exam-style questions
=e7
70 − 20v
70 − 20v = 7e− 1 a a= 16
( x + 2 )2
v = 3.50 m s−1
 v dv = 16(x + 2)−2
b For terminal velocity, a = 0 dx
−2
So 70 − 20v = 0
∫ v dv = ∫ 16 ( x + 2) dx
So v = 3.5 m s−1 1 v2 = −16(x + 2)−1 + c
2
6 a = −(10 + 2v2)
1 2 −16
dv = −(10 + 2v2) v = +c
dt
2 ( x + 2)

 1 dv = ∫ −1 dt When x = 2, v = 0.5 m s−1
⌡ 10 + 2v 2 1 = −4 + c
8
1 tan −1  v  = −t + c So c = 33
  8
2 5 5
1 2 −16 33
So 2 v = +
When t = 0, v = 25 ( x + 2) 8
1  
So c = tan −1  25  v= 33 − 32
2 5  5 4 x+2

So t = 1 tan −1  25  − 1 tan −1  v  33
2 5  5 2 5  5
b As x → ∞, v → = 2.87 m s−1
4
2 a 0.4a = − 0.4 × 10 − 0.8v
Maximum height is when v = 0.
0.4v dv = −(4 + 0.8v)
So time to reach maximum height is 0.331 seconds. dx

48
©HarperCollinsPublishers 2018 Cambridge International AS & A Level Mathematics: Further Mechanics 9780008271893

71893_P044_054.indd 48 14/06/18 11:07 PM


5
WORKED SOLUTIONS

0.4v − 3 x
∫ 4 + 0.8v dv = ∫ −1 dx 12 000 − v 3 = 11 936 e 400

v
∫ 10 + 2v dv = ∫ −1 dx v 3 = 12 000 − 11 936 e
− 3 x
400

1 5
∫ 2 − 10 + 2v dv = ∫ −1 dx 3 − 3 x
v = 12 000 − 11 936 e 400
1 5
v − ln (10 + 2v ) = −x + c b When v = 15
2 2
 
When x = 0, v = 25 x = 400 ln  11 936 3 
3  12 000 − 15 
So c = 25 − 5 ln ( 60 )
2 2 x = 43.3 m
So x =
25
2

1
2
5
v + ln
2 (
10 + 2v
60 ) 4 a x = ∫ v dt

Maximum height is when v = 0. x = ∫ e −5t + 5t 2 dt


So maximum height = 8.02 m. 1 5
x = − e −5t + t 3 + c
5 3
b 0.4 dv = −(4 + 0.8v) When t = 0, x = 3
dt
1 5 16
0.4 x = − e −5t + Sot c3 =
+
∫ 4 + 0.8v dv = ∫ −1dt 5 3 5
1 5 16
So x = − e −5t + t 3 +
1 5 3 5
ln(4 + 0.8v) = −t + c
2 When t = 5
When t = 0, v = 25 1 625 16
x = e −25 + +
So c = 1 ln(24) 5 3 5
2
  
= 211.533… = 212 (3 s.f.)
So t = 1 ln
2 (
24
4 + 0.8v ) b a = dv
dt
Maximum height is when v = 0. v = e−5t + 5t2
So time to reach maximum height = 0.896 dv = −5e−5t + 10t
seconds. dt

3 a 2000a = 60 000 − 5v 2 When t = 3


v
a = 30.0 m s−2
dv 60 000
2000v = − 5v 2 1
dx v 5 a a = e −5t
3
dv 12 000 1
400v = − v2 v = ∫ 3 e −5t dt
dx v
dv 12 000 − v 3 1
400v = v = − e −5t+ c
dx v 15

v2 When t = 0, v = 5
400⌠ dv = ∫ 1 dx
⌡ 12 000 − v 3 5 = − 1 e −5× 0 + c
15
400 ln(12 000 − v3) = x + c

3 So c = 5 + 1 = 76
15 15
When x = 0, v = 4 76 1 −5t
So v = − e
15 15
400
So c = − 3 ln(11 936)
b When t = 3
400  11 936  v = 5.07 m s−1
So x = ln
3  12 000 − v 3 
c 5  v  76 − 1 e −5t
15 15
3  11 936 
x = ln  6 a a = −(1 × 10 + 3v2)
400  12 000 − v 3 
v dv = −(10 + 3v2)
3 x dx
11 936
e 400 = ⌠ v
12 000 − v 3  dv = ∫ −1dx
⌡ 10 + 3v 2

49
©HarperCollinsPublishers 2018 Cambridge International AS & A Level Mathematics: Further Mechanics 9780008271893

71893_P044_054.indd 49 14/06/18 11:07 PM


5 Linear motion under a variable force

1 2 ⌠ 30 dv = ∫ 1 dt
6 ln(10 + 3v ) = −x + c ⌡ 120 − 7v
When x = 0, v = 25 −30 ln(120 − 7v) = t + c
7
So c = 1 ln(1885) When t = 0, v = 0
6
1  1885  So c = − 30 ln (120 )
So x = ln 7
6  10 + 3v 2 
Maximum height is when v = 0.
So t = 30
7
ln ( 120
)
120 − 7v

ln (
120 − 7v ) 30
So x = 0.873 m 120 7
= t
So maximum height of object =
 1.2 + 0.873
7t
= 2.07 m. 120
= e 30
120 − 7v
b dv = −(10 + 3v2) −7t
30
dt 120 − 7v = 120e

⌠ 1 120  − 7 t
 dv = ∫ −1 dt v=
7 
1 − e 30 
⌡ 10 + 3v 2 
b As t → ∞, v → 120 = 17.1 m s−1.
⌠ 1 7
 dv = ∫ −1 dt
( ) +( ) 50a = 50 × 10 − 0.08v2
2 2 9
⌡ 10 3v
50v dv = 500 − 0.08v2
1  3  dx
tan −1  v = −t + c
30  10  ⌠ 50v
 dv = ∫ 1 dx
⌡ 500 − 0.08v 2
When t = 0, v = 25
  50 ln(500 − 0.08v2) = x + c
So c = 1 tan −1  1875  −
0.16
30  10 
625 ln(500 − 0.08v2) = x + c

 1875   3  2
1 1
So t = tan −1  − tan −1  v
30  10  30  10  When x = 0, v = 0

Maximum height is when v = 0. So c = − 625 ln ( 500 )


2
So time to reach maximum height = 0.273 625  500 
So x = ln
seconds. 2  500 − 0.08v 2 
−1 x When x = 50
7 a a = 1e 2
3 625  500 
dv 1 − 12 x 50 = ln
v = e 2  500 − 0.08v 2 
dx 3
= ln  
1 −1 x 4 500
∫ v dv = ⌠⌡ 3 e 2 dx 25  500 − 0.08v 2 
4
1 2 2 −1 x 500
= e 25
v =− e 2 +c
2 3 500 − 0.08v 2
When x = 0, v = 2 − 4
  500 − 0.08v2 = 500e 25
1 2 8
So c = 2 × 4 + 3 = 3 v = 30.4 m s−1

1 2 8 2 − 12 x 10 a 0.2a = ex − (1 + 2x)
v = − e
2 3 3
0.2a = ex − 1 − 2x
16 4 − 12 x
v= − e  a = 5ex − 10x − 5
3 3
v dv = 5ex − 10x − 5
b As x → ∞, v → 2.31 m s−1. dx
8 a 1500a = 9000 − 3000 − 350v
∫ v dv = ∫ (5e )
x
b − 10x − 5 dx
30a = 120 − 7v
1 v2 = 5e x − 5x2 − 5x + c
30 dv = 120 − 7v 2
dt When x = 0, v = 4

50
©HarperCollinsPublishers 2018 Cambridge International AS & A Level Mathematics: Further Mechanics 9780008271893

71893_P044_054.indd 50 14/06/18 11:07 PM


5
WORKED SOLUTIONS

8=5+c
c a = 72
So c = 3 3v
1 dv 7
So 2 v 2 = 5e x − 5x2 − 5x + 3 v =
dx 3v 2
When x = 10 dv 7
v3 =
1 2 dx 3
10
2 v = 5e − 500 − 50 + 3 7
∫ v dv = ∫ 3 dx
3

v = 468 m s−1
v4 7
11 a a = 0.2e0.2x = x+c
4 3
dv = 0.2e0.2x 4 28
v v = x+d
dx 3
When x = 0, v = 1
∫ v dv = ∫ 0.2e
0.2x
dx
14 = 0 + d so d = 1
1 2 0.2x v4 = 28 x + 1
2v = e +c 3
When x = 0, v = 0 13 a F = 60a = 10 cosec v
cosv
So c = −1
a = cosec v
So 1 v 2 = e0.2x – 1 6cos v
2
dv cosec v
v = 2e 0.2x − 2 =
dt 6cos v
b When x = 10 6cos v
∫ cosec v dv = ∫ 1 dt
v = 3.57 m s−1
c a=0 ∫ 6cosv sin v dv = ∫ 1 dt
12 a F = 6a = 1202     ∫ 3sin 2v dv = ∫ 1 dt
kv
3
  − cos 2v = t + c
a = 202 2
When t = 0, v = π
kv
6
dv 20
= 3 π
dt kv 2 So c = − cos
2 3
20
∫ v dv = ⌠⌡ k dt
2
c= − 3
4
v 3 20t
3
= +c So t = 3 − 3 cos 2v
k 4 2
60t 4t = 3 − 6 cos 2v
v3 = +d
k
6 cos 2v = 3 − 4t
When t = 0, v = 1 and when t = 1, v =2
cos 2v = 3 − 4t
1 = 0 + d so d = 1 6
60t
v3 = +1 b When t = 1
k
1
60 cos 2v = −
8= +1 6
k
2v = 1.738 or 2v = 4.454
60
7=
k Since 0 < v < π
2
2v = 1.738
k = 607
v = 0.869
b v = t +1
3 60
So v = 0.869 m s–1 when t = 1
k
60t 14 a F = ma = 0.6a = 0.5v2 +1 – 0.25v2 – 0.5
v3 = +1
60
7 0.6a = 0.25v 2 + 0.5
v3 = 7t + 1 dv
0.6v = 0.25v2 + 0.5
dx

51
©HarperCollinsPublishers 2018 Cambridge International AS & A Level Mathematics: Further Mechanics 9780008271893

71893_P044_054.indd 51 14/06/18 11:07 PM


5 Linear motion under a variable force

⌠ v 5 8
 dv = ∫ dx − ln(80 − 3v) = t + c
⌡ 0.25v 2 + 0.5 3 3
When t = 0, v = 0
5
2 ln (0.25v 2 + 0.5) = x+c
3 So c = − 8 ln(80)
3

( )
When x = 0, v = 2 .
8 80
2 ln (1) = c = 0 So t = 3 ln 80 − 3v
5
2 ln (0.25v 2 + 0.5) = x When t = 5
3

ln (0.25v 2 + 0.5) =
5
6
x
ln( 80
80 − 3v
= )
15
8
15
80
5x =e8
0.25v 2 + 0.5 = e 6 80 − 3v
− 15
80 − 3v = 80e 8
 5x 
v2 = 4  e 6 − 0.5
  80  − 15 
v= 1−e 8 
3  
5x
v = 4(e 6 − 0.5) v = 22.6 m s−1
b When x = 1.5
b For terminal velocity, a = 0
5x
v= 4(e 6 − 0.5) 80a = 800 − 30v − 45v2
v = 3.46 m s–1. 0 = 800 − 30v − 45v2

15 a 4a = v3 e5x − 4 0 = 9v2 + 6v − 160

4v dv = v3 e5x − 4 v = −4.56 or 3.90


dx
So terminal velocity = 3.90 m s−1
⌠ 4
 2 dv = ∫ e 5x − 4 dx  2
⌡v 17 a ma = −  mg + mv2 
 4λ 
4 1 5x − 4
− = e +c
v 5
 2 
a = − 10 + v 2 
When x =0.8, v = 2  4λ 
−2 = 1 e0 + c
5  2 2
a = − 40λ +2 v 
c = − 11  4λ 
5
4 1 5x − 4 11  40λ 2 + v 2 
− = 5e − v
dv
= −
v 5
dx  4λ 2 
20
− = e 5x − 4 − 11
v ⌠ 4λ 2v dv = −1 dx
20

⌡ 40λ 2 + v 2 ∫
v=−
e 5x − 4 − 11 2l2 ln(40l2 + v 2) = −x + c
20 When x = 0, v = 40l
v=
11 − e 5x − 4
So c = 2l2 ln(40l2 + (40l2 ))
b When x = 0.5  40λ 2 + ( 40λ )2 
20 x = 2λ 2 ln  2 2 
v=  40λ + v 
11 − e 5(0.5)− 4
v = 1.86 m s−1 Maximum height is when v = 0.
 40λ 2 + ( 40λ )2 
16 a 80a = 80 × 10 − 30v x = 2λ 2 ln  
 40λ 2 
80a = 800 − 30v
8a = 80 − 3v x = 2l2 ln(1 + 40)
8 dv = 80 − 3v x = 2l2 ln(41)
dt
8
∫ 80 − 3v dv = ∫ 1 dt
52
©HarperCollinsPublishers 2018 Cambridge International AS & A Level Mathematics: Further Mechanics 9780008271893

71893_P044_054.indd 52 14/06/18 11:07 PM


5
WORKED SOLUTIONS

 2   k  k
b a = − 10 + v 2  t=
1
tan −1  u −
1
tan −1  v
 4λ  gk  g  gk  g 

 2 2
a = − 40λ +2 v  Maximum height is when v = 0.
 4λ  So time to maximum height
dv  40λ 2 + v 2  1  k
= − tan −1  u
dt  4λ 2  =
gk  g 
⌠ 4λ 2 x
 dv = ∫ −1 dt 19 a 0.5 a =
⌡ 40λ 2 + v 2 ( )
sec x 2
2
4λ  v  0.5 a = xcos(x2)
tan −1  = −t + c
λ 40  λ 40 
0.5 v dv = xcos(x2)
dx
4λ  v 
tan −1  = −t + c
 λ 40  ∫ 0.5v dv = ∫ x cos ( x ) dx
2
40

v = ∫ x cos ( x 2 ) dx
When t = 0, v = 40l 1 2
So c = 4λ tan −1 40 ( ) 4
40
( )
v 2 = ∫ 4x cos x 2 dx
t = 4λ tan −1
40
( 40 − )

40
 v 
tan −1 
 λ 40  b Let u = x2
du
When v = 0 = 2x
dx
t = 4λ tan −1
40
( 40 ) 1
So v = ∫ 4x cos(u) 2x du
2

18 a ma = −(mg + mkv 2) v 2 = ∫ 2cos (u) du


a = −(g + kv 2)
v2 = 2 sin (u) + c
v dv = −(g + kv 2)
dx v2 = 2 sin (x2) + c
⌠ v When x = 0, v = 4
 dv = ∫ −1 dx
⌡ g + kv 2 So c = 16
1 ln(g + kv 2) = −x + c
v = 2sin(x 2) + 16
2k
When x = 0, v = u c Max of 2 sin (x2) + 16 = 18
So c = 1 ln(g + ku2) Min of 2 sin (x2) + 16 = 14
2k
 g + ku 2  So max of v is 18 = 3 2
1
x= ln
2k  g + kv 2  Min of v is 14
Maximum height is when v = 0. 20 a 10a = x2 e−v
1  g + ku 2 
Maximum height = ln 
2k  g  10v dv = x2 e−v
dx

∫ 10ve dv = ∫ x 2 dx
v
b ma = −(mg + mkv2)
a = −(g + kv2)
x3
dv = −(g + kv2) = ∫ 10ve v dv
3
dt
x 3 = 30∫ ve v dv
⌠ 1
dv = ∫ −1 dt

⌡ g + kv 2
∫ ve dv = ve v − ∫ e v dv
v
b
1  k
tan −1  v = −t + c = vev − ev + c
gk  g 
= ev(v − 1) + c
When t = 0, v = u
 k So x3 = 30ev(v − 1) + c
1
So c = tan −1  u
gk  g 
53
©HarperCollinsPublishers 2018 Cambridge International AS & A Level Mathematics: Further Mechanics 9780008271893

71893_P044_054.indd 53 14/06/18 11:07 PM


5 Linear motion under a variable force

When x = 0, v = 0 so c = 30 3 Terminal velocity, a = 0


So x3 = 30ev(v − 1) + 30 dv
180 = 1800 − (20v + 28v2)
dt
c When v = 2 28v 2 + 20v − 1800 = 0
x3 = 30e2(2 − 1) + 30 v = −8.38 or 7.67
x3 = 251.7 So her maximum possible speed is 7.67 m s−1.
x = 6.31 m

Mathematics in life and work

1 180a = 180 × 10 − 25v


180 dv = 1800 − 25v
dt
36 dv = 360 − 5v
dt
⌠ 36
 dv = ∫ 1 dt
⌡ 360 − 5v
36
− ln(360 − 5v) = t + c
5
When t = 0, v = 0
So c = − 36 ln(360)
5

So t = 36 ln
5
360
(
360 − 5v )
When t = 45
ln 
360  225
=
 360 − 5v  36
225
360
= e 36
360 − 5v
− 225
360 − 5v = 360e 36
v = 71.861… = 71.9 m s−1
dv
2 36v = 360 − 5v
dx
⌠ 36v
 dv = ∫ 1 dx
⌡ 360 − 5v
36 v
5 ∫ 72 − v
dv = ∫ 1 dx
36 v
5 ∫ 72 − v
dv = ∫ 1 dx
36 72
5 ∫
−1 + dv = ∫ 1 dx
72 − v
36
5 (−v −72 ln(72 − v)) = x + c
When x = 0, v = 0
2592
So c = − ln(72)
5
x=
2592
5
ln
72
(
72 − v

36
5
v)
When v = 71.861, x = 2723 m
When she opens her parachute she will be
3200 − 2723 = 477 m above the ground.

54
©HarperCollinsPublishers 2018 Cambridge International AS & A Level Mathematics: Further Mechanics 9780008271893

71893_P044_054.indd 54 14/06/18 11:07 PM


6
WORKED SOLUTIONS

6 Momentum
Please note: Full worked solutions are provided as an aid to learning, and represent one approach to answering
the question. In some cases, alternative methods are shown for contrast.
All sample answers have been written by the authors. Cambridge Assessment International Education bears no
responsibility for the example answers to questions taken from its past question papers, which are contained in this
publication.
Non-exact numerical answers should be given correct to 3 significant figures, or 1 decimal place for angles in
degrees, unless a different level of accuracy is specified in the question.

Prerequisite knowledge u2 = v2 − 2as


= 02 − 2 × −10 × 0.75
1 Momentum = mv = 2.6 × 3 = 7.8 kg m s−1
= 15
2 Momentum = mv = 0.56 × 2 = 1.12 kg m s−1
u = 3.87 m s−1
3 2 × 5 + 3 × 2 = 2 × v1 + 3 × 4
v1 = 2 m s−1 in the same direction as before Speed immediately after impact is 3.87 m s−1
3.87
4 4 × 2 + 4.5 × −3 = 4 × −1 + 4.5 × v2 So e = = 0.726
5.33
v2 = − 1 m s−1 i.e. 0.333 m s−1 in opposite direction 6 u2 = v2 − 2as
3
5 a) Add both equations together = 02 − 2 × −10 × 2.1
2x = 4 = 42
x=2 u = 6.48 m s−1
Substituting x = 2 into the first equation gives: Speed immediately after impact is 6.48 m s−1
2 + y = 5 so y = 3 Speed immediately before impact is
b) Add both equations together v 6.48
u= = = 8.53 m s−1
e 0.76
5x = 2.5
v 2 − u 2 0 2 − 8.532
x = 0.5 s= = = 3.64 m
2a 2 × −10
Substituting x = 0.5 into the second equation gives:
7 Speed before first impact:
1.5 – y = 1.2 so y = 0.3
v2 = u2 + 2as
Exercise 6.1A = 02 + 2 × 10 × 3
v 2.6 = 60
1 e= = = 0.65
u 4 v = 7.75 m s−1
v 10 Speed immediately after first impact
2 e= = = 0.833
u 12
= 7.75 × 0.7 = 5.42 m s−1
v 3.2
3 u= = = 5.81 m s−1 Speed immediately after second impact
e 0.55
4 a v2 = u2 + 2as = 5.42 × 0.7 = 3.80 m s−1
= 02 + 2 × 10 × 1.5 Speed immediately after third impact
= 30 = 3.80 × 0.7 = 2.66 m s−1

v = 5.48 m s−1 Height reached after third impact:


b v = eu = 0.4 × 5.48 = 2.19 m s−1 v 2 − u 2 0 2 − 2.66 2
s= = = 0.353 m
2 2 2 2 2a 2 × −10
v −u 0 − 2.19
c s= = = 0.24 m So the maximum height reached after the third
2a 2 × −10
5 2 2
v = u + 2as bounce is 35.3 cm.

= 02 + 2 × 10 × 1.42 8 a Speed after impact with wall = 15 × 0.6 = 9 m s−1


= 28.4 b U = 9 m s−1, q = 0°, ay = 10 m s−2, y = 0 m, h = 1.2 m
v = 5.33 m s−1 1
Substitute into y = Ut sin q − gt2 + h
2
Speed immediately before impact is 5.33 m s−1
1
0 = 9 sin 0° × t − × 10 × t2 + 1.2
2

55
©HarperCollinsPublishers 2018 Cambridge International AS & A Level Mathematics: Further Mechanics 9780008271893

71893_P055_066.indd 55 14/06/18 11:15 PM


6 Momentum

Simplify: By Newton’s experimental law:


5t2 − 1.2 = 0 vB − vA = 0.8(0.5 − −0.9)
t2 = 0.24 vB − vA = 1.12
t = 0.490 or −0.490 Solving these simultaneous equations:
The time of flight for the ball is 0.490 seconds. 2vB = 0.72 so vB = 0.36 m s−1
c U= 9 m s−1, q = 0°, t = 0.4899 s 0.36 − vA = 1.12 so vA = −0.76 m s−1 i.e.
Substitute into x = Ut cos q 0.76 m s−1 in the opposite direction.
x = 9 × 0.490 × cos 0 mA = 5 kg, uA = 6 m s−1, mB = 3 kg, uB = −5 m s−1,
c 
= 4.41 e = 0.2
The ball will hit the ground 4.41 m from the By conservation of momentum:
base of the wall. 5 × 6 + 3 × −5 = 5 × vA + 3 × vB
9 U = ? m s−1, q = 0°, ay = 10 m s−2, y = 0 m, h = 2 m, t = ? 5vA + 3vB = 15
1 By Newton’s experimental law:
Substitute into y = Ut sin q − gt2 + h
2 vB − vA = 0.2(6 − −5)
0 = Ut sin 0° − 0.5 × 10 × t2 + 2
vB − vA = 2.2
t2 = 0.4
Solving these simultaneous equations:
t = 0.63246
8vB = 26 so vB = 3.25 m s−1
U = ? m s−1, q = 0°, t = 0.63246
3.25 − vA = 2.2 so vA = 1.05 m s−1
Substitute into x = Ut cos q
d mA = 4 kg, uA = 2 m s−1, mB = 3 kg, uB = −6 m s−1, e = 1
5 = U × 0.63246 × cos 0°
By conservation of momentum:
0.63246U = 5
4 × 2 + 3 × −6 = 4 × vA + 3 × vB
U = 7.906
4vA + 3vB = −10
The ball leaves the wall with a speed of 7.906 m s−1.
By Newton’s experimental law:
So the speed before impact is 7.906 = 9.88 m s−1.
0.8 vB − vA = 1(2 − −6)
Exercise 6.2A vB − vA = 8
Solving these simultaneous equations:
1 mA = 3 kg, uA = 5 m s−1, mB = 4 kg,
a 
7vB = 22 so vB = 3.14 m s−1
uB = 2 m s−1, e = 0.4
3.14 − vA = 8 so vA = −4.86 m s−1 i.e. 4.86 m s−1 in
By conservation of momentum:
the opposite direction
3 × 5 + 4 × 2 = 3 × vA + 4 × vB
2 mA = 2 kg, uA = 3 m s−1, mB = 3 kg, uB = 0 m s−1, e = 0.7
3vA + 2vB = 23
By conservation of momentum:
By Newton’s experimental law:
2 × 3 + 3 × 0 = 2 × vA + 3 × vB
vB − vA = 0.4(5 − 2)
2vA + 3vB = 6
vB − vA = 1.2
By Newton’s experimental law:
Solving these simultaneous equations:
vB − vA = 0.7(3 − 0)
7vB = 26.6 so vB = 3.8 m s−1
vB − vA = 2.1
3.8 − vA = 1.2 so vA = 2.6 m s−1
Solving these simultaneous equations:
mA = 1 kg, uA = 0.5 m s−1, mB = 1 kg,
b 
5vB = 10.2 so vB = 2.04 m s−1 in the same direction
uB = −0.9 m s−1, e = 0.8
that A was originally moving in.
By conservation of momentum:
3 mA = 4m kg, uA = 5 m s−1, mB = 5m kg, uB = −5 m s−1,
1 × 0.5 + 1 × −0.9 = 1 × vA + 1 × vB
e = 0.55
vA + vB = −0.4
By conservation of momentum:
4m × 5 + 5m × −5 = 4m × vA + 5m × vB
4vA + 5vB = −5
56
©HarperCollinsPublishers 2018 Cambridge International AS & A Level Mathematics: Further Mechanics 9780008271893

71893_P055_066.indd 56 14/06/18 11:15 PM


6
WORKED SOLUTIONS

By Newton’s experimental law: By conservation of momentum:


vB − vA = 0.55(5 − −5) m × 4 + m × 0 = m × vA + m × vB
vB − vA = 5.5 vA + vB = 4
Solving these simultaneous equations: By Newton’s experimental law:
9vB = 17 so vB = 1.89 m s−1 vB − vA = 0.6(4 − 0)
1.89 − vA = 5.5 so vA = −3.61 m s−1 vB − vA = 2.4
So both balls are moving in the opposite Solving these simultaneous equations:
direction compared to before the collision with 2vB = 6.4
vA = −3.61 m s−1 and vB = 1.89 m s−1.
vB = 3.2 m s−1
4 mA = 4 kg, uA = 5 m s−1, mB = 3 kg, uB = 0 m s−1, e = 0.6 Second impact:
By conservation of momentum: mB = m kg, uB = 3.2 m s−1, mC = m kg, uC = 0 m s−1,
4 × 5 + 3 × 0 = 4 × vA + 3 × vB e = 0.6
4vA + 3vB = 20 By conservation of momentum:
By Newton’s experimental law: m × 3.2 + m × 0 = m × vB + m × vC
vB − vA = 0.6(5 − 0) vB + vC = 3.2
vB − vA = 3 By Newton’s experimental law:
Solving these simultaneous equations: vC − vB = 0.6(3.2 − 0)
7vB = 32 so vB = 4.57 m s−1 vC − vB = 1.92
4.57 − vA = 3 so vA = 1.57 m s−1 Solving these simultaneous equations:
So both balls are moving in the same direction 2vC = 5.12
with vA = 1.57 m s−1 and vB = 4.57 m s−1. vC = 2.56 m s−1
5 mA = 2m kg, uA = u m s−1, mB = 3m kg,
a 
uB = −3u m s−1, e = e Exercise 6.3A
By conservation of momentum: 1 Parallel to the surface:
2m × u + 3m × −3u = 2m × vA + 3m × vB Speed before = 4.5 cos 30°, speed after = 4.5 cos 30°
2mvA + 3mvB = −7mu Perpendicular to the surface:
2vA + 3vB = −7u Speed before = 4.5 sin 30°,
By Newton’s experimental law: speed after = 0.45 × 4.5 sin 30° = 2.025 sin 30° m s−1
vB − vA = e(u − −3u) Velocity after = (4.5cos30°)2 + 1.01252 = 4.03 m s−1
2.025 sin 30° 
vB − vA = 4eu Direction θ = tan −1  = 14.6° above
 4.5cos 30° 
Solving these simultaneous equations: the horizontal
2vA + 3vB = −7u 2 Parallel to the surface:
2vB − 2vA = 8eu Speed before = 12 cos 50°, speed after = 12 cos 50°
5vB = 8eu − 7u Perpendicular to the surface:
u Speed before = 12 sin 50°,
vB = (8e − 7)
5
speed after = 0.4 × 12 sin 50° = 4.8 sin 50°
b vB − vA = 4eu
u  (12cos 50°)2 + (4.8sin 50°)2
Velocity after =
(8e − 7) − vA = 4eu = 8.55 m s−1
5
4.8sin 50° 
8eu − 7u − 5vA = 20eu Direction θ = tan −1  = 25.5° above
 12cos 50° 
5vA = −12eu − 7u the horizontal
−u
vA = (12e + 7) 3 Parallel to the surface:
5
6 First impact: Speed before = 2.5 cos 80°,
mA = m kg, uA = 4 m s−1, mB = m kg, uB = 0 m s−1, speed after = 2.5 cos 80°
e = 0.6
57
©HarperCollinsPublishers 2018 Cambridge International AS & A Level Mathematics: Further Mechanics 9780008271893

71893_P055_066.indd 57 14/06/18 11:15 PM


6 Momentum

Perpendicular to the surface: speed after = 0.6 × 3.2 sin 40°


Speed before = 2.5 sin 80°, = 1.92 sin 40°
speed after = 0.7 × 2.5 sin 50° = 1.75 sin 50° Velocity after = (3.2cos 40°)2 + (1.92sin 40°)2
Velocity after = (2.5cos 80°)2 + (1.75sin 80°)2 = 2.74 m s−1
1.92sin 40° 
= 1.78 m s−1 Direction θ = tan −1  = 26.7°
Direction θ = tan −1 (
1.75sin 80°
2.5cos80°
= 75.9°) above the horizontal
 3.2cos 40° 

to the wall.
U 2 sin 2 θ
4 Parallel to the cushion: b Maximum height =
2g
Speed before = 4 cos 72°, speed after = 4 cos 72° 2.74452 sin 226.7234°
=
Perpendicular to the cushion: 20
= 0.0762 m or 7.62 cm
Speed before = 4 sin 72°,
U 2 sin 2θ
speed after = 0.78 × 4 sin 72° = 3.12 sin 72° c Range =
g
 (4cos 72°)2 + (3.12sin 72°)2
Velocity after =
2.74452 sin 53.45°
= 3.21 m s−1 = 10
3.12sin 72° 
Direction θ = tan −1  = 67.4° between = 0.605 m or 60.5 cm
 4cos 72° 
the direction of the ball and the cushion 9 a Time of flight:
y = Ut sin q − 1 gt2 + h
5 Let a be the angle at which the particle hit the
a  2
surface. 0 = 5.1t sin 30° − 0.5 × 10 × t2 + 2

tan 30° = 0.3 tana so a = 62.5° 5t2 − 2.55t − 2 = 0


2.55 ± (−2.55)2 − 4 × 5 × −2
b Parallel to the surface: t=
2×5
Speed after = 3.2 cos 30°, t = −0.427 or t = 0.937
speed before = 3.2 cos 30° So the ball is in air for 0.937 seconds before
Perpendicular to the surface: it hits the ground.
Speed after = 3.2 sin 30°, So the distance to point of impact is
speed before = 3.2 sin 30° ÷ 0.3 = 10.67 sin 30° x = Ut cos q = 5.1 × 0.937 × cos 30° = 4.14 m.
Velocity before = (3.2cos 30°)2 + (10.67sin 30°)2 At point of impact:
= 6.01 m s−1 Horizontal velocity = 5.1 cos 30° = 4.42 m s−1
Vertical velocity = U sin q − gt
6 a Parallel to the surface:
= 5.1 sin 30° − 10 × 0.937
Speed before = 3.2 cos 36°, speed after = 3.2 cos 36°
= −6.82 m s−1
Also speed after = 2.8 cos a so i.e. 6.82 m s−1 downwards
3.2cos 36° 
α = cos−1  = 22.4° Horizontal velocity before bounce = 4.42 m s−1
 2.8 
so horizontal velocity after bounce = 4.42 m s−1
b tan 22.393° = e tan 36°
Vertical velocity before bounce = 6.82 m s−1
e = 0.567
so vertical velocity after bounce = 0.7 × 6.82
7 Let a be the angle between the wall and the ball = 4.77 m s−1
before the collision. Resultant velocity = 4.422 + 4.772 = 6.50 m s−1
Then 0.88 tan a = tan 50° Angle after bounce = tan −1
4.77
( )
= 47.2°
( )
4.42
So α = tan −1 tan 50° Distance to highest point
0.88
U 2 sin 2θ
a = 53.6° = 0.5 × range = 2g
8 a Parallel to the surface: 2
6.50 sin 94.4°
=
Speed before = 3.2 cos 40°, speed after = 3.2 cos 40° 20
= 2.11 m
Perpendicular to the surface:
So total horizontal distance = 4.14 + 2.11
Speed before = 3.2 sin 40°,
= 6.25 m

58
©HarperCollinsPublishers 2018 Cambridge International AS & A Level Mathematics: Further Mechanics 9780008271893

71893_P055_066.indd 58 14/06/18 11:15 PM


6
WORKED SOLUTIONS

b Time of flight to bounce = 0.937 s 2 2 2 2


= U sin θ = 10.37 sin 15.355° = 0.377 m,
Time from bounce to catch 2g 20
which is less
= 0.5 × 2U sin θ = 13sin 47.2°
g 20 than 0.5 m
= 0.477 seconds So the maximum height of the ball is less than
So total time of flight = 0.937 + 0.477 = 1.41 s 50 cm for the first time after the third bounce.

c Motion occurs only in two dimensions.


Exercise 6.4A
Air resistance is negligible.
Gravity remains constant. 1 a 
Perpendicular to the line joining the centres of
the spheres:
There is no spin applied to the ball.
Sphere A: speed after collision =
 3 sin 52°
10 After each bounce, the horizontal velocity will = 2.364 m s−1
remain 10 m s−1.
Sphere B: speed after collision = 0 m s−1
Find time of first flight by substituting y = 0 into
1 Parallel to the line joining the centres of the
y = Ut sin q − 2 gt2 + h
spheres:
0 = 10t sin 0° − 0.5 × 10 × t2 + 5
By the conservation of linear momentum:
5t2 = 5
3 cos 52° = vA + vB
t = 1s
By Newton’s experimental law:
Vertical velocity = U sin q − gt
= 10 sin 0° − 10 × 1 = 10 m s−1 downwards vB − vA = 0.6 × 3 cos 52°

After first bounce: vx = 10 m s−1, vB − vA = 1.8 cos 52°


vy = 0.65 × 10 = 6.5 m s−1 Solving simultaneously:
2 2
Resultant velocity = 10 + 6.5 = 11.927 m s−1 2vB = 4.8 cos 52°
Angle after bounce = tan −1
6.5
10 ( )
= 33.024° vB = 1.478 m s−1
vA + 1.478 = 3 cos 52°
Maximum height after bounce
2 2 2 2 vA = 0.369 m s−1 to 3 s.f.
= U sin θ = 11.927 sin 33.024° = 2.11 m, which is
2g 20 Sphere A:
greater than 0.5 m
 2.3642 + 0.3692
Velocity after impact =
Second stage of journey: = 2.39 m s−1
Immediately before second bounce:
vx = 10 m s−1, vy = 6.5 m s−1
a = tan−1 (
2.364
0.369 )
= 81.1° above the line

After second bounce: vx = 10 m s−1, joining the centres of the two spheres.
vy = 0.65 × 6.5 = 4.225 m s−1 Sphere B:
2 2
Resultant velocity = 10 + 4.225 = 10.856 m s−1 Sphere B is travelling at a velocity of
Angle after bounce = tan ( )
−1 4.23
10 = 22.904°
1.48 m s−1 parallel to the line joining the
centres of the two spheres.
Maximum height after bounce
U 2 sin 2 θ 10.8562 sin 2 22.904° b 
Perpendicular to the line joining the centres
= = = 0.893 m, which is
2g 20 of the spheres:
greater than 0.5 m Sphere A: speed after collision =
 10 sin 45°
Third stage of journey: = 7.0711 m s−1
Immediately before third bounce: vx = 10 m s−1, Sphere B: speed after collision =
 8 sin 30°
vy = 4.225 m s−1 = 4 m s−1
After third bounce: vx = 10 m s−1, Parallel to the line joining the centres of the
vy = 0.65 × 4.225 = 2.746 m s−1 spheres:
2 2
Resultant velocity = 10 + 2.746 = 10.37 m s−1 By the conservation of linear momentum:
Angle after bounce = tan (
−1 2.746
)
10 = 15.355°
10 cos 45° − 8 cos 30° = vA + vB

Maximum height after bounce vA + vB = 0.1429

59
©HarperCollinsPublishers 2018 Cambridge International AS & A Level Mathematics: Further Mechanics 9780008271893

71893_P055_066.indd 59 14/06/18 11:15 PM


6 Momentum

By Newton’s experimental law: Velocity after impact = 12 + 2.49642 = 2.69 m s−1


vB − vA = 0.5 × (10 cos 45° − (−8 cos 30°))
vB − vA = 6.9996
1
( )
a = tan−1 2.5 = 21.8° to the line joining the
centres of the two spheres in the opposite
Solving simultaneously: direction to B (upwards).
2vB = 7.1425
d Perpendicular to the line joining the centres of
vB = 3.5713 m s−1 the spheres:
vA + 3.5713 = 0.1429 Sphere A: speed after collision = 5 sin 60°
vA = −3.4284 m s−1 = 4.3301 m s−1 (to the left)
Sphere A: Sphere B: speed after collision = 4 sin 40°
2
 7.0711 + (−3.4284)
Velocity after impact = 2 = 2.5712 m s−1 (to the right)
= 7.86 m s−1 Parallel to the line joining the centres of the
a = tan−1 (
7.0711
3.4284 )
= 64.1° below the line joining
spheres:
By the conservation of linear momentum:
the centres of the two spheres.
5 cos 60° − 4 cos 40° = vA + vB
Sphere B:
vA + vB = −0.5642
Velocity after impact = 4 2 + 3.57132 = 5.36 m s−1
( )
3.5713 By Newton’s experimental law:
a = tan−1 = 41.8° below the line joining
4 vB − vA = 0.3 × (5 cos 60° − (−4 cos 40°))
the centres of the two spheres in the opposite vB − vA = 1.6693
direction to B.
Solving simultaneously:
c 
Perpendicular to the line joining the centres of 2vB = 1.1051
the spheres:
vB = 0.5526 m s−1
Sphere A: speed after collision = −3 sin 25°
vA + 0.5526 = −0.5642
= −1.2679 m s−1 (to the left)
vA = −1.1168 m s−1
Sphere B: speed after collision = 2 sin 30°
= 1 m s−1 (to the right) Sphere A:
Parallel to the line joining the centres of the Velocity after impact = (4.3301)2 + (−1.1168)2
spheres: = 4.47 m s−1
By the conservation of linear momentum:
3 cos 25° − 2 cos 30° = vA + vB
a = tan−1 ( 1.1168
4.3301
) = 75.5° to the line joining the
centres of the two spheres
vA + vB = 0.9869
Sphere B:
By Newton’s experimental law:
 2.57122 + 0.55262
Velocity after impact =
vB − vA = 0.9 × (3 cos 25° − (−2 cos 30°)) = 2.63 m s−1
vB − vA = 4.0059
Solving simultaneously:
( )
a = tan−1 2.57 = 77.9° to the line joining the
0.56
centres of the two spheres in the opposite
2vB = 4.9928 direction to B.
vB = 2.4964 m s−1
2 Perpendicular to the line joining the centres of the
vA + 2.4964 = 0.9869 spheres:
vA = −1.5095 m s−1 Red ball: speed after collision = 0.5 sin 15° = 0.129 m s−1
Sphere A: Blue ball: speed after collision = 0 m s−1
2 2
 (−1.2679) + (−1.5095)
Velocity after impact = Parallel to the line joining the centres of the spheres:
= 1.97 m s−1

( )
By the conservation of linear momentum:
a = tan−1 1.2679 = 40.0° to the line joining the
1.5095 0.5 cos 15° = vR + vB
centres of the two spheres (downwards) vR + vB = 0.4830
Sphere B: By Newton’s experimental law:

60
©HarperCollinsPublishers 2018 Cambridge International AS & A Level Mathematics: Further Mechanics 9780008271893

71893_P055_066.indd 60 14/06/18 11:15 PM


6
WORKED SOLUTIONS

vB − vR = 0.3 × (0.5 cos 15° − 0) Before the collision there was an angle of 157°
vB − vR = 0.1449 between their directions of motion.
Solving simultaneously: 4 Perpendicular to the line joining the centres of the
2vB = 0.6279 spheres:
vB = 0.314 m s−1 Sphere A: speed before collision = 0 m s−1
vR + 0.314 = 0.483 Sphere B: speed after collision = u m s−1
vR = 0.169 m s−1 Parallel to the line joining the centres of the spheres:
Red ball: By the conservation of linear momentum:
2
 (0.129) + (0.169)
Velocity after impact = 2 u = vA + vB
= 0.213 m s−1 By Newton’s experimental law:
a = tan−1 (
0.129
0.169 )
= 37.4° to the line joining the vB − vA = −0.5u
centres of the two spheres Solving simultaneously:
Blue ball moves with a velocity of 0.314 m s−1 parallel 2vB = 0.5u
to the line joining the centres. vB = 0.25u m s−1
3 Perpendicular to the line joining the centres of the vA + 0.25u = u
spheres: vA = 0.75u m s−1
Marble A: speed after collision =
 2 sin 25° Sphere A will travel at 0.75u m s−1 parallel to the
= 0.8452 m s−1 line joining the centres of the two spheres.
Marble B: speed after collision =
 1.5 sin 30° Sphere B:
= 0.75 m s−1 Velocity after impact = u 2 + (0.25u)2 = 1.03u m s−1
Parallel to the line joining the centres of the
spheres:
a = tan−1( u
0.25u )
= 76.0 (3 s.f.) to the line joining
the centres of the two spheres.
By the conservation of linear momentum:
5 4 5
uA + uB = −2 cos 25° + 1.5 cos 30° 5 If tan q = then cos θ = and sin q = .
4 41 41
uA + uB = −0.5136 Perpendicular to the line joining the centres of the
spheres:
By Newton’s experimental law:
Sphere A: speed after collision = −16 sin q
1.5 cos 30° − (−2 cos 25°) = 0.4 × (uA − uB)
= −12.4939 m s−1
0.4uA − 0.4uB = 3.1117
Sphere B: speed after collision = 10 sin q = 7.8087 m s−1
uA − uB = 7.7791
Parallel to the line joining the centres of the spheres:
Solving simultaneously:
By the conservation of linear momentum:
2uA = 7.2655
4 × 16 cos q − 10 cos q = 4vA + vB
uA = 3.633 m s−1
4vA + vB = 33.7335
uB + 3.633 = −0.5136
By Newton’s experimental law:
uB = −4.147 m s−1
vB − vA = 0.22 × (16 cos q − (−10 cos q))
Marble A:
vB − vA = 3.5733
Velocity before impact = (0.845)2 + (3.633)2
Solving simultaneously:
= 3.73 m s−1
(
0.845
)
a = tan−1 3.633 = 13.1° to the line joining the
5vB = 48.0267
vB = 9.605 m s−1
centres of the two marbles
4vA + 9.605 = 33.7335
Marble B:
vA = 6.032 m s−1
 0.752 + (−4.147)2
Velocity before impact =
Sphere A:
= 4.21 m s−1

( )
a = tan−1 0.75 = 10.3° to the line joining the
4.147
Velocity after impact = (−12.494)2 + (6.032)2
= 13.9 m s−1
centres of the two marbles.

61
©HarperCollinsPublishers 2018 Cambridge International AS & A Level Mathematics: Further Mechanics 9780008271893

71893_P055_066.indd 61 14/06/18 11:15 PM


6 Momentum

( )
a = tan−1 12.5 = 64.2° to the line joining the
6.03
256 – 163.84 sin2 q = 144
163.84 sin2 q = 112
centres of the two spheres.
sin q = 0.8268
Sphere B:
q = 55.8º
Velocity after impact = 7.8092 + 9.6052
= 12.4 m s−1 4 mA = 3m kg, uA = u m s−1, mB = 5m kg, uB = 0, e = e
a = tan−1 ( )
7.81
9.60
= 39.1° to the line joining the By conservation of momentum:
3m × u + 5m × 0 = 3m × vA + 5m × vB
centres of the two spheres in the opposite
direction to B. 3vA + 5vB = 3u
After the collision there will be an angle of 76.7° By Newton’s experimental law:
between their directions of motion. vB − vA = e(u − 0)
vB − vA = eu
Exam-style questions
Solving these simultaneous equations:
1 mA = 5m kg, uA = 2u m s−1, mB = 3m kg, uB = –4u, e = e 8vB = 3u(1 + e) so vB = 3 u(1 + e)
8
By conservation of linear momentum vB − vA = eu
10mu – 12mu = 5mvA + 3mvB 3 u(1 + e) − v = eu
–2u = 5vA + 3vB 8 A

3 3
By Newton’s experimental law: vA = u + eu − eu
8 8
vB – vA = e (2u + 4u) 3 3
vA = u + eu − eu
8 8
vB – vA = 6eu
vA = 3 u − 5 eu
Solving these simultaneous equations: 8 8
8vB = 30eu – 2u 1
vA = u(3 − 5e)
8
u
vB = (15e − 1)
4 5 a Parallel to the surface:
For B to travel in the opposite direction we Speed before = 5.2 cos 42°, speed after = 5.2 cos 42°
need vB to be greater than 0.
1 Perpendicular to the surface:
This will be when e >
15 Speed before = 5.2 sin 42°, speed after = 5.2e sin 42°
2 Parallel to the surface:
Velocity after = (5.2cos42°)2 + (5.2e sin 42°)2 = 3.9
Speed before = 5 cos 45°, speed after = 5 cos 45°
14.933 + 12.107e2 = 15.21
Perpendicular to the surface:
e = 0.151
Speed before = 5 sin 45°, speed after = 5e sin 45°
b tan q = 0.1513 × tan 42°
Velocity after = (5cos45°)2 + (5e sin 45°)2 = 4
q = 7.76°
(5 cos 45°)2 + (5e sin 45°)2 = 16
6 a Speed before first impact:
12.5 + 12.5e2 = 16
v2 = u2 + 2as
12.5(1 + e2) = 16
= 02 + 2 × 10 × 2
e = 0.529
= 40
3 Parallel to the wall:
v = 6.32 m s−1
Before = 16 cos q, speed after = 16 cos q
b Speed immediately after first impact :
Perpendicular to the wall:
v2 = u2 + 2as
Speed before = 16 sin q, speed after = 9.6sin q
02 = u2 + 2 × −10 × 1.2
After the collision v = 12 m s−1
u2 = 24
So (16cos θ )2 + (9.6sin θ )2 = 12
u = 4.90 m s−1
256 cos2 q + 92.16 sin2 q = 144
4.90
256 (1 – sin2 q) + 92.16 sin2 q = 144 c e = 6.32 = 0.775

62
©HarperCollinsPublishers 2018 Cambridge International AS & A Level Mathematics: Further Mechanics 9780008271893

71893_P055_066.indd 62 14/06/18 11:15 PM


6
WORKED SOLUTIONS

7 a Speed before first impact: b Speed of B before impact with wall = 3.4 m s−1
v2 = u2 + 2as Speed of B after impact with wall
= 02 + 2 × 10 × 6 = 0.5 × 3.4 = 1.7 m s−1
= 120 10 a This is motion in a straight line.
v = 10.9545 m s−1 mA = 0.05 kg, uA = 4 m s−1, mB = 0.04 kg, uB = 0 m s−1,
Speed immediately after first impact e = ?, vA = 1 m s−1
= 10.9545 × 0.8 = 8.764 m s−1 By the conservation of linear momentum:
Height reached after first impact: 4 × 0.05 + 0 × 0.04 = 0.05 × 1 + 0.04vB
v −u2 2
0 − 8.764 2 2 0.15
s= = = 3.84 m vB = = 3.75 m s−1
2a 2 × −10 0.04
v −u b By Newton’s experimental law:
b Time between bounces =
 2×
a 3.75 − 1 = e( 4 − 0)
0 − 8.764
=2× = 1.75 s e = 0.688
−10
c Speed immediately before second impact is 11 a tan 30° = e tan 60°
8.76 m s−1 1
e=
3
8 a Speed after impact = 0.7 × 12 = 8.4 m s−1
b Parallel to the surface:
b Lands when y = 0
Speed before = 5u cos 60°, speed after = 5u cos 60°
y = Ut sin q − 1 gt2 + h Perpendicular to the surface:
2
5
0 = 8.4 × t × sin 0 − 1 ×10 × t2 + 2 Speed before = 5u sin 60°, speed after = u sin 60°
3
2
5t2 = 2
( )
2
t = 0.632 s Speed after = (5u cos 60°)2 + 5 u sin 60° = lu
3
c At time of landing: 25 u2 + 25 u2 = l2u2
vx = U cos q = 8.4 cos 0 = 8.4 m s−1 4 12
25 + 25 = l2
vy = U sin q − gt = 8.4 sin 0° − 10 × 0.63246 4 12
= 6.3246 m s−1 downwards l = 2.89
After impact: 12 a Time of flight:
vx = 8.4 m s−1 1
y = Ut sin q − gt2 + h
2
vy = 0.5 × 6.3246 = 3.1623 m s−1 0 = 4t sin 40° − 0.5 × 10 × t2 + 3
So speed = 8.4 2 + 3.16232 = 8.976 m s−1 5t2 − 2.57115t − 3 = 0

8.4 (
θ = tan −1 3.1623 = 20.63° ) t=
2.57115 ± (−2.57115)2 − 4 × 5 × −3
2×5
2 × 8.976 × sin 20.63
T= = 0.633 s t = − 0.559 s or t = 1.07327 s
10
9 mA = m kg, uA = 4 m s−1, mB = m kg, uB = 0 m s−1,
a  So the ball is in air for 1.07327 seconds
e = 0.7 before it hits the ground.

By conservation of momentum: At point of impact:


Horizontal velocity = 4 cos 40° = 3.0642 m s−1
m × 4 + m × 0 = m × vA + m × vB
Vertical velocity = U sin q − gt
vA + vB = 4
= 4 sin 40° − 10 × 1.07327 = −8.1615 m s−1
By Newton’s experimental law:
i.e. 8.1615 m s−1 downwards
vB − vA = 0.7(4 − 0)
Horizontal velocity before bounce = 3.0642 m s−1
vB − vA = 2.8 so horizontal velocity after bounce = 3.0642 m s−1
Solving these simultaneous equations: Vertical velocity before bounce = 8.1615 m s−1 so
2vB = 6.8 vertical velocity after = 0.6 × 8.1615 = 4.8969 m s−1
vB = 3.4 m s−1 Resultant velocity after bounce
3.4 − vA = 2.8 = 3.06422 + 4.89692 = 5.78 m s−1
vA = 0.6 m s−1 Angle after bounce = tan −1 ( )
4.88
3.06
= 58.0°
63
©HarperCollinsPublishers 2018 Cambridge International AS & A Level Mathematics: Further Mechanics 9780008271893

71893_P055_066.indd 63 14/06/18 11:15 PM


6 Momentum

b Time of flight to first bounce = 1.07327 s Solving simultaneously:


Distance travelled to first bounce = Ut cos q 5vB = 1.1049
= 4 × 1.07327 × cos 40° = 3.289 m vB = 0.22098 m s−1
2U sin θ
Time of flight to second bounce = 0.22098 − vA = 1.01771
g
2 × 5.7766 sin 57.964 ° vA = −0.79673 m s−1
= = 0.9794 s
10
Sphere A:
Distance travelled to first bounce = Ut cos q
= 5.7766 × 0.9794 × cos 57.964° = 3.001 m Velocity after impact = (2.0075)2 + (−0.7967)2
Total horizontal distance = 6.29 m = 2.16 m s−1

13 Time to first bounce:


a= tan−1 (
2.0075
0.7967 )
= 68.4° to the line joining

v2 = u2 + 2as the centres of the two spheres


= 02 + 2 × 10 × 10 Sphere B:
v = 10 2 Velocity after impact = (1.8641)2 + (0.221)2
  = 1.88 m s−1
( )
Time to reach ground after dropping: 1.8641
v − u 10 2 a = tan−1 = 83.2° to the line joining
0.221
t= = = 2
a 10 the centres of the two spheres
Speed after first bounce = e 2
b Sphere B will hit the wall at an angle of 83.24°
Speed after second bounce = e 2 2 , etc.
between the wall and the direction of travel with
2u
Time of flight = a speed of 1.877 m s−1.
g
So total time of flight Parallel to the surface:
20e 2 20e 2 2 20e 3 2 Speed before = 1.877 cos 83.24° = 0.2209 m s−1,
= 2+ + + +…
10 10 10 speed after = 0.2209 m s−1
(
= 2 + 2 2e + 2e 2 + 2e 3 + … ) Perpendicular to the surface:
= 2 + 2 × 2e Speed before = 1.877 sin 83.24° = 1.864 m s−1,
1−e speed after = 0.6 × 1.864 = 1.118 m s−1
( )
= 2 1 + 2e
1−e Velocity after = 0.22092 + 1.1182 = 1.14 m s−1

( )
= 2 (1 − e) + 2e
(1 − e) (1 − e) ( )
Direction = tan −1 1.118 = 78.8° between the
0.2209
2(
1− e)
= 1+e wall and the direction of travel
15 a mA = 3 kg, uA = 4 m s−1, mB = 4 kg, uB = −2 m s−1,
14 a Perpendicular to the line joining the centres e = 0.2
of the spheres:
By conservation of momentum:
Sphere A: speed after collision = 3.5 sin 35° 3 × 4 + 4 × −2 = 3 × vA + 4 × vB
= 2.0075 m s−1
3vA + 4vB = 4
Sphere B: speed after collision = 2.9 sin 40°
By Newton’s experimental law:
= 1.8641 m s−1
vB − vA = 0.2(4 − −2)
Parallel to the line joining the centres of the
spheres: vB − vA = 1.2

By the conservation of linear momentum: Solving these simultaneous equations:

2m × 3.5 cos 35° − 3m × 2.9 cos 40° = 2mvA + 3mvB 7vB = 7.6

2vA + 3vB = −0.93052 vB = 1.09 m s−1


1.086 − vA = 1.2
By Newton’s experimental law:
vA = −0.114 m s−1
vB − vA = 0.2 × (3.5 cos 35° − (−2.9 cos 40°))
vB − vA = 1.01771

64
©HarperCollinsPublishers 2018 Cambridge International AS & A Level Mathematics: Further Mechanics 9780008271893

71893_P055_066.indd 64 14/06/18 11:15 PM


6
WORKED SOLUTIONS

b Time ball B is in the air: Collision between B and C


1
s = ut + at 2 By conservation of linear momentum
2
1 3 × 0.4375 + 1 × 0 = 3vB + vC
0.8 = 0 × t + × 10 × t2
2 3vB + vC = 1.3125
So t2 = 0.16 By Newton’s experimental law:
So t = 0.4 s vC – vB = 0.8 (0.4375)
Horizontal range: vC – vB = 0.35
x = 1.086 × 0.4 × cos 0° 3vC – 3vB = 1.05
= 0.434 m Solving simultaneously,
So the ball will land 0.434 m away from the 4vC = 2.3625
base of the table.
vC = 0.591 m s−1
16 a Perpendicular to the line joining the centres
of the spheres: The speed of C after the collision with B is 0.591 m s−1.

Sphere A: speed after collision = u sin 50° 18 a By conservation of linear momentum


Sphere B: speed after collision = 0 m s−1 5 × 4 + 3 × 0 = 5vA + 4vB
Parallel to the line joining the centres of the 5vA + 4vB = 20
spheres: By Newton’s experimental law:
By the conservation of linear momentum: vB – vA = 0.8 (4)
u cos 50° = vA + vB vB – vA = 3.2
By Newton’s experimental law: 5vB – 5vA = 16
vB − vA = e × (u cos 50°) Solving simultaneously,
vB − vA = eu cos 50° 9vB = 36
Solving simultaneously: vB = 4 m s−1.
2vB = u cos 50° (1 + e) 4 – vA = 3.2
u vA = 0.8 m s−1 (i.e. in the same direction
vB = cos 50°(1 + e)
2
as before the collision)
vA = u cos 50° − u cos 50°(1 + e)
2
u b Parallel to the surface:
vA = cos 50°(1 − e)
2 Speed before = 4 cos 45°,
Velocity of A after impact speed after = 4 cos 45°
u2 Perpendicular to the surface:
cos2 50° (1 − e )
2
= u 2sin 2 50° +
4
Speed before = 4 sin 45°,
u
b Velocity of B after impact = cos 50°(1 + e) speed after = 4e sin 45°
2
17 mA = 5 kg, mB = 3 kg, mC = 1 kg Velocity after = (4cos 45°)2 + (4e sin 45°)2
Collision between A and B = 2.9
By conservation of linear momentum 8 + 8e 2 = 2.9
5 × 0.5 + 3 × 0 = 5vA + 3vB 8e2 = 0.41
5vA + 3vB = 2.5 e = 0.05125
By Newton’s experimental law:  = 0.226
vB – vA = 0.4 (0.5) c q = angle between direction of B and wall
after collision
vB – vA = 0.2
0.226 tan 45° = tan q
5vB – 5vA = 1
So q = 12.7º
Solving simultaneously,
8vB = 3.5
vB = 0.4375 m s−1.

65
©HarperCollinsPublishers 2018 Cambridge International AS & A Level Mathematics: Further Mechanics 9780008271893

71893_P055_066.indd 65 14/06/18 11:15 PM


6 Momentum

19 Collision between A and B Horizontal velocity before bounce = 10 m s−1 so


By the conservation of linear momentum horizontal velocity after bounce = 10 m s−1
1 × 2 + 1 × 0 = (1 + 1) vAB Vertical velocity before bounce = 6.32456 m s−1 so
vertical velocity after = 0.8 × 6.32456 = 5.0596 m s−1
2 = 2vAB
 10 2 + 5.05962
Resultant velocity after bounce =
vAB = 1 m s−1.
= 11.21 m s−1
Collision between AB and C
KE of AB before = 1 × 2 × 12 = 1 J
Angle after bounce = tan −1 (
5.0596
10 )
= 26.84°
2 To reach the assistant’s mouth, y = 1.1
1 1 1
KE of AB after = × 1 = J 1.1 = 11.21t sin 26.84° − × 10 × t2
3 3 2
1 5t2 − 5.0613t + 1.1 = 0
So velocity of AB after collision = 3
1 ×2 t = 0.316 or t = 0.696
= 0.5774 m s−1. 2
Total time taken to reach the assistant’s mouth
Collision between AB and C
is either 0.948 s or 1.33 s.
By the conservation of linear momentum
2 At these times, the distance since the bounce is:
2 × 1 + 1 × 0 = 2vAB + vC
distance = 11.21 × 0.316 × cos 26.84° = 3.161 m
2vAB + vC = 2
and distance = 11.21 × 0.696 × cos 26.84° = 6.962 m
By Newton’s experimental law
 istance between release and first bounce
D
vC – vAB = e (1)
= 10 × 0.6325 = 6.325 m
2vC – 2vAB = 2e
 o distance between the two entertainers is
S
Solving simultaneously, 9.49 m or 13.3 m.
3vC = 2 + 2e
vC = 2 (1 + e)
3
vAB = vC – e
= 2 (1 + e) − e
3
2 1
= − e
3 3
From above, vAB = 0.5774
2 1
− e = 0.5774
3 3
e = 0.268
Mathematics in life and work

1 Time of flight until first bounce:


1
y = Ut sin q − gt2 + h
2
0 = 10t sin 0° − 0.5 × 10 × t2 + 2
5t2 = 2
t = 0.632456 s
So the ball is in air for 0.632 seconds before it
hits the ground.
At point of impact:
Horizontal velocity = 10 m s−1
Vertical velocity = U sin q − gt
= 10 sin 0° − 10 × 0.632456 = −6.32456 m s−1
i.e. 6.32456 m s−1 downwards

66
©HarperCollinsPublishers 2018 Cambridge International AS & A Level Mathematics: Further Mechanics 9780008271893

71893_P055_066.indd 66 14/06/18 11:15 PM


WORKED SOLUTIONS

Summary Review
Please note: Full worked solutions are provided as an aid to learning, and represent one approach to answering
the question. In some cases, alternative methods are shown for contrast.
All sample answers have been written by the authors. Cambridge Assessment International Education bears no
responsibility for the example answers to questions taken from its past question papers, which are contained in this
publication.
Non-exact numerical answers should be given correct to 3 significant figures, or 1 decimal place for angles in
degrees, unless a different level of accuracy is specified in the question.
Warm-up questions Since F = 0.42R
F 2
1 tan a = 8 so cos a = 15 and sin a = 8 + P × = 5g  2
15 17 17 0.42 7
Resolving vertically: Substituting 1 in 2 and making P the subject:
15
T× =3 50
17 P= = 19.47 N
 745 2 
T = 3.4 N +
 0.42 7 
Resolving horizontally:  
8 Toppling:
F = 3.4 × = 1.6
17 3
4096 = 16 cm = 0.16m
2 i Using F = μR and resolving horizontally, R = W Moments about A:
So F = 1.25W 5g × 0.08 = P sin a × 0.16 + P cos a × 0.16
As W < F, the minimum vertical force to move 8P 0.16 45P
4= +
the block is less than the minimum horizontal 175 7
force to move the block. P = 20.10 N
ii Using Newton’s second law: P for sliding is less than P for toppling, so the cube
will slide first.
P − 1.25 × 60 = 6 × 4
2 i F = ma
P = 99
1
3 i Work done = Fd 0.4v 2 = 0.6 v dv
dx
1
= (30 cos a + 40 cos β) × 20 Dividing by 0.2v 2 :
= (30 × 0.6 + 40 × 0.8) × 20 = 1000 J 1
2 = 3 v 2 dv
ii 
Constant speed so friction is equal to driving dx
force and F = μR. 1
ii 2∫dx = 3∫v 2dv
Resolving vertically: R = W
2 3
F = μR. 2x = 3 × v 2 + c
3
5
50 = W 3
8 2x = 2v 2 + c
W = 80 N When x = 1, v = 1
2=2+c
A Level questions
So c = 0
1 sin a = 2 so cos a = 45 3
7 7 x =v2
Sliding: 2
v = x3
Resolving horizontally: 2
dx
P cos a = F iii = x3
dt
45 −2
F=
7
P 1 ∫x 3 dx = ∫dt
1
Resolving vertically:
=t +c
3x 3
R + P sin a = 5g When t = 0, x = 1
67
©HarperCollinsPublishers 2018 Cambridge International AS & A Level Mathematics: Further Mechanics 9780008271893

71893_P067_080.indd 67 15/06/18 8:16 PM


SUMMARY REVIEW

So c = 3 V cos 45° t = V cos 60° (t + 1)


1 cos 45° t = cos 60° (t + 1)
3x 3=t +3 t t +1
=
At B, x = 8 2 2
3×2=t+3 2t t + 1
=
t=3 2 2

3 Let vA and vB be the speeds of A and B after the


( )
2 −1 t =1

collision. t = 2 + 1 = 2.414 213 5 = 2.414 seconds to
Conservation of momentum: 3 decimal places, as required.
2mu = 2mvA + mvB Or using your calculator from line 2 in the above
2u = 2vA + vB 1 working:
Newton’s experimental law: 0.707 11t = 0.5(t + 1)
Make sure you write sufficient decimal places to
eu = vB – vA 2
show the answer is correct to 3 d.p.
Solving simultaneously:
As before, t = 2.414 to 3 decimal places as
2u – eu = 3vA required.
u(2 − e)
vA =
3 ii A
 t point of collision P and Q must have travelled
2u + 2eu = vB + 2vB the same distance vertically.
1
2u(1 + e) = 3vB Use s = ut + at 2 vertically.
2
2u(1 + e)
vB = Vertical distance =
3
gt 2 g(t + 1)2
Newton’s experimental law for B bouncing off wall: V sin45° t – = V sin60° (t + 1) −
2 2
0.4( 3 )
2u(1 + e) u(2 − e)
=
3 V sin45° t − V sin60° (t + 1) =
gt 2 g(t + 1)2
2

2
0.8(1 + e) = 2 – e Substitute in for t:
2 10(2.414)2 10(3.414)2
e= V 1.707 – V 2.957 = −
3 2 2
Speed = distance ÷ time −1.250 V = 29.13698 – 58.27698
For B (from first collision to wall): V = 23.3
10u d
= iii Greatest height,
9 t
9d U 2(sinq )2 23.32sin2 60°
t= H = = = 20.35
10u 2g 20
So the distance travelled by A during that time is At point of collision, use S = Ut + 1 at 2
2
given by dA where:
h = 23.3 × sin 60° × 3.414 − 5 × (3.414)2

4u d A
= = 68.89 – 58.28 = 10.61
9 9d
10u Vertical distance below greatest height = 9.74 m
4u 10ud A NB: You may have slightly different figures
=
9 9d depending on the number of decimal
2d
dA = places you work with.
5
When B reaches the wall, A has travelled 2d metres. 5 i tan q = 5 so cos q = 12 and sin q = 5
5 12 13 13
From the wall to the second collision, the particles are Conservation of energy (with bottom of circle as
travelling towards each other at the same constant zero potential energy):
speed, so (by symmetry), they will meet in the middle. mg × 2 = mg ( 2 − 2cos q ) + 1 mv 2
The distance between A and B is 3d , so B will be 2
5
3d metres away from the wall when the particles    20 = 20 − 20 × 12 + 1 v 2
13 2
10
1
  v = 2 240
collide again. 2 13
4 i 
At point of collision P and Q must have travelled    v = 480 ms−1
the same distance horizontally. 13

As acceleration = 0 horizontally, use S = Ut

68
©HarperCollinsPublishers 2018 Cambridge International AS & A Level Mathematics: Further Mechanics 9780008271893

71893_P067_080.indd 68 15/06/18 8:16 PM


WORKED SOLUTIONS

ii 
At B, the particle is h m above the ground,
7 i Let m be the mass of the particle.
where h = 3 − 2cos q = 15 .
13 T = 3mg
Let vG be the speed at impact with the ground. Resolving vertically:
Conservation of energy (with ground as zero T cos q = mg
potential energy): 3mg cos q = mg
8
Energy at B = energy at impact with the ground cos q = 1 so sin q =
3 3
15 1 480 1
10m × + m× = mvG2 By Newton’s second law:
13 2 13 2
150 240 1 2 T sin q = mrw 2
+ = v
13 13 2 G 8
780 3mg × = mr × 52
vG2 = 3
13
r = 1.13 m
780
vG = ms−1 Using trigonometry on the triangle:
13

iii Speed = distance ÷ time, so the horizontal sin q = r


l
displacement of the particle at time t is: 8 1.13…
=
 480 12  3 l
x= × t
 13 13  l = 1.2 m
 13 13  ii v = rw
t = x
 12 480  v = 1.13… × 5 = 5.66 m s–1
Using s = ut + 1 at 2 , the vertical displacement of 8 i
2 Shape Arc Rod Total
the particle at time t is:
 480 5 1 Mass (kg) 1.8π 3.6 1.8π + 3.6
y = ×  t + (−10)t 2
 13
()
13  2 Distance 0
1.8sin π
y
 5 480  of centre 2
y = t − 5t 2 π
 13 13  of mass
2
Substituting for t: from O (m)

(1312 ) ( 480 )x
2
13 2 5 480 13 13 Centre of mass:
y = −5 + × x
13 13 12 480 1.8 × 2
1.8π × = (1.8π + 3.6 ) y
2197 2 5 π
y=− x + x
1152 12 6.48 = y
480 2197 2 1.8π + 3.6
y= x− x
1152 1152      y = 0.700m
x
y=
1152
( 480 − 2197x ) ii
Shape Frame Lamina Total
6 Let x be the extension when the particle is initially
Mass (kg) M 2.75 M + 2.75
at rest.
By Hooke’s law:
28x
Distance
of centre
0.70…
2 × 1.8sin π
2 ()
= 2.4
y

T= = 17.5x of mass 3π π
1.6
2
Resolving vertically: from O
(m)
17.5x = 0.35g
x = 0.2 m y = 1.8tan 22° = 0.7272
Conservation of energy: Centre of mass:
2.75 × 2.4
28 × 0.22 1 1 0.7002 M + = ( 2.75 + M ) × 0.7272
+ × 0.35 × 1.8 2 = 0.35 × 10 × 0.2 + × 0.35v 2 π
2 × 1.6 2 2
6.6
0.35 + 0.567 = 0.7 + 0.175v2 0.7002 M + = 2.000 + 0.7272 M
π
v2 = 1.24 6.6 − 2.000
M= π = 3.74 kg
v = 1.11 m s–1 0.7272 − 0.7002
So the weight of the frame is 37.4 N.
69
©HarperCollinsPublishers 2018 Cambridge International AS & A Level Mathematics: Further Mechanics 9780008271893

71893_P067_080.indd 69 15/06/18 8:16 PM


SUMMARY REVIEW

9 ii 14 m s–1
R2
1m

q 1m
4m u m s–1
3m 2g 74.5° q
q
R1 e = 0.4
8g
Newton’s experimental law:
q 0.4 × ° = u=cos
× sin 74.5°
14 cos74.5 q q
u sin
F
u sin q = 1.4422 1
Motion parallel to the ground:
sin q = 3 and cos q = 7
4 4
14 cos74.5° = u cosq
Resolving vertically:
u cos q = 1 2
R1 = 10g = 100 N
Square and add 1 and 2 :
F = μR1 = 100μ
u2 = 1.44222 + 12 = 3.080
Resolving horizontally:
u = 1.75 m s–1 (3 sf)
F = R2 = 100μ
Substituting in (1):
Moments about the base of the ladder:
2 × 8g cos q + 3 × 2g cos q = 4R2 sin q sin q = 1.4422 = 0.82178
1.7550
220 ×
7 3
= 4 × × 100µ q = 55.3° (3 sf)
4 4
For vertical motion after the bounce: using
220 7 = 1200µ v2 = u2 + 2as
u = 1.755 sin 55.26° m s–1, a = –10 m s–2, s = ?,
µ = 11 7
60 v = 0 m s–1
10 i Using v2 = u2 + 2as 0 = 1.4422 + 2 ×(–10) × s
u = 2sin 60° = 3 m s−1, a = –10 m s–2, s = – 0.5, s = 1.4422 = 0.0721m = 7.21cm (3 sf )
20
v=?
v2 = 3 + 2 × (–10) × (–0.5) = 13 11 For complete circular motion, tension ⩾ 0 at the
top of the circle.
v = ± 13
By Newton’s second law:
As the ball is moving towards the ground,
mu 2
v = − 13 m s−1. T + mg =
a
Horizontal motion is a constant speed of mu 2
T= − mg
2 cos 60° = 1 m s−1. a
Let vG be the speed on impact with the ground. But T ⩾ 0, so
mu 2 − mg  0
By Pythagoras: a
vG2 = 12 + 13 u2  g
a
vG = 14 m s−1
u  ag
By trigonometry:
By conservation of energy:
tan α = 13 1 1
mg ( 2a ) + mu 2 = mv 2
a = 74.5° 2 2
The ball hits the ground at a speed of 14 m s−1 1 1 2
2ag + ag = v
2 2
at an angle of 74.5° to the horizontal ground. 5 1 2
ag = v
2 2

70
©HarperCollinsPublishers 2018 Cambridge International AS & A Level Mathematics: Further Mechanics 9780008271893

71893_P067_080.indd 70 15/06/18 8:16 PM


WORKED SOLUTIONS

v2 = 5ag Use the quadratic formula, taking only the


v = 5ag (where v is the speed of P before the positive root (the negative root is before the
collision) motion started)
t = 2.58 s
Let vC be the speed of the combined particle after
13 i Take moments about A
the collision.
10 × 1.2 sin (q – 30°) = 6 × 0.8 sinq
By conservation of momentum: Use:
5m 3 1
m 5ag =
4
× vC sin (q − 30° ) = sin q cos30° − cos q sin 30° = sin q − cos q
2 2
4  3 1 
vC = 5ag 12 ×  sin q − cos q  = 6 × 0.8sin q
5
 2 2 
By conservation of energy:
   3sin q − cos q = 0.8sin q
( )
1 5m
2 4
×
16
25
× 5ag =
5mg
4 (
a + a cos q ) +
2 4 ( )
1 5m 2
v ( )
3 − 0.8 sin q = cos q
1
5mga 5mga cos q 5mv 2 tanq = = 1.0729
2mga =
4
+
4
+
8 ( )
3 − 0.8
2
6mga = 10mga cos q + 5mv  1 q = 47.0°
By Newton’s second law:
ii Resolve horizontally
5mg cos q 5m v 2 Reaction at A = 10 sin 30° (as the rod is in
T + =
4 4 a equilibrium)
4aT + 5mga cos q = 5mv 2  2 Resolve vertically
2 in 1 : Friction force at A = 10cos 30° – 6
6mga = 10mga cos q + 4aT + 5mga cos q Use F ⩽ μR, Limiting equilibrium, so F = μR
10cos 30° – 6 = μ × 10 sin 30°
4aT = 6mga − 15mga cos q
(10cos30° − 6)
µ= = 0.532
3mg 15mg cos q 10sin30°
T = −
2 4
1 1 3 3
The string becomes slack when T = 0: 14 i At A: KE = 2 mv 2 = 2 m 10 ga = 2 ma ,
3 15
− cosq = 0 PE = mgh = 10ma
2 4
cos q = 6 = 2 1
At bottom: KE = 2 mv 2
15 5
12 i Horizontally acceleration = 0 so velocity is Conservation of energy:
3 1
constant and = 14cos 60° = 7 ma + 10ma = mv 2
2 2
Vertically use v = u + at 3a + 20a = v2
v = 14 sin 60° – 1.8g = –5.8756 v 2 = 23a
7 m s–1 v 2 23a
Acceleration towards centre = = = 23
q r a
F = ma = 23m
v 5.8756 m s–1
R = 23m + mg = 33m

ii At B: PE = mgh = 10m ( a − asin q ) = 30 ma,


4
v2 = 72 + 5.87562 = 83.52 1 2
v = 9.139 m s–1 KE = mv
2
tan q = 5.8756/7 Conservation of energy:
q = 40.0° 1 m × 23a = 30 + 1 mv 2
Speed = 9.14 m s–1 in direction 40.0˚ below the 2 4 2
horizontal 46ma = 30ma + 2mv2
v 2 = 8a
ii Use s = ut + 1 at 2 in vertical direction v = 8a = 2.83 a
2
−2 = (14sin60 )t − 1 gt 2
2
5t2 – 12.124t – 2 = 0

71
©HarperCollinsPublishers 2018 Cambridge International AS & A Level Mathematics: Further Mechanics 9780008271893

71893_P067_080.indd 71 15/06/18 8:17 PM


SUMMARY REVIEW

iii Vertical component of speed at B = 8a cos q iii A


 t top position, resultant force
λ x = 6 + 45 × 1 = 36
 sin q = 1 , ∴ cos q = 15 (Note: as q is acute = mg +
l 1.5
4 4
sin2 q + cos2 q = 1) F = ma, 0.6a = 36, a = 60 m s–2
Use v2 = u2 + 2aS for vertical motion of At bottom position, resultant force
projectile. λ x − mg = 45 × 1.22 − 6 = 30.6
=
At greatest height above point of projection v = 0 l 1.5
0 = 8acos2 q – 20S F = ma, 0.6a = 30.6, a = 51 m s–2
15a
20S = Greatest magnitude of the acceleration of P is
2
15a 3a 60 m s–2
S= =
40 8
At moment of projection let h be the distance of 16 i Vertically, resultant force = 0.6g – 3v
the bead below the level of O. Use F = ma with acceleration as dv to find t
dt
sin q = h
a 0.6g – 3v = 0.6 dv
Greatest height of bead above the level of O dt
dv
3a 1 1 0.6 = 0.6g − 3v
= − a = a as required. dt
8 4 8 Divide both sides by 0.6
15 i At point of release PE = mgh = 6h, KE = 0,       dv = g − 5v
2 dt
EE = λ x = 45 × 1 = 15
2l 2 × 1.5    1
When P comes to rest PE = 0, KE = 0, ∫ 10 − 5v dv = ∫dt
2 2
EE = λ x = 45 × h = 15h 2 1 −5 dv = dt
5 ∫ 10 − 5v ∫

2l 2 × 1.5
Conservation of energy,    − 1 ln (10 − 5v ) = t + c
15h2 = 15 + 6h 5
5h2 – 2h – 5 = 0 Substitute initial conditions, i.e. V = 0 when t = 0
1
Solve for h, taking the positive solution − ln (10 ) = c
5
h = 1.22 m 1 1
t = ln (10 ) − ln (10 − 5 × 1.95)
ii Greatest speed occurs at equilibrium position. 5 5
1 1
Let x be the distance between the point of t = ln (10 ) − ln ( 0.25)
5 5
release and the equilibrium position. 1
t = ln ( 40 )
Resolving vertically using Hooke’s Law 5
λ x = λ(1 − x) + mg   t = 0.738 seconds
l l ii Use F = ma with acceleration as v dv to find x
dx
45x = 45(1 − x) + 6
1.5 1.5   −3v = 0.6v dv
dx
45x = 45 – 45x + 9 dv = −5
90x = 54 dx
x = 0.6
Distance below A = 0.6 + 1.5 = 2.1m
∫dv = −∫5dx
   v = –5x + c
Energy at equilibrium position
S
 ubstitute initial conditions, i.e. v = 1.95 and
λ x 2 + λ (1 − x ) = 1 0.6v 2 + 45 × 0.6 2 + 45 (1 − 0.6 )
2 2
1
= mv 2 + x = 00.3v 2 + 5.4 + 2.4
2 2l 2l 2 2 × 1.5 2 × 1.5
c = 1.95
λ x + λ (1 − x ) = 1 0.6v 2 + 45 × 0.6 + 45 (1 − 0.6 ) = 0.3v 2 + 5.4 + 2.4
2 2 2 2
mv 2 + v = –5x + 1.95
2l 2l 2 2 × 1.5 2 × 1.5
× 0.6 2 45 (1 − 0.6 )
2
2 P comes to rest when v = 0
+ = 0.3v + 5.4 + 2.4
× 1.5 2 × 1.5 5x = 1.95
= 7.8 + 0.3v2
x = 0.39 m
Conservation of energy:
17 Let vA and vB be velocities of spheres A and B after
7.8 + 0.3v 2 = 0.6g × 0.6 + 45 × 1
2 × 1.5 impact, respectively.
7.8 + 0.3v2 = 3.6 + 15 Momentum before = momentum after
0.3v2 = 10.8 4mu = 4mvA + 2mvB
v2 = 36 2u = 2vA + vB 1
v = 6 m s–1
72
©HarperCollinsPublishers 2018 Cambridge International AS & A Level Mathematics: Further Mechanics 9780008271893

71893_P067_080.indd 72 15/06/18 8:17 PM


WORKED SOLUTIONS

e × (speed of approach) = speed of separation 18 i Horizontally, s = ut, x = (15cos41°)t


eu = vB – vA Vertically, s = ut + 1 at 2 , y = (15sin41°)t – 5t2
vB = eu + vA 2 2
Substitute t = x
Substitute 2 into Eqn 1 15cos41°

( )
2u = 2vA + eu + vA = eu + 3 vA 2
x x
y = (15sin41°) × −5
2u = eu + 3 vA 15cos41° 15cos41°

(2 – e) u = 3 vA  3 y = 0.869x – 0.0390x2
ii When x = 1.5
Consider the change in KE of sphere A
y = 0.869 × 1.5 – 0.0390 × 1.52 = 1.216
1 1 1
(4m)v 2A = × (4m)u 2 Remember to add the height of O above the
2 4 2
1 horizontal.
     v 2A = u 2  4
4 Height of fence, h = 1.6 + 1.216 = 2.82 m
Substitute 4 into the square of 3 Ball lands when y = –1.6
(2 – e)2 u2 = 9 (vA )2 –1.6 = 0.869x – 0.0390x2
1 0 = 1.6 + 0.869x – 0.0390x2
(2 – e)2 u2 = 9 u 2
4
Solve using the quadratic formula
4 (2 – e)2 = 9
x = 24.0
 4e2 – 16e + 7 = 0
Distance from fence to A = 24.0 – 1.5 = 22.5 m
  e 2 − 4e + 7 = 0
4 19 i For triangle, centre of mass lies on the line of
      e = 1 or e = 7 symmetry.
2 2
Height of triangle = 0.36 × 2 ÷ 0.6 = 1.2
1
As 0  e  1, e = Distance of centre of mass from base of
2
triangle = 1.2 ÷ 3 = 0.4
From above v A = 1 u 2, so v A = 1 u
2
For semicircle, centre of mass lies on line of
4 2
eu = vB – vA symmetry at a distance of 4r from the straight
1 1 3π
2
u = v B − u , so vB = u
2 edge: 4r = 0.8
3π π
Consider the collision between spheres B and C Let mass per sq metre be m
Let the speeds of spheres B and C after Take moments about the axis OB
collision be v'B and vC, respectively If the centre of mass of the lamina lies on OB
e × (speed of approach) = speed of separation then these moments will be equal.
1 For triangle: mg × 0.36 × 0.4 = 0.144mg
e × u = vC − v' B
2 2
1
v' B − vC = − u  For semicircle: mg × π0.6 × 0.8 = 0.144mg
4
5 2 π
Conservation of momentum Centre of mass lies on OB.
1
2mu + mu = 2mv 'B + mvC ii Let x be the distance of the centre of mass of the
2
lamina from the point O.
5 u = 2v' + v  6
2 B C Take moments about the axis OC:
For the parts: 0.36mg × 0.3. (Note: centre of
Solve 5 and 6 simultaneously
mass of semicircle is on this axis.)
5
2v' B + vC = u 2
2 For the whole: (0.36mg + π × 0.6 mg ) × x
   v' − v = − 1 u 2
B C 4 2
0.36mg × 0.3 = (0.36mg + π × 0.6 mg ) × x
     3v' = 9 u 2
B 4 2
     v' = 3 u 0.36 × 0.3 = (0.36 + π × 0.6 ) × x
B 4 2
3
As v' B = u and v A = 1 u and 3 u > 1 u, spheres    x = 0.117 m
4 2 4 2
A and B will not collide again, so no further
collisions between the spheres.

73
©HarperCollinsPublishers 2018 Cambridge International AS & A Level Mathematics: Further Mechanics 9780008271893

71893_P067_080.indd 73 15/06/18 8:17 PM


SUMMARY REVIEW

20 O 21 i Consider forces vertically.


q a Resultant force, F = mg + T
T =
λ x = 19.2 × 1.5
l 1.2
R v m s–1 19.2 × 1.5
F = 0.4 × 10 + = 28
1.2
h q
Use F = ma
mg 28 = 0.4a
28
mg a= = 70 m s–2
0.4
ii Initial energy = PE + EE
Initially energy = KE = 1 m × 7 ga = 7 mga 2
2 2 4
= mgh + λ x
After motion has started, P also has PE due to the 2l
19.2 × 1.52
vertical height gained. = 0.4 × 10 × 2.7 +
2 × 1.2
Conservation of energy: = 10.8 + 18 = 28.8
7 1
mga = mgh + mv 2 At A, energy = 1 mv 2
4 2 2
7 mga = mg(a − acos q ) + 1 mv 2
= 0.2v 2
4 2
Conservation of energy:
v 2 = 7 ga − 2ga + 2gacos q
    2 0.2v 2 = 28.8
2 3 28.8
v = ga + 2gacos q    v2 = = 144
    2 0.2
2
Acceleration towards centre = v   v = 12 m s−1
r
22 i Use F = ma
mv 2 mv 2 dv
F = ma =
r
=
a 4e −x − 2.4x 2 = 0.8 × v
dx
2
Resolving forces towards centre: R = mv + mgcos q 4e −x 2.4x 2 dv
a − =v
0.8 0.8 dx
Substituting for v2 dv −x 2

( )
v = 5e − 3x
dx
R = m × 3 ga + 2gacos q + mgcos q
a 2 dv
ii v = 5e −x − 3x 2
3mg dx
R= + 2mgcos q + mgcos q
2 Separate and integrate.
3mg
∫v dv = ∫(5e
−x
R= + 3mgcos q − 3x 2)dx
2
 v 2
3mg = −5e −x − x 3 + c
2 (
R= 1 + 2cos q ) as required 2
Substitute initial conditions, i.e. v = 6, x = 0
i It loses contact with the sphere when R = 0
 18 = –5 + c, c = 23
1 + 2cos q = 0
v2
    cos q = − 1 = −5e −x − x 3 + 23
2 2
      v 2 = 3 ga + 2gacos q = 3 ga − ga = 1 ga Now substitute x = 2 to find the required
2 2 2 value of v.
    1 v2
v= ga = −5e −2 − 23 + 23
2 2
ii Consider energy when P passes through the v2
= 14.32
horizontal plane level with O. 2
2
   v = 28.64
1
mv 2 + mga
2   v = 5.35 m s–1
Conservation of energy: 23 First collision:
7 1 Let vA and vB be the speeds of A and B, respectively,
mga = mv 2 + mga
4 2
after the collision.
7
  ga = v 2 + ga1 Momentum before = momentum after
4 2

( )
2mu = 2mvA + mvB
   v = 2 × 7 ga − ga = 3 ga
2
2u = 2vA + vB
4 2
e × (speed of approach) = speed of separation
   3 −2
v= ga u = v A − vB
2 3
2u = 2vA + vB
74
©HarperCollinsPublishers 2018 Cambridge International AS & A Level Mathematics: Further Mechanics 9780008271893

71893_P067_080.indd 74 15/06/18 8:17 PM


WORKED SOLUTIONS

Adding gives:
24 i Horizontally, use s = ut x = ucos 45°t = 20 t
4 4 2
u = 3v A vA = u
3 9 x = 10 2 t
2u = 2vA + vB
Vertically, use s = ut + 1 at 2 y = 20sin 45°t – 5t2
2u = 2 × 4 u + vB 2
9
y = 10 2t − 5t 2
10 10
u = vB vB = u x
9 9 ii x = 10 2 t ∴ t =
10 2
Second collision (sphere B and the barrier): Substitute this into y = 10 2t − 5t 2
Let the speed of B be v'B after the collision.
2
e × (speed of approach) = speed of separation x  x  5x 2
y = 10 2 × − 5  = x − 200
10 10 2  10 2
e u = v' B
9 x2
y=x−
After A and B collide again, they move either in the 40
same direction or in opposite directions. iii Ball strikes the ground when y = 0
If they move in the same direction:
Conservation of Momentum:
0=x−
x2
40 (
= x 1−
x
40 )
 x = 40, so ball first strikes the ground 40 m
8mu 10meu
− = 2mwA + 5mwA from O.
9 9
8u 10eu 25 Centre of mass: distance from O = 2rsinα
− = 7wA 1 2α
9 9  on line/plane of symmetry
Newton’s Experimental Law:
2r sin π ()
(
2 4u 10eu
3 9
+
9 )
= 5w A − w A = 4w A
2r sinα

=
3 π
3
3
()
=
π
3r

6( 9 9 )
1 4u 10eu
+ =w 2 Take moments about O:

()
A
 π = 7.5r
For 15N force 15rcos
2 in 1 3

9

9
=
6 9
+(
8u 10eu 7 4u 10eu
9 ) π
( )
3
For centre of mass 20 cos 3 − q × π r

48u − 60eu = 28u + 70eu


π 3
At limit of equilibrium: 7.5r = 20cos 3 − q × π r( )
130eu = 20u
2
( )
cos π − q = 7.5 × π = 0.6802
3 20 3

( )
e= π − q = 0.8228
13
3
If they move in opposite directions:
q = 0.224 radians
Conservation of Momentum:
8mu 10meu 26 O a P
− = −2mwA + 5mwA
9 9
mg
8u 10eu
− = 3wA 1
9 9 
Newton’s Experimental Law:

( )
A
2 4u 10eu
3 9
+
9
= 5wA + wA = 6wA T q
P

3( 9 9 )
1 4u 10eu
+ = 3w A 2 mg

2 in 1 Initial energy = PE = mga

9 9 3 9
+(
8u 10eu 1 4u 10eu
− =
9 ) At base of motion energy = KE = 1 mv 2
Conservation of energy :
2

24u − 30eu = 4u + 10eu 1


mv 2 = mga
2
40eu = 20u    v 2 = 2ga
1 Circular motion about A, when angle is q
e=
2 Energy = 1 mv 2 + mg ( a − x ) (1 − cos q )
2
75
©HarperCollinsPublishers 2018 Cambridge International AS & A Level Mathematics: Further Mechanics 9780008271893

71893_P067_080.indd 75 15/06/18 8:17 PM


SUMMARY REVIEW

Conservation of energy: Conservation of energy


1 mv 2 + mg a − x 1 − cosq = mga 3.6 = 2.4 + 0.2v2
2 ( )( ) v2 =
1.2
=6
0.2
v = 2gx + 2gacos q – 2gxcos q
2
v = 2.45 m s–1

  = 2gx + 2g(a – x)cos q
2
Acceleration towards centre = v 28 i Resultant force = mg – 0.8v = 2 – 0.8v
r Use F = ma
v2 mv 2
F = ma = m = 2 – 0.8v = 0.2a
r (a − x) 2 0.8v
a= − = (10 − 4v) m s–2
Resolving forces towards centre: 0.2 0.2
2
T = mgcosq + mv ii Use a = dv
(a − x) dt
m(2gx + 2g ( a − x ) cosq ) dv = (10 − 4v)
T = mgcosq + dt
(a − x)
1 −4 dv
−4 ∫ 10 − 4v
2mgx = ∫dt
T = mgcosq + + 2mgcosq
a−x
−1
2mgx ln (10 − 4v ) = t + c
T = 3mgcos q + 4
a−x
v = 0 when t = 0
T = mg(3cos q + 2x )
a−x    −1 ln 10 = c
4
( )
P completes a circle if T ⩾ 0 at the top of the −1 1
motion; for minimum value, T = 0 at the top of the ln (10 − 4v ) = t − ln (10 )
4 4
circle, which is when q = π radians. When t = 0.6
0 = mg(3cos q + 2x ) −1 1
   a−x ln (10 − 4v ) = 0.6 − ln (10 )
4 4
2x
3=    10 – 4v = 0.9072
    a − x
3a – 3x = 2x         v = 2.27 m s–1
  3a = 5x 29 Conservation of momentum
x 3 Momentum before = momentum after
=
   a 5 8mu – 3mu = 2mvA + vB
27 i Tension = 0.4g = 4 N 5mu = 2mvA + vB 1
Use Hooke’s law T = λ x Coefficient of restitution
l
16 × x e × (speed of approach) = speed of separation
4=
0.8
–e(4u + 3u) = vA – vB
  x = 0.2 m
ii When P comes to rest all energy is elastic, –7eu = vA – vB 2
16 × ( 0.6 )
2 2 1 + 2
EPE = λ x =
2l 2 × 0.8 5mu – 7eu = 3vA
When P is projected, PE = mgh =  0.4 × 10 × 0.4 1
v A = (5u − 7ue)
= 1.6 3
vA < 0 if 5u – 7ue < 0
1 2 1 2
KE = 2 mv = 2 × 0.4u 5u < 7ue
5
PE = λ x = 16 × ( 0.2) = 0.4
2
2 e>
7
2l 2 × 0.8
Conservation of energy
Extension questions
3.6 = 1.6 + 0.2u2 + 0.4
1.6 = 0.2u2 1 Let h be the ‘height’ of the cone.
u2 = 8 2 2π
Volume of the hemisphere = π ( 0.1) =
3
−1 3 3000
   u = 8 = 2.828427… = 2.83m s
iii When string becomes slack, Volume of the cone = 1 π ( 0.1)2 h = π(h)
3 300
PE = mgh = 0.4 × 10 × (1.4 – 0.8) = 2.4
1 1 Mass = density × volume
KE = mv 2 = × 0.4 × v 2 = 0.2v 2
2 2
EPE = 0 (as there is no extension x = 0)

76
©HarperCollinsPublishers 2018 Cambridge International AS & A Level Mathematics: Further Mechanics 9780008271893

71893_P067_080.indd 76 15/06/18 8:17 PM


WORKED SOLUTIONS

Moments about the common circle: For particle A

3000( )(
D × 2π × 3 × 0.1 = D × πh × h
8 300 ) 4 ( )() 1
After collision KE = 2 m (1.1 2gl )2

0.6 h2 At point at which it comes to rest PE = mgh


=
24 000 1200 where h = height above point of collision
h2 = 0.03 h = l(1 – cosq )
h = 0.1732 m Conservation of energy:
For the hemisphere end: 1
× m(1.1 2gl )2 = mgh
r = 0.1 so v = 0.1 × 4 = 0.4 m s−1 2
1.21l = h
For the cone end:
r' = 0.1732 1.21l = l(1 – cosq )

v = 0.1732 × 4 = 0.693 m s−1 cosq = –0.21


q = 102°
2 i Before collision for A initially: PE = mgl
3 For the initial fall: using v2 = u2 + 2as
Vertically below O, before collision KE = 1 mv 2
2 u = 0 m s–1, a = 10 m s–2, s = 20 m, v = ?
Conservation of energy:
1 v2 = 2 × 10 × 20 = 400
mv 2 = mgl
2 v = 20 m s–1
v2 = 2gl
v = 2gl The speed at which the ball leaves the ground
is equal to the speed at which the ball hits the
Similarly for B PE = 3mgl ground again when it comes back down.
KE = 1 (3m)v 2 For a height of ⩽ 5 m:
2
1 3m v 2 = 3mgl and similarly v = 2gl u = ?, a = –10 m s–2, s ⩽ 5 m, v = 0 m s–1
2( )
02 ⩽ u2 + 2 × (–10) × 5
ii e(speed of approach) = speed of separation u2 ⩾ 100
( )
0.4 2 2gl = v A + v B
u ⩾ 10 m s–1
0.8 2gl = v A + v B By Newton’s experimental law: v = 0.85u
Conservation of momentum So after n bounces, the speed of the ball
(left as positive) immediately after the bounce is given by:
Momentum before = momentum after vn = 20 × 0.85n
−m 2gl + 3m 2gl = mv A − 3mv B For the height of the bounce is to remain below
5 m, vn ⩾ 10.
2 2gl = v A − 3v B
20 × 0.85n ⩾ 10
Solving the two equations leads to
0.85n ⩾ 0.5
v B = −0.3 2gl ln 0.85n ⩾ ln 0.5
v A = 1.1 2gl n ln 0.85 ⩾ ln 0.5

iii For particle B n  ln 0.5


ln 0.85
1
After collision KE = 2 3m (0.3 2gl )2 n ⩾ 4.265
At point at which it comes to rest PE = 3mgh So the ball will remain below 5 m after 5 bounces.
where h = height above point of collision
4 Use F = ma
h = l(1 – cos q )
x2 dv
= 2v
()
Conservation of energy: dx
sin v
1 × 3m (0.3 2gl )2 = 3mgh 2
2
Separate and integrate

()
0.09l = h
v
∫ x dx = 2∫vsin 2 dv
2
0.09l = l(1 – cos q )
cosq = 0.91
Integrate the RHS using integration by parts
q = 24.5°

77
©HarperCollinsPublishers 2018 Cambridge International AS & A Level Mathematics: Further Mechanics 9780008271893

71893_P067_080.indd 77 15/06/18 8:17 PM


SUMMARY REVIEW

3  () ()
x 3 = 2  −2vcos v − − 2cos v dv 
2 ∫ 2 
2
40
1
( 2 ) ( 2 ) 
3 x
x = 2  −2vcos v + 4sin v  + c 40
3  1
T 40
( 2) (2)
3 0.5 sin 45
x = −4vcos v + 8sin v + c
3 = 1
0.25 mg 2 2
Substitute in the given initial conditions, i.e. when
t = 0, v = π, x = 0

() ()
45°
0 = −4π cos π + 8sin π + c
2 2
0.5 cos 45
c = –8
= 1
x3
3
= −4vcos () ()
v
2
+ 8sin
v
2
−8 2 2
Relative to O, point of launch has coordinates
x = 3  −4vcos ( ) + 8sin ( ) − 8 
3 v v
 1 
 2 2   − 1 , 1 − 1
2 2 40 2 2 40 
x = −24 + 24sin ( ) − 12vcos ( )
3 v v
2 2 Therefore equation of trajectory relative to O is
2
5 Initially, elastic energy =  1 1  4 1 1  1 1
y = x − +  − x − +  + −
λ x 2 = 10 × 0.25 × 0.25 = 0.625  2 2 40  5  2 2 40  2 2 40
2l 1
6 i If P and Q collide, l < 4.2 m
At point of projection all energy is converted to KE
ii For particle P:
1 2
mv 2 = 0.625 Energy at P = λ x = 1
2 6
2l
v 2 = 12.5
Energy at A = 1 mv 2 = 1 due to conservation of
v = 12.5 2 6
energy
For motion from point of projection
3
Horizontally x = 12.5 × cos 45° × t = 2.5t v=
6
For particle Q:
x = 5 t , or t = 2 x
2 5 2
Vertically using s = ut + 1 at 2 Energy at Q = λ x = 5
2 2l 6
1 Energy at B = 1 mv 2 = 5
y = 12.5 sin 45° t − 10 t 2 2 6
2 5
v=
y = 2.5t – 5t2 3
4 e(speed of approach) = (speed of separation)
Substituting for t, y = x − x 2
5  3 5
0.7  + = v A + vB
This is the equation of motion for the particle from  6 3 
the point of launch. Momentum before = momentum after
For maximum speed, acceleration = 0 3 5
×4− × 3 = 3v B − 4v A
F = ma 6 3
T – mgcosq = 0  3 5
4 × 0.7  + = 4v A + 4v B
 6 3 
λ x = mgcosq
l solve simultaneously for vB leads to
10x 1 vB = 0.2591
= 0.1 × 10 ×
0.5 2 For Q: energy after collision = KE

x=
1
=
2
v = 1 mv 2 = 1 × 3 × (0.2591)2 = 0.1007
2 2
20 2 40
When Q stops moving all energy is elastic,
λ x 2 = 2x 2 = 0.1007
2l 2.4
x = 0.3476
Distance from B = 1.2 + 0.34766…
78 = 1.55 m (3 s.f.)
©HarperCollinsPublishers 2018 Cambridge International AS & A Level Mathematics: Further Mechanics 9780008271893

71893_P067_080.indd 78 15/06/18 8:18 PM


WORKED SOLUTIONS

7 Use F = ma
9 Use s = ut + 1 at 2
2x dv 2
e sin 2x = 4v At collision height, for P:
dx
Separate and integrate s = uT – 5T 2
∫ 4vdv = ∫ e2x sin 2xdx At collision height, for Q:
s = 2u(T – 2) – 5(T – 2)2
For the RHS use integration by parts twice.
e 2xsin 2x e 2xcos2x At collision height
2v 2 = − +c
4 4 uT – 5T 2 = 2u(T – 2) – 5(T – 2)2
Substitute initial conditions, x = 0, t = 0, v = 2 uT – 5T 2 = 2uT – 4u – 5T2 + 20T – 20
1
8 = 0 − + c , c = 8.25 4u + 20 = uT + 20T = T(u + 20)
4
e 2xsin 2x e 2xcos2x 4u + 20
2
2v = − + 8.25 T =
4 4 u + 20
10
e 2xsin 2x e 2xcos2x 33
v= − + 5
8 8 8
5 sin 60 =5 3
8 Initial energy
2
= 1 mv 2 + mgh = 1 × 100 × u 2 + 100 × 10 × (5sin 30° + 5)
2 2 60°
= 50u2 + 7500 5 cos 60
At point of collision = 5
2
energy = 1 mv 2 = 0.5 × 100 × v 2 = 50v 2
2 8
8 sin 30 = 4
Conservation of energy
30°
50v2 = 50u2 + 7500
8 cos 30
v2 = u2 + 150 =4 3
e(speed of approach) = speed of separation Use s = ut horizontally and s = ut + 1 at 2 vertically
2
2
e u + 150 = vQ + v P 5
For P horizontally x = t
2
e u 2 + 150 = 5u + v P 5 3
vertically y = t − 5t 2
2
Conservation of momentum For Q horizontally x = 4 3t
Before = after vertically y = 4t – 5t2
2
100 u + 150 = 5vQ − 100v P When P and Q collide the time of flight for Q is t
and time of flight for P is t + T
2
100 u + 150 = 25u − 100v P
 5 (t T ) 
 4 3t   + 
Eliminate vP using the equation above for e. 2
  = 
100 u 2 + 150 = 25u − 100 e u 2 + 150 − 5u 
2
 4t − 5t   5 3 (t + T ) − 5(t + T )2 
   2 
100 u 2 + 150 = 25u + 500u − 100e u 2 + 150 2
Using the x component (t + T ) = 4 3t
5
100 u 2 + 150 + 100e u 2 + 150 = 525u Substitute this into the y component

(1 + e )100 u 2 + 150 = 525u ( ) (


4t − 5t 2 = 5 3 2 4 3t − 5 2 4 3t
2 5 5
2
)
525u 4
(1 + e ) = 4t − 5t 2 = 12t − × 48t 2
5
100 u 2 + 150
0 = 8t – 33.4t2
525u
e= −1 0 = t(8 – 33.4t)
100 u 2 + 150
Solving, t = 0.2395

79
©HarperCollinsPublishers 2018 Cambridge International AS & A Level Mathematics: Further Mechanics 9780008271893

71893_P067_080.indd 79 15/06/18 8:18 PM


SUMMARY REVIEW

2 Substitute v = 4 when x = 3 65
Substitute into (t + T ) = 4 3t
5
( )
( 52 × 4 3 − 1)t
2 = ln 3 65 + 24 + c
T =
c = 2 − ln(24 + 3 65 )
T = 0.424 s
2
11 i For a projectile the range of flight u sin 2α
2
g 8 ( )
v 2 = ln x + x 2 − 9 + 2 − ln(24 + 3 65 )

For particle A, range of flight = u When x = 6


10
For particle B, range of flight =
u2
10
u2
8 ( )
v 2 = ln 6 + 6 2 − 9 + 2 − ln(24 + 3 65 )

Given that the particles collide, 0 < l < 5 v2 = 4.324


ii Horizontal speed before collision =  10cos 45° v = ±2.08 m s–1
= 10
2
Horizontally:
e(speed of approach) = speed of separation
 10  5
0.5  2 ×  = v P + vQ v P = vQ =
 2 2
Collision takes place 7 m from O
Consider horizontal motion

s = ut 7 = ucos 45°t, 7 = 10 t i.e. t = 7 2


2 10

Collision takes place 7 2 seconds after


10
particles are projected.
Total time of flight – consider vertical motion
s = ut + 1 at 2
2
 10 
0 = 10sin 45°t − 5t 2 = t  − 5t 
 2 

t = 0, or 5t = 10 ie t = 2 seconds
2
Time of flight left after collision takes place
= 2 −7 2
10
Horizontal distance covered in this time
  5 35
=  2 − 7 2 × =5− = 1.5
 10  2 10
Both particles cover this distance, ∴ distance
between A and B is 3 m
4
12 a =
x2 − 9
v v =
d 4
dx 2
x −9
Separate and integrate
1 v dv = 1
4∫ ∫ x 2 − 9 dx

(
v 2 = ln x + x 2 − 9 + c
8 )
80
©HarperCollinsPublishers 2018 Cambridge International AS & A Level Mathematics: Further Mechanics 9780008271893

71893_P067_080.indd 80 15/06/18 8:18 PM

You might also like